Download as doc, pdf, or txt
Download as doc, pdf, or txt
You are on page 1of 268

CPHQ Questions

1 in order for a quality improvement team to deal effectively with


conflict ,it is important to appoint which of the following to its
membership
A. facilitator?
B. human resources representative
C. risk manger
D. senior leader ?

2 The use of clinical path ways and guidelines in hospitals should


do which of the following?
A. minimize variation in patient care
B. reduce length of stay
C. improve patient satisfaction
D. identify errors in patient care

3 A critically ill patient is admitted and requires a specialist


procedure .however; the surgeon does not have the privilege at the
facility. Which of the following documents will be most helpful
identifying the course of action? The hospital should take?
A. patient safety manual
B. risk management plan
C. medical staff bylaws
D. surgical policies and procedure

4 Which of the following step is the first step in the strategic


planning process?
A-setting goals and objectives?
B-defining organizational structure?
C-determining productivity indicator
D-establishing and controlling a budget

5 which of the following national patient safety goals is applicable


to everyone in health care facility
A-communication
B-medication safety
C-health care related associated infection
d-reconcile medication

1 of 268
6 The best evaluation of performance improvement plan is
a- Process improvement
b- measurable objectives
c- applicable deliverables
d- timeline

7 quality assurance focused on

1 APerformance of individuals

2 Plan Apprisal

3 System?

4 Process?

8 quality performance improvement focused on

1 Process?

2 System?

3 Individual

4 steps

9 according to pareto principle, in the era of quality management 80


of the problem present in

1 System

2 Performance

3 Individual

10 the function which describes the assessment of educational


needs is

1 Quality improvement

2 Utilization management

3 Risk management

4 Process map

2 of 268
11 the most appropriate to evaluate case manager

1 los

2 nurse setting referral

3 patient complaints

19 strategy of brain storming

A. Discussion of issues

B. Prioritization of issues

C. Recording of issues

20 main function of clinical pathway is

A. Convert medicine into mathematics

B. Monitor variations

C. Limit Dr intervention

21 a 75 y patient with rat side hip replacement climbed upstairs and


had been fallen, as a risk manager before the bill writing what type of
risk do you consider this event
A. Risk resistance

B. Loss reduction

C. Potential compensable event ?

22 the tool that keeps the team on track to follow all tasks

A. Business plan?

B. Gannet chart ?

C. Flowchart

D. Control chart

23 the best way to display LOS variation is

A. Line graph

B. Histogram

C. Scatter diagram
3 of 268
D. Pare to chart

24 the best way to communicate the reduction of falls results

A. . Bar graph shows the best 2 units achieved the goal

B. . Line graph shows the falls during last two years

C. It's better to hide such data

25 SBAR is a tool for

A. Calculate average LOS

B. Statistics between absolute averages

C. Improve communication between care givers

D. Financial control

26 when 2 facilities have a network, what is the most cost effective


issue increases the patient satisfaction of the services
A. When the patient don't have to repeat investigations in other facility

B. When his satisfaction survey results are collected from both facilities on
the fly

C. When both facilities have equal staff competencies

D. When the network allows doctors to communicate through the internet

27 first tool to use in Root cause analysis is

A. why diagram

B. Flow chart

C. Gannet chart

D. Cause and effect diagram

28 0-occurance 100 % analysis is

A. Occupancy report

B. Sentinel event

C. Cause and effect analysis

D. Cost benefit analysis

29 vision statement represents the organization


4 of 268
A.Domain of action for a year

B.Why does it exists

C.Future

30 the most appropriate design for strategic management is

A.Vision mission core values goals

B.Masterplan, business plan and departmental objectives

31 The most appropriate sequence for strategic management


implementation is

A.Strategic goals mission vision?

B.Vision mission values goals?

32 it's noticed that increase aggressive behaviour among psychiatric


patients , what is the appropriate action

A.Trend data over time

B.Focus group with end user

C.Review restrains policy

33 the team member that keeps team on track and clarify issues

A.Leader

B.Member

C.Facilitator

D.Time keeper

34 the member - group responsible for continuous improvement of


organization

CEO

Quality council

Share holders

Governance board

35 the ultimate responsibility of implementation of quality relays on

5 of 268
CEO ?

Quality council?

Share holders

Governance board

36 the quality professional reviews the surgical consents and found


allot of incompetent consents, who is the first person to communicate

CEO

Information management professional

Director of nurses (DON)

Chief surgeon

37 When comparing your performance against field top leaders, this


is considered as?

Best practice

Benchmarking

Severity adjustment

Setting goals

38 to protect your organization against unannounced surveys the


most important to keep in your organization

Continuous readiness

All plans unannounced

Patient medical records for 3 months only

Copy of all incident reports

39 just culture means

A. Culture of no abuse

B. Blame free culture

C. Quality culture

D. A culture of timeliness

40 last step in FMEA methodology is


6 of 268
A. Analysis of effects

B. Measure results

C. Keep monitoring

D. Results reporting

41 first step in strategic management is

A. External and internal analysis

B. Vision

C. Setting goals

D. Setting measurable objectives

42 hushing planning most valuable step is

A. Roll down of strategies to departmental level

B. Annual reporting

C. Use of PDCA

D. Prioritization of goals

43 if your department has contract with another facility to provide a


risky service this is considered as
a. Risk shift
b. Risk adjustment
c. Claim against you from 3rd party
d. Negligence
44 Hospital Acquired infections HAI are considered as
a. Sentinel events
b. Outcome measurement
c. Process measurement
d. Near misses

45 Z-Score represents:
a. Standard Deviation
b. Average
c. Ultimate goal
d. Regression value

When a major processes redesign is needed, the most appropriate -47


approach is
A. Quality circle PDSA
B. Total quality management

7 of 268
C. Six sigma
D. Continues quality improvement

48-the information gathering technique known as focus group include


all of the following except:
a- provision of written survey information

b- identification of customer requirements

c- solicitation of perception in meeting requirements

49-Design of piece of device make errors. This type of error is:


a-active
b-negligent
c-latent
d- organizational

51-A healthcare organization wants to adopt concurrent review process


instead of retrospective review. To facilitate this changes the first to be
inspired are
A-Leaders?

B-Mangers?

C-Physician

D-Nurses

52-In a hospital using quality improvementstrategies, the number of


overall patients last quarter was 1000 patients,100 admissions with
average LOS (Length of stay) 5 days. The interdisciplinary improvement
team goal is 20% decline in LOS. How many admissions will be gained
by reaching this:

A.80
B.200
C.400
D. 300

53-When a surveyor reached for regulatory visit, the first question that
the quality professional will ask for is

A-Please let me see your identification.

B-Let us sit to schedule for your visits.


8 of 268
c-Which area of compliance would you like to review.

54-When hiring a specialist from a consulting firm to evaulate perfor -


mance improvement program ,as a quality professional what u should
do?
a. Schedule the program for the consulting firm.?
b. identify areas with problems to the consulting firm.?
c- identify outcomes? ( search the correct answer)

55-you are goining to make major changes in the in the organization. To


increase the acceptance of the staff to change:
a.medical staff education.
b.long rang plan.
c.pilot project

56-.which of the following is the most useful in mortality /morbidty review:

a.LOS.

b.autopsy.

c.physician profile.

57-A health organization is becoming a member in a health plan. The


most important educative program to be delivered to the staff is

a-organizational change?

b. conflict of interest

c. consumer needs?

d. accreditation needs

58-Which of the following charts is used to institute quality improvement


& monitor cost reduction on ongoing basis?
A. Pie chart
B. Control chart
C. Pareto chart
D. Fishbone diagram

59- Patient safety indicators are developed by AHRQ are designed to be


useful in all of the following except
A. Collect patient safety data
9 of 268
B. Support root cause analysis
C. Monitor change initiatives post implementation
D. Monitoring of the staff performance

60- To encourage adoption of quality improvement concepts in day work


you do all the following except
A. Distribute quality newsletter
B. Make quality lectures
C. Distribute external quality papers within the organization
D. Ask the physicians to make self study on quality improvement

61- a large facility has fostered a culture of patient safety through staff
education , support of process improvement ,department levels of
implementation of non-punitive approach to error reporting compliance
with patient safety goals ranges from 75-100% in assessing culture of
patient safety cphq should:

a-survey of all employee &physician

b-survey pt last 6 m

c-review collected data through incident reporting

d-review post surgical infection rate data

62 -Even when appropriate process are in place , error can occur,


understanding this, leader coordinating any safety program should
focus on:

a-pt survey

b-time constrain

c-policies

d-performance feed back

63- When choosing an outside consultants to lead employee focus


group, what priority areas of expertise should cphq look 4?

a-Team Development & Management

b-organizational assessment & change management

10 of 268
c-group dynamics & facilitation

d-organizational design & reengineering

65-A psychiatric hospital is reporting a significant level of patient


aggression as a quality professional the appropriate action to make is:
A. Generate a policy of restraining all patients
B. Switch from physical to chemical restrain
C. Adopt restrain free policy
D. Make a system of early identification of patient characteristics
may be indicative of aggression

66- Benchmarking is based in identifying


A. Best practice
B. Competition
C. Deficiencies
D. Statistical control

67- Which of the following is most helpful in integrating data collected in


the performance improvement process?

A. Discussing performance improvement findings with senior


management
B. Developing a performance improvement prioritization matrix
C. Creating a scatter diagram using the data
D. Integrating the data based on team consensus

68- According to quality assurance point of view, the main focus is on:

A-System
B-Individual
C-Processes
D – Customer
Answer: B

 Quality assurance focus on monitoring the What of care? (Patient


care given) and Who of care? (Patient care giver), so the answer is B
"INDIVIDUAL”. If we add the How of care “patient care process" and the
Result of care “patient care outcomes”, It will become quality
improvement not quality assurance. This explanation as per Janet A.
Brown which I do agree with

69- the physical difference between traditional QA & QI that QI is:

a- focus on process& systems while QA focus on individual faults


11 of 268
b- stress peer review while QA focus on customer

70-.the best tool to display length of stay variation over the last 5 years :
a.pareto chart.
b.control chart.
c.regression anaylsis.

d.bar gragh

71- which of the following tool could be used to assess the effeciency of lab
speceimen processing?

a. pareto chart.

b. regression analysis.

c. profit and loss sheet.

d. productivity index.

73-one of the team members hesitates to share in team discussions. as a


team leader what should you do?

a. offer to help the member to prepare the data.

b. exclude the member from the team.

c. tell the member the excpectation to contribution to discussion.

d- ask him in non decisive issues

74-CPHQ try to improve care through accurate definition of indicators


which of the following indicators reflect performance of surgeons:

a) No of patients referrals to ICU after surgery / no of all surgeons.


b) No of patients admitted to ICU /total no of surgeries.
c) NO of patients referral to ICU after minor surgery / no of minor surgery.
d) No of patient admitted to ICU / no of major surgery.

75-appropriateness of appendectomy

A. preadmission test
B. pathology test
C. age
D. clinical test

76-critical value determined by

a-the regulations and laws

b-accrediting body

12 of 268
c-organization

77-Which of the following are attributes to culture of safety?

A- Transparency & increased patient acuity level

B- Error –proof environment & empowered staff

C- Empowered staff & transparency

D- Increased patient acuity level & error-proof environment

78-In order to facilitate development of strategic plan, what to consider


first?

A-Cultural assessment, planning, implementation & evaluation

B-Risk management, gap analysis, identification of organization wide


functions

C-Mission, vision, values , short & long term goals & objectives

D-Creation of master plan, identify customers, goals & objectives

79-When facility make development of clinical indicator criteria,


Healthcare quality professional should:

A-Selecting indicators that are approved by accrediting organization

B- Selecting indicators that are approved by Payers

C-develop criteria that reflect processes &outcomes

D-develop criteria that reflect department policies

80-An organization leader has directed a Healthcare Quality Professional to


measure the success of a corrective action plan on patient care planning. The
organization leader wants to be at least 95 % confident of the accuracy of

13 of 268
results. The average daily census at the organization is 1000 patients. The
most accurate & efficient sampling technique for this study would be:

A- Review 100% of all active records on one day of past month

B-review 10% of all discharge records for the past quarter

C-estimate the percentage of records to be reviewed using an accepted


statistical formula appropriate for the population

D- Identify 30% of all records that failed preliminary care plan compliance
review

81-A healthcare organization implementing ongoing performance


improvement, which of the following would most likely benefit the PI goals of
the organization?

A- A system selected by middle & senior management resulting from


proposals by consultants

B-Cross functional processes evaluated by multidisciplinary teams with


support of management

C-Discrete systems relevant to & monitored by individual department

D- Comprehensive process developed, implemented & monitored by the QM


department

82-Problem solving, cross functional understanding, expanded area of


expertise, gain in status & power & increase span of knowledge are examples
of

A- The benefits of teams


B- Resource requirements
C- Customer expectation
D- Strategic alliance
83-Comparison of surgeon specific infection rate is example of

A- Practitioner profiling

14 of 268
B- Root cause analysis

C- Practitioner credentialing

D- Incident analysis

84-All of the following conditions contribute to system improvement except

A- Measuring performance of processes & their outcomes using valid


statistics methods
B- Taking action to improve the way the processes are designed &
carried out
C- Studying & understanding the complex process that contribute to care
D- Identifying & Responding to individual performance issues
85-Healthcare quality professional has written patient safety plan that
includes: purpose, goals, objectives (scope & processes)

A review of outcomes data has been completed, which of the following


additional information should be in the plan :

A- Disaster preparedness

B- Steps to improve patient satisfaction

C- Equipment management

D- Efforts to reduce harm

86-Analysis of events, trends & customer needs ……is the initial phase of

A- cost benefit analysis


B- strategic planning
C- forecasting
D- resource allocation
87-Comparing Healthcare organizations by using medical error rates:

A- may present bias due to difference in reporting practices


B- provide best method for benchmarking patient safety
C- can't be performed by facilities less than 100 beds
D- must include minimum of 10 facilities
88-The best tool to begin investigate causes of laboratory labeling errors

15 of 268
A- histogram
B- flowchart
C- affinity diagram
D- prioritization matrix
89-Organizational continuing education needs are best identified through what
type of activities

A- UM
B- QI
C- Budgeting
D- HR
90-Clinical pathway identifies all of the following except:

A- Patient satisfaction
B- Better & Best Practice
C- Cost survey
D- Outcome of processes
-------------------------------------------------------------------

91- Arrange ;

1-Gathering data

2-Evaluate effectiveness & improvement

3-Make commitment

4-Implementation

A- 2- 1- 3 -4
B- 3- 2- 1- 4
C- 1 – 2- 3 -4
D- 3 -1 -4 -2
92-A poster contain information will most effectively convey outcome
information to internal customers?

A- 2 Bar graphs showing the 2 unites with fewest number of falls over past
year

16 of 268
B- (Patient fall decreased over 4 years) printed above a line graph showing
percentage of falls to patient days

C- Patient fall indicate downward trend. Go for team!

D- (Patient fall last year were 0.5% of patient days) printed to photograph of
the organization staff

93-The annual evaluation of QI process must

A- be accomplished by healthcare quality professional


B- document all problems identified in care / service
C- be based on organizational objectives
D- survey all departments & teams

94-The chief executive officer of healthcare organization has requested a


recommendation for the most effective method of assessing the organization's
readiness to adopt CQI, which of the following methods should CPHQ
recommend:

A- review aggregate results of employee performance appraisals


B- Hire a consultant to conduct personal interview of staff
C- Conduct leadership ''walk through'' of the organization
D- Administer surveys to evaluate organization culture.

95-The medical record manager reports that authentication of verbal orders


occurs 25% of the time , as compared to a reported 85% in situations ,which
of the following is the initial action for the manager to take ?

A- Recommend continued measurement of the indicator.


B- Share the data with the medical staff
C- Organize a PI team
D- Recommend improvement strategies
96-An effective risk management plan includes all of the following except:

17 of 268
A- description of educational programs
B- statement of purpose
C- description of reporting mechanisms
D- scope of the program

97-Which of the following is true regarding medical error

A- prevented by review of evidence based practice


B- caused by gaps between patient expectation & practice
C- avoided by uniform practice
D- associated with process failure

-------------------------------------------------------------------

98-Primary purpose of integration of financial & quality management

A-develop physician profiles

B-Identify potential cash flow problem

C- Identify problem in resource management

D-Determine medical necessity of treatment

99-Developing educational training program in QI , What component


should be included :

A- quality definition & principles


B- performance appraisal results
C- discussion of incidents
D- Individual focus of activities
100-The best way to evaluate effectiveness of performance improvement
training is through

18 of 268
A- self assessment
B- participants' feedback
C- observed behavioral changes
D- post-test results

101-In evaluating length of stay & outcome data on cardiac cathertization.


HealthCare quality professional identified direct relationship between Adverse
outcomes & physician practice pattern. This integrated approach involves
correlating

A- case/care management & finance


B- UM & QM
C- Finance & UM
D- Discharge planning & QI
102-A Performance improvement program for supervisors should include

A- Rapid cycle process

B- Results of FMEA

C- Budget variance reporting

D- Review of patient falls

103-Conclusions from statistical study generalized to

- sample
- population

105-When choosing an outside consultant to lead employee focus groups,


what priority areas of expertise should CPHQ look for?

a- team development & management

19 of 268
B-Organizational assessment & change management

C-Group dynamics & facilitation

D-Organizational design & re-engineering

106-In reviewing medical records in 1st& 2nd quarters of year

surgical time out performed 9 95


0

communication of critical results 9 95


1

pain score used 5 60


0

initial patient assessment performed 5 45


2

Which to be reviewed??

107-Responsibility of Patient safety:

A- Entire staff
B- Senior leader
C- Patient safety officer
D- Medical executive officer

108-To assess culture:

A- Incident reports of all physicians & employees.

B- Facilitator expectation & records of team efforts.

C- Health insurance company contract.

109-The greatest fear from using electronic medical record system

A-Data sabotage

B-Process reboot
20 of 268
110-Organization committed to improving patient safety, Key areas to
influence change are

A- Staff willingness to change, policies& procedures, redesign the


structure & improvement of morale.
B- Structure, environment, equipment, process, application, leadership,
culture.
C- Medication delivery,structure,staff willingness to change, on-slip floors,
improvement of wards , environment
D- Leadership ,culture ,policies& procedures ,staff incentives, better
lightening , evaluation of processes & Electronic medical records

111-Successful development of clinical practice guidelines:

A-Physician involvement

B-Staff education

C- Quality improvement tools

D-Patient education

114-Quality improvement activities should be considered for all except:

A-Compatibility with facilities mission

B-Ease of development of data collection tool

C- Processes that are high volume for the facility

D- Findings from patient satisfaction surveys

115-Deploying a CQI team would be first approach in addressing which:

A-Several patient complained their call lights not answered during night shifts

B-Several physicians don't allocate enough time for procedures which booking
surgical cases

21 of 268
C- Finance billing outpatient procedures as ambulatory surgery

D- Results of preadmission testing for inpatient survey are unavailable 35% of


time causing delays.

116-Choosing software for physicians, what is the role of healthcare


quality professional?

A- assess use of technology


B- Cost-benefit analysis
C- Focus group with end users
117-Generic screening is a type of risk:

A- Identification
B- Evaluation
C- Reduction
D- Intervention

118- A surgeon’s wound infection rate is 32%. Further examination of


which of the following data will provide the most useful
information in determining the cause of this surgeon’s infection
rate? :

A.mortality rate

B.facility infection rate

C. use of prophylactic antibiotics

D. type of anesthesia used

Explanation:

A. Mortality may be a sequence of infection or any other known or


unknown reason. It is not a definite or correlated CAUSE.
B. Facility infection rate is a global scheme that includes all inpatient rates
and not specific only to post surgical wound infections.
C. Use of prophylactic pre and post surgical antibiotics is a an
accreditation prerequisite and patient files must be reviewed preliminary to
ensure compliance with clinical pathways and ascertain surgeon’s
competency.
D. There is no correlation between postsurgical infection rates and type of
used anesthetic.

22 of 268
119- The Balanced Scorecard answers which questions? :

a. "How are we going to get there?" "Which way do we go?"


b. "How are we doing?" "Are we there yet?"
c. "Where are we going?" "What are we doing?"
d. "Why are we here?" "What is our purpose?"

120-If a sphygmomanometer was used to measure the blood pressure of


one patient, the blood pressure at the first trial was 110/70; one minute
later, the blood pressure of the same patient was measured by the same
nurse and the result was 180/120; then this measurement tool can be
described as:

a) unreliable
b) invalid
c) unreliable and invalid
d) incorrect
Explanation:
A. The first requisite for blood pressure determination is a correctly
calibrated manometer. Reliability is the consistency of measurement, or
the degree to which an instrument measures the same way each time it
is used under the same condition with the same subjects. In short, it is
the repeatability of measurement under same conditions.

B. Validity is the relationship between treatment and observed outcome.


It involves the degree to which you are measuring what you are
supposed to. The more consistent is the relationship the more degree of
validity. A Sphygnomanomer is made to measure Blood Pressure and it
does so, this means that it is valid for the purpose. It doesn’t measure
temperature or process food. Its accuracy, dependability or consistency
is an issue that can be handled differently.

C. See explanations in both A and B.

D. Incorrect describes the reading not the device. By logic, either one or
both readings are incorrect. So the answer is partially true but if
remained with answer A, it will be dropped out since answer A is more
robust. If I were to set the question, I would prefer not to put this
distractor but rather replace it with other terms such as unpleasant,
unsafe, uncertain or flexible. In such kinds of question, my advice is just
(go with the flow) and don’t make much fuss. Just pass the exam their
way then after certification, act your better own one ( think global and
act local).Equally remember that USA is a nation of immigrants and
English is very likely to be a second language and terms may not
interpreted as precise as they should be.

23 of 268
121-Reengineering QI in a newly merged multi-specialty medical group
practice represents which process in the quality/performance
improvement function? :

a. Planning/design
b. Measurement/monitoring
c. Assessment/analysis
d. Improvement/re-monitoring

122-Under the quality improvement paradigm, which statement is


incorrect?

a. The focus is on the competency of individual practitioners.


b. The focus is on the efficacy and effectiveness of processes.
c. The focus is on the patient.
d. The focus is on organization performance.

123-Choose the best indicator(s) to use when determining the


effectiveness of services at the emergency department (ED) in your
hospital:

a) The number of unplanned returns to the ED


b) The percentage of ED patients admitted as inpatients
c) The patient waiting time at the ED
d) a and b

124-An accurate sequence of team formation and growth would begin


with “forming”, then:

a. norming, performing, storming.


b. performing, norming, storming.
c. storming, norming, performing.
d. storming, performing, norming.

125-That dimension of quality/ performance that is dependent upon


evaluation by the recipients and/or observers of care is:

a. respect/caring.
b. safety.
c. continuity.
d. availability.

 The recipients of service evaluate its quality mostly by level of


respect and care they receive from care providers, that's what matters to
them; they wouldn't be really able to evaluate technical details,
guidelines, competency or other dimensions of quality.

24 of 268
126-Incorporating TQM key concepts, compartmentalization of QM/QI
activities by organizational structure, i.e., by department or discipline,
is:

a.weakness in implementing quality improvement .


b. the most efficient structure.
c.consistent withTQM philosophy.
d. important for preservation of medical stuff autonomy.

127-Whenever a quality problem has multiple causes, just a few of those


causes account for most of the incidents is an expression of:

A. Deming's first principle


B. Deming's third precept
C. the Pareto principle
D. the quality improvement principle

128-The concept of "benchmarking" includes which of the following


actions? :

A. Responding only as problems arise


B. Changing based on the values of service providers
C. Doing things the same way for a long time
D. Comparing products and processes to those of the best competitor

129-Which of the following is the BEST reason why the graphic


representation of data is usually superior to verbal or written
representations?

A. Faster interpretation
B. Easier to perform
C. Most managers can't understand numbers
D. Easier on the eyes

130-Which of the following is the BEST definition of process? :

A. The steps required to provide care


B. A series of steps that achieve a desired outcome
C. Patient care activities
D. Technical aspects of providing care

131-Organizational culture is best defined as:

A. assumptions about individuals and how work gets done


B. ethnic make-up of employees
C. provision of activities to employees such as National Nurses Week
D. professional development of employees

25 of 268
132-Responsibility of quality improvement within the organization is to:

A. Chief executive officer


B. Quality manger
C. Frontline staff
D. Everyone within the organization

133-The CEO has the following responsibility except:

A- Implementing the policies of the governing body


B- Monitoring the productivity of the organization
C- Monitoring the quality improvement function
D- Approval of the reappointment

Approval of reappointment of medical staff is the ultimate responsbility


of the Governing Board although the recommendations are made by the
CEO through chief of staff affairs.

134-Before the end of the meeting the most important to do is:

A. Putting the agenda for next meeting


B. Make summery for the meeting
C. Instruct the members to do their duties
D. Make summery to the leader

135-One of the team members that keep members on track&focus on the


process is:

A. Leader
B. Facilitator
C. Time keeper
D. Recorder

-Facilitator is not one of the team member.


- Time keeper is part of team members and the one who monitors the
time spent in meetings and keeps people in track ,so that times are well
spent and don’t exceed scheduled times.
- Recorder is the secretary and part of team members is responsible for
keeping documents and creating reports or documents as needed by
the team.
- Team leader is the member of the team and is the one who provide
direction.
-The answer is (A) the leader.

26 of 268
136-The following is important in development of practice guidelines
except:

A. Evidence based researches


B. Experience of peers
C. Patient expectation
D. Clinical knowledge of peer physician

137-In development of the practice guidelines, the following is involved


except:

A. Physician
B. Quality manger
C. Evidence based research
D. Nurses

138-Nurse to patient ratio is an example of what type of measures? :

A-Structure
B-Process
C-Outcome
D-Monitoring

139-On presentation of graphed data to governing body, which of the


following you need to add to help interpretation of data:

A. Methods of data collection


B. Compare with bench mark
C. Names of data collectors
D. Duration of data collection

140-When the team members start to interest in hearing each other,


being on focus on goals, this is the stage of

A. Performing
B. Storming
C. Norming
D. Forming

141-Validity of measures is defined as:

A-Repeated measuring leads to the same results


B-Low in cost
C-Well understood
D-Measure what's intended to measure

27 of 268
142-Avoiding waste, in particular waste of equipment, supplies, ideas,
and time actualize which quality dimension:

A-effectiveness
B-efficiency
C-appropriateness
D-efficacy
Efficiency is the maximum utilization of the available resources,
maximum unit production from unit of resources.
Efficacy: the capacity of the resources to deliver + results proved by
research
Effectiveness: the amount of the desired +ve results reached when we
actually utilize the resources
Appropriateness: is that the service we deliver is related and in
accordance to the real need of the patient

143-A team approach to problem solving is most useful when:


1. The organization goals are unclear
2. Diverse areas of expertise are required
3. Communication challenges exist
4. There are ample resources within the organization.

144-After completing a surgery the nurse responsible for the surgical


instruments didn’t find one surgical clamp, an ultrasound was done to
the patient and nothing was found in her abdomen ,in your opinion this
case is:
1. Claim management
2. Res ipsa loquitor
3. Malpractice from the nurse.
4. Doesn’t matter since nothing happen to the patient
The answer is 3. malpractice from the nurse because it can’t be claim
management or res ipsa loquitor because there is no evidence(‫دليل‬
‫)ملموس‬about the negligence and if I didn’t do nothing then this is not
quality as of course something will happen in the future, so I will
consider malpractice from the nurse and I will investigate to find the
root causes for this accident in order to prevent it in the future (risk
prevention).

145-Who is responsible for developing vision for change:


1. CEO
2. Quality Council.
3. Quality Leader.
4. Quality Manager.
Those who set the vision have to be leaders and there is no single
person responsible for setting the vision alone so the perfect answer
here would be QUALITY COUNCIL as it is formed of group of quality
leaders.

146-To successfully achieve Quality Improvement, Leaders should:


1. power over others.
2. power through others.
28 of 268
3. power with others.
4. delegate power to others.

147-Which of the following statements and documents are most likely to


reveal the organization's underlying or true value system? :

a. Mission, ethics policy, strategic initiatives


b. Vision, ethics policy, corporate bylaws.
c. Values, QM/QI plan, utilization management plan.
d. Mission, Vision, Values.

The key words of this question are:


UNDERLYING or TRUE VALUE SYSTEM, this means that there are 2
types of value systems ,the one which is documented and planned for
before the implementation and the one which really represents the
value system in our organizations during the implementation which is
not necessarily the same as the one documented. so we have to seek in
the answers for the answer which represent the value system which is
documented and the one really implemented ( how the work really gets
done).
Answers, B C and D, all represents written documents, the right answer
is A and the key here is SRATEGIC INTIATIVES which is a key word for
the implementation of all those written documents.

148-There’s multiple payment systems for healthcare systems, the


following system foster quality improvement by the healthcare
organizations:
A-pay for service
B-prospective payment system
C-payment responsibility is held by the patient
D-retrospective payment system

149-the quality professional should master all of the following skills but
the most effective one is:
1. data analysis
2. financial management.
3. system’s design.
4. written and verbal communication

first of all u have to know how to answer theses kind of questions, the
key word here is the most effective one: this refers to that all the
answers are skills that the quality professional must own and master in
order to perform properly, he has to know how to analyze data and use
these data for decision making and prioterization, he has to to know
how to design or share in the designing and planning of different
systems, he has to be aware about different aspects of financial
management in order to control and evaluate the cost in order to add
29 of 268
value to the process of care.
but imagine with me that this quality specialist is a bad communicator
how could all his tasks be effective?????? how could he communicate
the results of the analysis of data? how could he share in designing
systems with people he cant communicate with ?
and the most important how could he lead?
how could he educate:
how could he be a marketer for quality?
how could he lead by example?
how could he reach the buy in of the leaders and the detailed
management?
how could he reach the leadership and the organizationwide
commitment?
how could create a culture of quality?
communication communication communication
u have to own this skill to make dreams come true.

150-Leader that transfer values of the organization to the employee to


make

A. face to face communication

B.open, timely and 2-way communication

C. top-down instruction

D. paper-mediated, closed door communication

151-In behavioral health care setting CQI team working for 1 year to
decrease the chemical& physical restraint. After application of the
program, the falls with subsequent injuries increase for 1 standard
deviation, the following action is-2

Return to the physical restraint & increase monitoring-A

Continue the program & seek opportunities to decrease falls-B

Stop the program & reengineer the process-C

Use more chemical restraint & educate the physicians-D

152-On presentation of the annual review to the governing body, the


following is important to include the presentation

Graphs & tables -A

Minutes -B
30 of 268
Team achievement-C

Complaints -D

153-In the first meeting of quality improvement team the following is


done except

Introduce the members-A

Studying the process -B

Agreeing on ground rules-C

Organize the dates of the meetings –D

154-The drawback of raw data for interpretation is

Can't be graphed-A

Can be measured for adherence to standard only?-B

No reference points for interpretation?-C

Can't be averaged-D

155-On presentation of graphed data to governing body, which of the


following you need to add to help interpretation of data

Methods of data collection-A

Compare with bench mark-B

Names of data collectors-C

Duration of data collection-D

156-The upper & lower limits of control charts are measured from

The actual process measurement-A

Benchmark-B

Community advised measures-C

Recommendation of accrediting body-D

157-For variance interpretation, the most suitable statistical tool is

Flowchart-A

Control chart-B
31 of 268
Bar graph-C

Pie charts-D

158-After patient death due to medication interaction, the following


action is

Couse & effect diagram-A

Root cause analysis-B

Failure mood & effect analysis-D

159-A hospital found a high rate of vaginal delivery after primary


caesarian section, the appropriate action is

Recommended to increase rate of caesarian-A

Review competence of the obstetricians-B

Review the diagnostic criteria leads to caesarian-C

Try to maintain this strategy-D

160-The paradigm shift is

Change the frame of thinking-A

Improve the monitoring measures-B

Increase the standard-c

Use the recent in medicine and technology-D

161-The facilitator has knowledge of performance improvement& can


use tools properly & must not

Be a member of the team& invest interested in the process-A

Quality professional-B

Skillful in team management-C

Be working in the organization-D

162-When the team members start to interest in hearing each other's &
being on focus on goals & to respect each other's, this is the stage of
32 of 268
Performing-A

Storming-B

Norming-C

Forming-D

163-In a culture of patient safety, the most appropriate surveillance to


assess the infection rate is

Total house surveillance?-A

Targeted,perioterized surveillance?-B

Community surveillance-C

None of the above-D

164-An organization hires a quality professional to pass quality


improvement concepts to the staff. The first thing the quality
professional should do

Deliver lectures to the staff-A

Assess the present knowledge of the staff-B

Review the previous performance of the staff-C

Make interview with the staff-D

165-The responsibility to pass quality improvement values to the


organization is of the

Leader-A

Quality manger-B

External consultant-C

Nurse manger-D

166-A physician complain a nurse to quality manger that the nurse


doesn’t do his orders ay time & ask him to tell the nurse manger what
should quality manger do first

Tell the nurse directly-A

Review the medical record-B

33 of 268
Talk to the nurse manger-C

Tell the CEO-D

-D

168-The responsibility of implementation the quality improvement


approach within the organization is to

Medical staff-A

Quality manger-B

Leader-C

Front line staff-D

169-Continuous quality improvement efforts find problems in h hospital


admission to provide breakthrough ideas in admission , the quality
improvement team seek ideas from

Other hospital-A

Previous lectures-B

Automobile industry-C

Hotel resort industry-D

170-All of the following is considered type of recognition to the team


except

Putting their names in the storyboard-A

Allowing them to make presentation to the management-B

Makes an articles in hospital newspaper-C

Give certificate signed by the leader-D

171-According to Deming one of his advises in quality improvement is

Stop annual inspection for employee-A

Make responsibility of quality for one person-B

Give more lectures to the staff-C


34 of 268
Increase the hierarchy of the staff-D

172-Surveillance from 2 teams reach to opposite out comes.as a quality


manger the following action is

Make anew surveillance-A

Check the method of data collection-B

Make decision depends on the outcome of one surveillance-C

Nothing-D

173-In the business cycle, the negative cash flow present in which of the
following stages:

Growth stage -A

Harvest stage -B

Maintenance stage -C

None of the above -D

174-Medication error rates are considered an indicator of which of the


following parameters:

Finance -A

Patients -B

Clinical process-C

Learning -D

175-Measurement of effectiveness of a seminar delivered to the staff on


new methods for training asthmatic patients to use metered dose inhaler
is best done by:

Satisfaction survey for the trainees -A

Tracking number of attendees -B

Incidence and survey for the patients -C

Satisfaction survey for the patient–D

35 of 268
176-To gain the leadership commitment to performance improvement
projects, the quality professional should capitalize on which of the
following:

A-Importance of the project on employee satisfaction and financial health of


the org

Importance of the project on the accreditation status and the clinical -B

Importance of the project on the financial health and accreditation status of


the org -C

Effect of the project on the community -D

177-The followings can enhance the spread of the change in the


organization except:

Inclusion of the leaders in the planning process -A

Seeking input from the staff -B

-C-Make punishments on errors related to the implementation of the


change

Adopt open door policy -D

178-Measuring morbidity and mortality is considered:

Administrative data -A

Support data -B

Managerial data-c

Clinical data-D

179-Information about customers can be obtained from all of the


followings except:

Complaint logs -A

Managerial observations -B

Satisfaction survey -C

Employee's opinions about customer's attitude-D

36 of 268
180-Data about the competitors may be obtained from all of the
followings sources except:

National standards -A

Individual customers -B

News media -C

Surveys performed by the local government -D

181-When there's uncertainty about the outcome of the process with


presence of guidelines and experienced staff, the process is considered
as:

A-Complicated

B-Complex

C-Simple

D-Flexible

182-Bias of a measure occurs when the measure:

A-Produce different results on repetitive measuring

B-Measure what's intended to measure

C-Produce data errors consistently and systemically

D-Exclude the negative cases consistently

183-The percentage of early diagnosed breast cancer after usage of new


imaging technique is considered:

A-Structure measure

B-Process measure

C-Outcome measure

D-Continuous measure

184-Make it safe to make mistakes, Will:

A-Increase the learning state within organization


37 of 268
B-Increase the errors

C-Decrease the loyalty of the customers

D-Decrease the self-esteem of the staff

185-All of the following are considered criteria for review of capital


budget except

A-Feasibility

B-project revenue

C-customer acceptance

D-actual costs

Cost benefit analysis is defined as-186

A-analysis of the capital expenditures for its effectiveness on the process

B-analysis of the capital expenditures for its viability and the broader
benefit

C-seeking for resources for the capital expenditures

D-a process done by the quality manger and approved by the governing body

Answer: B
A is the Cost effectiveness, the rest are not cost analysis means

187-All of the following may be realized as financial return on an


investment except

A-learning of the staff

B-reduction in losses

C-profit

D-avoided costs

188-To enhance coping of the desired behavior by the employee, you


should

A-punish the UN desired behavior maker

B-Make rewards on the desired behavior

C- make the desired behavior appear as normal requirement and needs no


recognition

D-blame and train the undesired behavior maker

38 of 268
189-Physician cooperation in quality improvement initiatives is
important and best gainedBy:

A-demonstrating the importance of the project on the competitive status of the


organization

B-clarifying the effect of the project on the effectiveness and efficiency


on the process

C-clarifying that TQM conflict with the way the physician thinks

D-demonstrating the importance of the project for the accreditation process

190-Training should be delivered to all levels of the organization,


training should be

A-company wide

B-just on time training

C-a and b

D- None of the above

191-After providing training, the specialist evaluate the increased skill


within the trainees, this is considered what level of evaluation

A-reaction level

B-result level

C-behavior level

D-learning level

192-The important things for a particular group are:

A-culture of the group

B-norms of behavior

C-value of the group

D-language communicated by the group

193-Avoiding waste in particular waste of equipment, supplies, ideas,


and energy is:

A-effectiveness

39 of 268
B-efficiency

C-appropriateness

D- patient-centeredness

194-In response to public concern the institute of medicine, published


the report "crossing the quality chasm" The following are domains for
health improvement identified in the report except

A-safety

B- patient-centeredness

C-equity

D-appropriateness

195- Avoiding injuries to patient from care that's intended to help them
is:

A-- patient-centeredness

B-equity

C-safety

D-timeliness

196-Health care organization is complex system. In complex system all


of the following are right except:

A-the interrelationships between agents are most important

B-The outcome is predictable

C-dealing with complex system require understanding the bid picture

D-here's a high chance for variation that may be identified as error or


innovation

197-System thinking is seen wholes, as regarding to system thinking all


of the following are right except:

A-facilitate the identifying the major component

B-foster integration

C-allow for narrow scope variation

D-provide basis for integration

40 of 268
198-In health care organization, the quality department developed an
indicator to measure the commitment of the staff to myocardial
infarction guidelines. This indicator measure:

A-process

B –structure

C-culture

D-outcome

199-All of the following are considered as rules that adopted health care
organization in the new century except:

A-decision making is evidence based

B-safety is system priority

C-the system react to needs

D-the patient is the source of control

because the system has to be proactive and not reactive...?

200-All of the following are goals for strategic management except:

A-Creating a fit between the organization and its external environment

B- Facilitate consistent decision making.

C-Foster anticipation, innovation and excellence

D-Make day to day work flow easy

201-Mission is defined as:

A-Statements of future goals of the organization

B-Organization's reason for existence

C-The plan developed by the organization in pursuit to its goal and objectives

D-The attitude and polices for employee

41 of 268
202-Goals and objectives are similar but could be differentiated by:

A-Goals are non-measurable while objectives are measurable

B-Goals are time-limited while objective are not

C-Goals are broad & general statement whole objectives are specific
statements

D-goals are non-observable while objectives are observable

203-When writing objectives, the following are guidelines except:

A-Keep the statements short and simple

B-State the desired outcome

C-Prioritize

D-Write the statement broadly

objectives must be written by following SMART criteria

204-The external environment affects the healthcare organizations, so


assessment of the environment is important. The followings are
examples for external forces except:

A-Economic forces

B-Societal values

C-Political issues

D-Other competitors

E-Equipment the organizations have

F- Research

205-Gap analysis may be used in:

A-Measuring the gap in performance between physicians

B-Measuring the economic gap between the individuals in the society

C-Measuring the gap in clinical status of patient

D-Evaluating the extent to which present strategy would have to be


changed in order to meet the goals and objectives

42 of 268
206-Hoshin planning allows integration of quality improvement with
strategic planning the primary reason to undertake hoshin planning is:

A-Allow long term planning

B-Align vision with day to day operations

C-Evaluate the extent to which strategy is accomplished

D-Demonstrate the gap between the organization and the best performer

207-Commitment of the governing body to quality improvement is


essential for the success of quality improvement activities. Quality
professional can enhance the board's commitment to quality by:

A-Assess knowledge and provide easily understood information

B-Ask them to make search on quality concepts

C-Provide them with materials to be studied on their own

D-Use of external educator

the quality specialist must be an excellent educator and communicator becoz he is


the one who convince others by his skills so he must help others to understand, feel
the importance of quality and commit to it using these skills , communication and
interaction is a must

208-The ultimate responsibility for setting policy for quality of care


provided by the organization is rested on

A-Staff

B-Quality manger

C-board of trustees

D-Department director

209-The American hospital association outlined 6 broad categories of


responsibilities for the board. One of the following is not included in
them

A-: Responsibility for ensuring access to education and research program

B-Responsibility for maintaining productive external relationship and for


influencing -public policies related to healthcare

C Responsibility for day-to-day activities

DResponsibility for protection or human resources

43 of 268
210-Health care quality professional can best communicate
organizational values & committed through

A-Leading by example

B-Disseminate monthly newsletter

C-Establishing a multidisciplinary task force

D-Creating mission statement

211-To allow changes to be maintained, you should ensure the change


in

A-The behavior of the staff

B-The hierarchy of the organization

C-The values within the organization

D-The reward system

212-All of the following leads to powerful culture for quality


improvement except

A-Consider sharing of the staff to quality activities at the time of


reappointment

B-Align reword to behavior support activities

C-Face the resistance to quality by strict action

D-Integrate quality improvement into strategic planning

213-Administration of penicillin to patient with history of allergy is

A-Potential drug event

B-Preventable adverse drug reaction

C-Non Preventable adverse drug reaction

D-Financial loss

214-All of the following are consider medical error except

A-Giving the patient drug of an in corrected type

B-Defibrillation by dead batteries

C-Misinterpretation of diagnostic test?

D-Rash caused by drug with negative history?

215-Which of the following is the primary goal of risk management?


44 of 268
A-Identify high risk areas of the organization

B-Maintain an effective reporting system

C-Perform failure mode & effect analysis

D-Reduce financial loss within the organization

216-The following are function of risk management except

A-Review for over usage of resources

B-Development risk management plan

C-Regulatory compliance

D-Claims management

217-The following are important aspect the risk manger should be


known except

A-Insurance industry

B-Aviation industry

C-Legislation and laws

D-Clinical area

218-Operative patient due to forgotten instrument inside the patient is


considered as

A-Common cause variation

B-Medication error

C-Sentinel event

D-Nurses fault

219-Root cause analysis is considered

A-Retrospective review

B-Concurrent review

C-Prospective review

D-Force field analysis

220-Which of the following is right order of steps in root cause analysis?

A-Finding, develop action plan, redesign the process, investigate event

45 of 268
B-Investigate event, develop action plan, report finding, and redesign the
process

C-Investigate event, develop action plan, redesign the process and report
finding

D-Investigate event report finding, develop action plan,, and redesign


the process

221-When an incident to patient is occurring, the appropriate action is

A-Distract the attention of the patient, discuss the incidence with the staff and
make action plan

B-Discuss the incidence with the patient; make punishment on responsible for
the incidence

C-Discuss the incidence with the patient and the staff and make an
action plan

D-Make no action

222-Which aspect of confidentiality of patient , the medical record may


be used in all of the flowing except

A-Education

B-Statistical evaluation

C-Research

D-Announcement to the organization

223-The departments that have more concern on discharge planning is

A-Quality management department

B-Utilization management department

C-Risk management department

D-Human resources department

224-All of the following are purpose of the utilization management


program except

A-Assure fair and consistent utilization management decision making

B-Ongoing monitoring to effective and efficient utilization of facilities

C-Education of staff on appropriate use of health care resources

46 of 268
D-Collaboration with the financial office staff regarding methods of risk
financing

225-Claim review is an example of

A-Prospective review

B-Retrospective review

C-Concurrent review

D-None of the above

226-The review of appropriateness of care provided by physician that's


done by another physician is called

A-Initial clinical review

B-Clinical peer review

C-Appeals consideration

D-Reappointment

227-Concurrent review that evaluate of each hospital case against


established criteria is

A-Preadmission test

B-Discharge planning

C-Review against criteria

D-Pattern review

228To change behavior within the organization the first step is

A-Create an urgency to change

B-Provide education bout the change

C-Anchor the improved behavior

D-Ask advice external consultant

229-Which of the following relate the cost of the process to the desired
outcome

A-Cost-effectiveness analysis?

B-Cost-benefit analysis?

C-Operation budget

D-Capital budget
47 of 268
230-In the first meeting of continuous quality improvement team the
following id done except

A-Introduce the members

B-Studying the process

C-Agreeing on ground rules

D-Organizing the dates meetings

231-Which from is the right order in development of teams

A-Forming, storming, performing, norming

B-Storming, forming, norming, performing

C-Forming, norming, storming, performing

D-Forming, storming, norming, performing

232-Conflict management is a function of leader to mange team in


certain stage of its development, this stage is

A-Storming

B-Norming

C-Adjourning

D-Forming

233-Evaluation of team is an issue to assure the effectiveness of the


performance improvementTeams, the following may be used in
evaluation except

A-Satisfaction of team members

B-Individual growth

C-Satisfaction of the board

D-Productivity

234-Provide training about quality improvement tools but may be


forgotten as it's not used after the training, the following may enhance
the use of the tools except

A-Giving example

B-Provide on time training

C-Make mock situation

48 of 268
D-Ask trainees to make on line search

235-Clinical decision support system can support medication safety by


altering prescribers to:
A. patient compliance and allergies
B. the need for dose adjustments and patient weight changes
C. drug interaction and patient weight changes
D. allergies and drug interactions

236-The surgery department monthly case review revealed 10 records


meeting criteria. six records did not meet the criteria in calculating the
incidence rate the dominator is
A. 4
B. 6
C. 10
D. 16

237-to be useful in preventing future error, a root cause analysis(RCA)


Should be performed
A. utilizing a multidisciplinary team
B. using practitioners who not involved in the event
C. starting no earlier than 45 days after the event
D. documenting opinion as well as facts

238-A quality council has charted a performance improvement team to


reduce medication errors, team has been meeting for several months
and progress has been very slow . Which of the following is the most
important factor for quality council to assess with team leader ?
A. composition of the team
B. number of medication errors since the team was charted
C. team member ability to interpret graphs
D. frequency of team meetings

239-A patient in acute psychiatric unit committed suicide by hanging


himself with his shoelaces to prevent this from occurring again the most
appropriate action is to institute
A. patient checks every 15 minutes
B. a policy allowing only non laced shoes
C. a 24 hour video monitoring system
D. a buddy system for the patient

Direct interference with the event by error proofing, prevent the cause

240-physician profiles are reviewed at time of reappointment to


A. ensure practitioner competency
B. compare practitioner to peers
C. review number of complaints
D. facilitate reappointment approval

49 of 268
241-which of the following is the best way to determine if quality
improvement initiatives is successful ?
A. Compare outcomes with pre established goals
B. Conduct a survey of employees
C. Present findings to the quality council
D. Survey patient and customers

244-management using quality improvement principles should


emphasize the importance of
A. Staff orientation
B. Customer expectations
C. Quarterly statistical reports
D. Team development

245-what sampling technique involves selecting the medical record of


every fifth patient undergoing cardiovascular bypass?
A. Convenience
B. Systematic
C. Stratified
D. Simple random

246-the most effective tool to improve communication between


caregivers is known as
A. SOAP
B. PDCA
C. PDSA
D. SBAR

247-quality improvement (team outcomes ) are best evaluated by which


of the following ?
A. Team leader
B. Senior leadership
C. PDCA process
D. Nominal group techniques

248-which of the following best describes an organizational vision


statement
A. It is used as a marketing strategy
B. It defines the structure of the institution
C. Describes the organization strategic plan
D. It reflects the organization inspiration

249-Two surveys were completed in a healthcare facility that showed


conflicting results concerning patient satisfaction with food
services ,the two surveys were independently designed and distributed
by different departments within the facility, the healthcare quality
professional should first :
A. Set up quality improvement team to improve food services
B. Distribute the surveys to obtain a larger sample size
C. Design,distribute,and analyze a new survey instruments
D. Meet with the departments to review the survey processes

50 of 268
250-Replacing retrospective review with concurrent review is an
example of
A. A paradigm shift
B. A process improvement
C. An empowerment process
D. Productivity enhancement

251-which of the following is essential component of performance


improvement report
A. Governing body approval
B. Data analysis and display
C. Individual performance review
D. team composition and attendance

253-A failure mode and effect analysis (FMEA) is performed


A. To immediately investigate an incident occurred
B. As a preventive measure before an incident occurs
C. If the severity of incident led to a patient death
D. When there is a chance of an incident reoccurring
254-A quality council has charted patient safety council the council is
concerned that staff may see this as another program that has been
added to their bust schedules that will eventually go away ,the best way
for the organization to establish patient safety as an ongoing part of the
organization's culture is to
A. Display the number of incident reports monthly with lessons
learned
B. Identify the patient safety goals and how they will be monitored
C. Make patient safety a part of the employee's job description
D. Include a presentation on patient safety in employee orientation

To increase learning curve and to promote culture of no blame

255-Standards of care based on the knowledge and research of


recognized experts are known as
A. Benchmarking data
B. Generic screens
C. Pre established criteria
D. Evidence based guidelines

256-An utilization management department of a hospital; has collected


data on length of stay (LOS ) and readmission rates . compared to
benchmarking the (LOS) rates are higher and readmission rates are
lower which of the following is the next step
A. Identify additional benchmarks to compare data
B. Conduct cost benefit analysis
C. Display readmission rates with run charts
D. Investigate the length of stay rates

51 of 268
257-In evaluating "long waiting times " a healthcare quality professional
best demonstrates components related to
staffing,methods,measure,materials and equipments utilizing
A. Run chart
B. Histogram
C. Pie chart
D. Ishikawa diagram

258- Hospital A has recently merged with hospital B after 6 months it is


noted that hospital A has successfully transmitted their staff to new
organizational values while hospital B still struggle .hospital A success
can best be attributed to
A. Required adoption of new values by all staff
B. Support of both hospital's mission statements
C. Acceptance of new mission and vision statements
D. Integrating technology and database
260- A healthcare quality professional is reviewing data with wide range
of values between highest &lowest points . the best way to rank order
using
A. Line graph
B. Simple frequency distribution
C. Ground frequency distribution
D. Bar graph

262-Analysis of post operative surgical infection rate over past year


demonstrates an increase in infections. Which of the following best
provides display of aggregated data compared to data from a local
facility?
A. Bar graph
B. Control chart
C. Pareto chart
D. Histogram

264-A healthcare quality professional hypothesize that a sample mean is


different from population mean . the standard deviation is not known .
the estimate must be calculated from a sample . Which of the following
statistical technique should be used :
A. Regulation analysis
B. Chi square analysis
C. T- test
D. analysis of variance

265-Which of the following are measures of central tendency?


A. Mean ,mode ,median
B. Standard deviation, variance, standard error
C. Grouped data, bell curve & distribution

52 of 268
D. Correlation, regression & T-test

267-Quality improvement teams are beneficial because they:


A. Maximize expertise & perspectives
B. Promote competition & pride among members
C. Improve management control
D. Authorize solutions to problems

270-A computer report is generated to assess the type of patient served


show 72% of visits were for obestric service which of the following
codes should be reviewed for verify the accuracy of percentage?
A. Procedural
B. Diagnostic
C. Medication
D. Attending physician

271-Data regarding the relationship between patient satisfaction & hours


per patient day on a medical unit were reported to be [r=0.69,p >0.05]
what's the correlation between these 2 values?
A. 0.05
B. 0.65
C. 0.69
D. 0.36

272-Surgery department's monthly case view revealed 10 records


meeting criteria,6 records didn't meet, when calculating the incidence
rate the denominator is
A. 4
B. 6
C. 10
D. 16

273-Quality improvement requires healthcare quality professional to


recognize that
A. Quality improvement generates its own change
B. The process is ongoing continuous & dynamic change
C. Process require radical change in short period of time
D. Quality improvement is managed by senior leaders

274- For which of the following the process capability best used
A. Identify if process is having intended effect
B. Focusing a team on the best thing to do
C. Narrowing down options through systematic approach of
comparison
D. Determining if a process meets established specifications
E. Fishbone diagram

53 of 268
275- When a case manager want to demonstrates length of stay data
that depicts both common cause & special cause variation which of the
following should be used :
A. Pareto chart
B. Scatter plot
C. Shewart chart
D. Frequency plot
276- The most effective way for healthcare quality professional to
communicate quality improvement activities to the medical staff is by
A. Developing professional relationships
B. Inviting medical staff to an in service on quality tools
C. Evaluating physician participation on quality teams
D. Providing outcomes data at medical staff meeting

277- which of the following does an outcome indicator measure?


A. What happens as result of a process
B. The steps leading to process
C. Individual performance of the process
D. Priority area to improve the process

278-physician profiling is reviewed at time of reappointment to:


A. Ensures practitioner competency
B. Compare practitioner to peers
C. Review number of complains
D. Delay reappointment approval

279- The use of clinical pathways & guidelines in hospital should:


A. Minimize variation in patient care
B. Reduce Length of stay
C. Improve patient satisfaction
D. Identify errors in patient care

280- Utilization management department of hospital has collected data


on length of stay & readmission errors compared to benchmarks. The
length of stay rates higher & readmission rates are lower .which of the
following is the next step?
A. Identify additional benchmarks to compare
B. Conduct cost benefit analysis
C. Analyze readmission rates with run chart
D. Continue to monitor length of stay rates

281-Which of the following best describe vision statement?


A. It is used as a marketing strategy
B. It is define the structure of the institution
C. It describe organization strategic plans
D. It reflect the organization culture

282-A facility is providing a new service for patients with chronic


pain .which of the following is the primary role of the healthcare quality
professional on evaluating this new service :
A. Comparing outcome to benchmark data
B. Evaluating cost benefit ratio
54 of 268
C. Assuring the staff is adequately trained
D. Developing performance monitoring criteria

283- Surveys were completed in healthcare quality facility that showed


conflicting results concerning patient satisfaction with food service. The
2 surveys were independently designed & distributed by different
departments within the facility. The healthcare quality professional
should:
A. Setup a quality improvement team to improve food services
B. Distribute the surveys to obtain a larger sample size
C. Design, distribute, analyze a new survey instrument
D. Meet with the department to review the survey process

285- Which of the following is the first step in strategic planning


process?
A. Setting goals and objectives
B. Define organizational structure
C. Determine productivity indicators
D. Establish & controlling of budget

286-A health care organization was recently reviewed by an outside


agency the most effective way to communicate the findings to senior
leadership is
A. Detailed report of finding
B. An explanation of findings that include data tables
C. An analysis of findings with graphs
D. A memo highlighting the main findings

287- A patient is being taught to self administer insulin which of the


following is the best method to assess the patient understanding of the
teaching
A. Return demonstration
B. Patient satisfaction survey
C. Family ability to verbalize instructions
D. Written pre& post test

288- A reengineering effort occurred at a facility. Activities particularly


those regarding staff layoffs were carefully planned, communicated &
implemented according to the plan ,one year later the business is stable
but staff morale is very low a healthcare quality professional has been
asked to consult in determining where the effort went wrong. Based on
the concepts of change theory the cause is most likely:
A. That reengineering decision was mistake
B. A failure to address the needs of staff that were retained
C. That leadership wasn't properly trained in the change
management
D. That a few disgruntled staff are instigating dissension in the ranks

55 of 268
289- A patient was admitted to a hospital with chest pain on Friday
evening, a myocardial infarction was ruled out & the patient discharged
on Sunday the utilization management coordinator reviewed the chart
on Monday to determine medical necessityfor admission. This type of
review
A. Avoidable
B. Prospective
C. Retrospectives
D. Concurrent
290- Which of the following tools should be used to collect patient and
practitioner special data
A. Flow chart
B. Graphs
C. Histogram
D. Spreadsheet

291-The main purpose of conducting focus group is to


A. Direct attention to identified problems
B. Determine customer needs
C. Track and trend occurrences
D. Obtain a clear picture of recurring problem

292- Which of the followings are primary reasons for developing drug
formulas?
A. Manage pharmacy cost & promote patient safety
B. Reduce medication errors& educate physicians
C. Encourage the appropriate use of medication& minimize inventory
D. Decrease food and drugs interactions and improve patient
satisfaction

293- An example of integrating the results of utilization management


assessment in the performance improvement process is
A. Educate case manager in discharge planning
B. Hiring more nurses
C. Reporting assessment result to executive staff
D. Assessing results of patient satisfaction survey

294-Priors to plotting a line graph, healthcare quality professional


should first
A. Calculate the mean
B. Define the axis measure
C. Identify the parameters
D. Develop a legend

295-Which of the following does quality leadership start, in contrast to


management by results
A. Profit & loss
B. Return on investments
C. Current procedures & services
D. Customer needs and expectations

56 of 268
296- Barriers in implementing the evidence based guidelines include all
of the following except
A. Lack of awareness that the guidelines are present
B. Lack of ability to implement
C. Lack of agreement by physicians
D. Lack of information of the patient acceptance

297-Comparing physicians performance will lead to all of the following


except
A. Increase the quality of care
B. Decrease costs
C. Increase competitions
D. Increase costs

298- Concepts of error reduction include all of the following except


A. Respect limits on attention
B. Encourage reporting of errors
C. Increase punishments on errors
D. Standardize

299- All of the followings related to patient safety except


A. Inpatient suicide
B. Wrong site injury
C. Patient discrimination
D. Infant abduction

300- To enhance patient safety communication has an important role the


type of communication should be
A. Closed doors, one way communication
B. Open door, two way communication
C. Paper mediated communication
D. Directive instructions

301-Sources of information that may enhance patient safety include all


of the following except
A. Internal risk screening as adverse events reports
B. Standards for the patient safety
C. Recommendation following analysis of sentinel events
D. Intuition of the leaders

302-The following are domains included in the IOM report "crossing the
quality chasm" except
A. Equity
B. Appropriateness
C. Safety
D. Effectiveness
303- All healthcare settings are required to adopt patient safety goals,
the following setting has the highest priority
A. Home care setting
B. Acute health care
C. Sub acute healthcare
D. Chronic setting
57 of 268
304-All of the following are methods to improve the accuracy of patient
identification at the blood transfusion except
A. Use 2 patient identifiers
B. Using bar codes
C. Using radio frequency identification tags
D. Asking the patient his name
305-Low medication error rating in a health care organization may be
due to one of the following except
A. Highly developed culture of safety
B. The systems has detect errors deficient
C. The staff is reluctant reports due to fear of reprisal
D. The organization adopt performance improvement approach lead to
error reduction

306- All of the following priorities safety issue to be improved except


A. Increase potential for harm
B. Increased frequency of occurrence
C. Decreased the likelihood that intervention could be implemented
D. Increased costs and claims from this issue in other healthcare
organization

307- To reach a culture of patient safety, the following order of steps is


A. Staff survey, plan & implement improvement, take in depth look,
documents the results and resurvey the staff
B. Staff survey, take in depth look, plan & implement improvement,
documents the results and resurvey the staff
C. Staff survey , plan & implement improvement, documents the results,
take in depth look and resurvey the staff
D. Staff survey. take in depth look, plan & implement improvement,
resurvey the staff and document the results

308- The approach of the six sigma


A. Find a problem, organize team. Clarify problem, understand variation
and specify solutions
B. Design, collect data, aggregate and analyze data, improve and
redesign
C. Determine, measure, analyze, improve and continue
D. Define , measure , analyze , improve & control

307- Analysis of data what phase of PDCA cycle


A. Plan
B. Do
C. Study
D. Act

308-The following criteria should be considered when selecting a


measurement except
A. Reliability & validity
B. Usability
C. Scientific acceptability
58 of 268
D. Approval by accreditation body

309-Risk adjustment is a method to


A. Collect data
B. Make the results comparable
C. Disseminate the results
D. Report the results to insurers
310-If the goal of data collection is to generate new knowledge point the
need of performance improvement, it's considered as
A. Recording
B. Sophisticated quality improvement
C. Benchmarks
D. Research

311- the incidence of catheter related nosocomial urinary tract infection


is
A. Structure measure
B. Outcome measure
C. Process measure
D. Sentinel events indicator

312- Patient specific indicators are not useful for


A. Reappointments of practitioner
B. Monitoring safety
C. Reporting to a senior management team
D. Comparing performance to external standards

313-Why the use of evidence based medicine likely to improve patient


safety?
A. It will help providers to avoid "cookbook "medicine practice
B. It will be prevent the inappropriate use of certain procedures
C. It will replace clinical decision making
D. Patient will receive consistent care based on sound scientific
research

314-Clinical outcome of medication include decreased


A. Hospital readmission, cost and medication errors
B. Hospital readmission, cost and discharge medication orders
C. Cost, confusion over medication and hospital staff satisfaction
D. Patient satisfaction, confusion over medication and costs

315- Determining what medication are to be taken at home is the


responsibility of the
A. Patient
B. Physician
C. Patient family
D. Beside nurse

316- Implementation of the medication reconciliation process require the


interdisciplinary effort of
A. Nurse, physicians, laboratory technicians and informatics
B. Nurse, physicians, pharmacists and informatics
59 of 268
C. Nurse, physicians, chaplains and informatics
D. Nurse, physicians, pharmacists and medical therapists

317-Medication reconciliation is-------????????


A. The reconciliation of duplicated dosage, frequency and discrepancies
B. The resolution of medication discrepancies in dose, frequency and
therapeutic duplication at time of discharge
C. The reconciliation of medication through the patient's hospital stay
D. The clarification of patient's medication at time of discharge
318- The most cost effective opportunity to enhance patient safety is to
A. Discover new therapies
B. Hire more competent staff
C. Discover how to deliver therapies that are known to be effective
D. Not accepting patients with known liability of complication

319- To insure that medication administration more safe, the steps of


administration should be
A. More complex
B. More branched
C. More simple
D. More dependent on experience of staff

320- What is not useful in making medication reconciliation successful?


A. Staff education
B. Collaboration between staff and patient
C. Making reconciliation errors part of staff reappointment process
D. Continues measurement by reliable measures

321-All of the following are considered as barriers to safety project


except
A. Poor coordination
B. High collaboration
C. Conflict of schedule
D. Resistance to change

322-Prevention quality indicators are useful in


A. Monitoring of mortality rate for medical and surgical patients
B. Identifying avoidable intervention
C. Identifying preventable complications and iatrogenic events for
pediatric patients
D. Identifying potentially avoidable complications

323-Indicators designed to identify potentially avoidable complications


A. Prevention quality indicators
B. Inpatient quality indicators
C. Patient safety indicators
D. Pediatric quality indicators

324- Patient safety incident is

60 of 268
A. Any unintentional event caused by healthcare that either did or
could have led to patient harm
B. Tool designed to elicit information from patient regarding certain
activities and behaviors that can influence health status
C. Making the member an active participant in choosing the course
of care
D. Examining claims for mistakes
325- Voluntary reporting system may under report incidents due to all of
the following except
A. Time constraints
B. Fear of shame
C. Developed safety culture
D. Blame litigation

326- The best strategy for preventing errors that cause harm is
A. Focus on who hold accountable to the harm
B. Provide a system of care in which patients are well protected
C. Refuse admission of patients prone to complications
D. Make the family responsible for safety of patient

327- As a quality professional to eliminate the misreading of


prescriptions or orders the appropriate action is
A. Make oral communications between the physicians and
pharmacists
B. Make prescriptions done by electronic means
C. Giving training to physicians to improve the writing behavior
D. Make penalties on both physicians and pharmacist when
prescription error occurs

328-When admission of patients prone to complications occurred the


appropriate action is
A. Decrease the nursing ratio
B. Increasing workers ratio
C. Increase physicians ratio
D. Increase the nursing ratio
329- The most common source of errors in diabetes care are
A. Physicians
B. Nurses
C. Pharmacists
D. Patients

Answer D; cause of lack of patient compliance on long term care plans

330- To be patient centered organization


A. Focus on cost containment
B. Focus on avoiding illogical waste
C. Focus on who is to blame
D. Offer more education allow the patients to demonstrate what they
know

61 of 268
331-All of the following should be incorporated in the solution for
medical except
A. Understanding the underlying cause of errors
B. Learn from reported errors
C. Eliminate conditions that contribute to preventable adverse events
D. Focus on staff producing errors
332- The least appropriate mean in training the staff to perform tasks in
a superior fashion with safer outcome is
A. Give team materials to be studied at home
B. Making workshops
C. Using mannequin and human simulator model
D. Web based education
333- Several types of information technology reduce the frequency of
medication errors which is not an example information technology
A. Computerized physician order entry
B. Robots
C. Computerized medication administration records
D. Registers
E. Automated pharmacy systems
F. Smart intravenous devices

334- The magnitude of benefit of information technology is greater in


pediatrics than in adult medicine due to
A. The pediatric are less competent
B. The variability in children is more than in adults
C. The liability for claims is more in pediatrics
D. Need for weight based dosing

It rule maximised in Weight base dose of medication to decrease errors

335-An order written for an albuterol sulphate inhaler without specifying


a frequency is
A. Adverse drug interaction
B. Medication error
C. Non preventable adverse drug event
D. Potential adverse drug event

Prescription must include 5 rights of medication;

right medication for the right patient by the right dose In the right time by the
right route; otherwise it's a medication error in prescribing

336- Development of cefazoline sodium rash in patients with known


allergy to cefazoline
A. Potential adverse drug event
B. Preventable adverse drug event
C. Nonpreventable adverse drug event
D. Health risk appraisal

62 of 268
337- System improvement by introducing information technology
applications are considered
A. Organizational change
B. Process change
C. Paradigm shift
D. Outcome change

338-What is of the following enhance patient safety culture


A. Openness about failure
B. An environment of team work and information sharing
C. An emphasis on learning
D. Accountability
E. All of the above

339 -Hand washing is an important aspect in infection and improvement


of patient safety the following may reduce infection
A. Use of alcohol based hand rubs
B. Surgical hand antiseptics
C. Elimination of the use of artificial nails
D. Natural nail tips limited to 1/4 inch
E. All of the above

340-An outpatient clinic assess medication side effect for children with
the following ages (1111122345)
The modal pattern of the children ages is
A- 1 B- 2 C- 3 D- 4
The median age is
A- 1 B- 1.5C- 3 D- 3

341- A healthcare organization is in a region with population with high


affection with G6PD deficiency in their children. As screening is
expensive, the organization screen samples only but the organization
found that treatment of undetected cases is more costly than screening
of whole population ,the organization should
A. Continue to screen samples
B. Screen all the population
C. Stop screening
D. Increase the size of sample

342-The sample include all available patients in the area is


A. Quota
B. Convenience
C. Stratified random
D. Purposive

343- The stratified random sample is


A. Random sampling after dividing the population into groups
B. Portions of the population
C. Choosing subjects fulfill the criteria
63 of 268
D. Sampling randomly

344- The following are steps in the data collection except


A. Interviewing
B. Monitoring
C. Correlations
D. Records reviewing

345-To identify the medication administration process which tool is


used
A. Control chart
B. Cause and effect analysis
C. Flow chart
D. Bar graph

346- After education of continuous quality improvement program to


evaluate effectiveness of the program
A. Do pre & post education exam
B. Evidence that the staff begin continuous quality improvement activities
C. Monitoring the previous performance of the staff
D. Review the attendance rate of the staff

347 – The primary purpose of the survey is to measure


A. Patient expectations
B. Capacity of the process
C. Competence of the staff
D. Utilization appropriateness

348- The process of monitoring newly privilege physician is


A. Credentialing
B. Proctoring
C. Appointment
D. Reappointment

349 –An organization ask a quality professional to help in preparedness


to survey of accrediting body the quality manager
A. Assign a team that makes presentation to the surveyors
B. Make mock surveys
C. Educate the staff types of questions that ay be asked
D. Review adherence of the organization to quality standards

350-The standard deviation can help in


A. Assess the variance of various points
B. Compare the difference between points
C. Correlate the points of each others
D. Identifying out of control

351-- As a quality manager to evaluate the effectiveness of dietary


department you review
A. The timeliness of diets delivered after physician orders
64 of 268
B. The appropriateness of the nurses' request to diet
C. The delivery of special diets ordered by physicians
D. Complication of the whole parental diet

352- A healthcare organization negotiate a contracts with insurance


company that required data about readmission of patients, as a quality
professional the first thing to take concern about is
A. Confidentiality of patients
B. Methods of data collection
C. Identify the exact data required by the company
D. Data collectors

353-Epowerment mix the employee


A. Makes money
B. Solve problems
C. Gain respect from others
D. Upward mobility

354- On discharge patient refuse billing because 2 out of 3 days of his


stay in the hospital is due to medication anaphylaxis this occurrence is
A. Billing error
B. Potentially compensable event
C. Nurse incompetence
D. Admission error

355- A nurse receives a verbal order for medication from physician, the
nurse should
A. Ask the medication from pharmacists
B. Neglect the order
C. Read the order back
D. Write and tell the order

356- Practice guidelines can be useful in all of the following except


A. Identify best practice
B. Cost saving
C. Patient expectation
D. Identify the process of care

357- The facilitator in patient focus group should do first


A. Choose homogenous group
B. Make ground rules
C. Make rapport with group
D. Instruct orders

358- Nurse in the post operative found missed clamp, X-ray has done to
the patient was negative & the patient has no symptoms this occurrence
is type of
A. Claim management
B. Potentially compensable event
C. Error rating
D. Incompetent surgeon

65 of 268
359-generic screening is an example of risk:

a. reduction.

b. identification.

c. prevention.

d. handling.

360-consensus means:

a. all members support decision.

b. unanimous agreement.

c. all the members are satisfied.

361- the most common cause of medication error is:

a. communication breakdown.

b. computer system error.

c. incompetent nurse.

362-Continuous quality improvement efforts find problems in hospital


admissions to provide breakthrough ideas in admission, the quality
improvement team seek ideas from:

a-other hospital

B-previous lectures

C-automobile industry

C-hotel and resort industry

363- On discharge woman refuse billing because 2 out of 3 days of his


stay in the hospital is due to medication anaphylaxis ,pt complain from
E. Billing error
F. Medication error
G. Admission error

364- Team cohesion is found in any stage of team development:

a- storming

b- performing

c- norming

d- forming
66 of 268
365- Root Cause Analysis conducted e' analysis of aggregated data , is
considered:

a- prospective

b- concurrent

c- retrospective

366- To display the stability of nosocomial infection rate overtime ,use which
of the following tools:

a- pareto chart

b- control chart

c-flow chart

d- bar chart

367- To prioritize

a- pareto chart

b- flow chart

c- run chart

368- The ultimate responsibility of setting policy for quality of care provided
by organization is rested on:

a- quality manager

b- staff

c- governing body

369-There is increase in rate of medication errors in a unit , what should


Quality Professional do?

a- review technology medication

b- ask advice from other successful units

c- review the delivery process of medications in the unit

67 of 268
370- Journal publish new article include 3 new patient safety intiatives. as
QP,what should you do first?

a- enter the intiatives in policy of patient safty

b- review the organizition for applicability of intiatives

371- The least prefered outcome of QI:

a- empowerment

b- increased statistical data

c- increased communication bet. staff

d- customer satisfaction

372- Staff education in organizational change include all of the following


except:

a- negotiating process

b- project & time mangement

c- conflict resolution

d- budjeting techniques

373- The best tool to display relation bet. Reimbursement & cost is:

a- scatter diagram

b- pareto diagram

c- flow chart

374- Benchmark is best describes as:

a- progressive attainment

b- compare e' measures

375- Valid data collection tool incorporates:

a- definition of data elements

b- reliable graphic presentation

68 of 268
376- CQI to be successful ,who must be included in the team?

a- Adminstrator

b- Quality council

c- persons performing process

377- The 1st crucial task of patient safety council is:

a- determine how patient safety goals will be monitored

b- make patient safety deparetment

c- make rapid response team

378- Integrating stratigic planning e' patient safety is considered:

a- cost benefit of patient safety program

b- organizitional performance culture

379- What is important for technology to reduce harm?

a- feasible & usable

b- used e' clinical work-flow

380- The indicator is driven from:

a- standards of accrediting body

b- community standards

c- important aspects of care & service

381- Best way to pass vision to all members of organization :

a- put in employee handbook

b- discuss vision at regular schedules

c- put in organization intranet

69 of 268
382- Which of the following is essential to an effective CQI?

a- consultation of legal advisor

b- support of leadership

c- direction from organization's quality department

383- Important reason for monitoring nearmiss record is to:

a- identify incompetent staff

b- provide lesson to staff

c- prevent negative publicity

384- Who is responsible of providing theorganizational guidance for a


facility's continous quality improvement?

a- facilitator

b- quality council

c- leader

385- gross cost 750.000$

net benefit 250.000$

net cost 250.000$

gross benefit 500.000$

- for every dollar spent , there is revenue:

a- 0.3 $

b- 1 $

c- 1.5 $

d- 3 $

386- when you see physician profile , what is your conclusion?

a- arthroscopy policy must be reviewed

b- number of cases seen by physician must be reviewed

70 of 268
c- clinical competency of physician is answerable

378- what should be added in physician profile to give complete idea


about him?

a- number of surgeries done by him

b- number of cases seen by physician per day

c- number of deaths related to physician

379- all of the following are considered type of reward & recognition of
team members except:

a- put their names in storyboard

b- makes article in hospital newspaper

c- acknowledging certificate during annual employee meeting

d- make team member present results to board of directors

380- which of following tools keeping team on the track to complete all
tasks?

a- flow chart

b- gantt chart

c- pareto chart

381-The facilitator's main role is concerned with the team

A keep the team focused

Quality professional

Help leader in assignment

382-If the gross cost of a quality improvement project is 750$, the net
cost is 250$, net profit is 250$ and the gross profit is 500$. Then the
return on investment (ROI) from such a project is:

a. 1

b. 0.33

c. 2

71 of 268
383-how to compare l.o.s of many patient by the physician

mean

correlation

s.d

range

384-indicators should come from

organization wide plan

process of care

standard indicators

accreditation indicator

385-QI activities should be considered for all except

a- comptability e~ facilities mission

b-ease of development of data collection tool

c-process that are high volume for facility

d-finding from pt satisfaction survey

386-software for physician what is role of cphq?

a-assess use of technology

b-cost-benefit analysis

c-focus group with end users

387-Risk control imposes providing mechanisms for :

a. Elimination of hazards that lead to risk occurrence.

b. Prevention of recurrence of risk occurrences.

c. Minimizing the loss after risk occurrence.

d. Dealing with the legal aspect of risk liabilities

388-Tracking positive variance

A. tends to show the costlier procedures

B. tends to show clues to cost-effective care

C. shows best what doesn't work

72 of 268
D. helps to track standard practice patterns

389-The role of a successful leader in problem solving is all of the


following,

but the most effecting is:


a. Identify the cause of the problem.
b. Guide others to solve the problem.
c. Bring order out of chaos caused by the problem.
d. Reward supporting behaviors during the problem.

390-All of the following are important roles of HQP in QM, but


his main duty is to:
a. Communicate Q. defects.
b. Report Q. defects.
c. Present possible solutions for Q. defects.
d. Document Q. defects.

391-To ensure implementation of the required improvements,


whose commitment must be sought :
a. Top management.
b. Leaders
c. Organization wide
d. Detailed management

392- In planning for a crises, a leader should:

a. Provide a sound infrastructure.

b. Develop a plan to deal with the situation during the crises.

c. Develop a plan to deal with the situation after the crises.

d. Provide no preliminary plans and face the situation as applicable.

393-Linking physicians incentives with quality performance runs


the risk of:
a. Over-utilization of care.
b. Under-utilization of care.
c. Misutilization of care.
d. Negligence of care

73 of 268
394-healthcare q. professional has been asked to present information to
senior leadership about hospital publicity, the report should include:
a-clinical expert
b-computer training
c-comparison of data with benchmark
d-customer satisfaction

395-the chief q. officer has the responsibility of education &


implementation of CQI process, to affect cultural change administration
must:
a-believe the costs are justified by the benefits
b-be assigned as member of the team
c-receive quarterly reports

396-for continuous q. improvement team to be successful who must be


included in the team:
a-administrator
b-department supervisor
c-staff
d-facilitator

397-the key for creating sustained value in the org. is to:


a-delegate policy-making &oversight to q.council
b-develop a strategy that derives from the vision,strategic goals &cost benefit
analysis
c-adopt an organizational ethics policy &code linked to mission,vision &values
d-act on predictive performance measures aligned to strategic goals &
department objectives

398-in evaluating the current improvement program for strengths


74 of 268
&weaknesses, it's not necessary to assess:
a-strategic initiatives
b-managed care contracts
c-team minutes
d-alternative q.management software productsproject

399-Which of the following is the best approach when implementing a


National Patient Safety Goal related to identifying potential errors in a
patient's care, treatment, and services?
A. providing the patient and family an opportunity to ask questions
B. having the patient provide return demonstration of the knowledge provided
C. showing a video to a patient and their family
D. giving both written and verbal instructions to a patient and family

400- Appropriateness" of care refers to:

a. The degree to which the care is accessible and obtainable.

b. The degree to which needed care is provided to the patient at the most
beneficial time.

c. The degree to which care provided is relevant to the patient's clinical


needs.

d. The coordination of needed healthcare services for a patient among all


practitioner and across various settings.

401-When a newly introduced drug is told to be able of producing


positive results for certain clinical conditions, then this drug should be
considered:

a. Effective.

b. Efficient.

c. Efficacious.

d. Appropriate.

75 of 268
402-When an employer contracts with a health plan or directly with a
provider, this employer should be concerned about which of the
following perspectives:

a. The cost of the care provided.

b. The quality of the care provided.

c. The outcomes of the care provided.

d.All of the above.

403-Which of the following healthcare reformers developed the


structure, process, and outcomes" model?

a. Ernest Codman.

b. Florence Nightingale.

c. Avedis Donabedian.

d. Donald Berwick.

404-Which of the following tools can be used to identify (Customer


Needs)?

a) Focus groups.

b) Brainstorming.

c) Surveys and interviews.

d) a and c only.

e) a, b and c.

405-When the health care delivered should not vary in Quality because
of patient's personal characteristics such as gender, ethnicity,
geographic location, and socioeconomic status; then this health care is

a) Safe.

b) Efficient.

c) Patient centered.

e) Equitable..

406-The healthcare quality professional should

76 of 268
a) Practice the profession with honesty and integrity.

b) Have enough knowledge about relevant laws and legislations.

c) Promote the right of privacy for all patients.

d) all of the above.

407-Which of the following situations best describes the term


“Misuse” of Resources at healthcare facilities?

a) Patients receive appropriate medical services that are provided


poorly, exposing them to added risks of preventable complications.

b) Patients undergo treatment or procedures from which they do not benefit.

c) Patients do not receive beneficial health services.

d) None of the above.

408-Which of the fol1owing items is not considered as an


"outcome"?
a) Mortality. .
b) Management of Complications
c) Patient/family satisfaction.
d) " Activities of daily living" status.

409-All of the following actions would aid in adopting a "Quality culture"


inside any healthcare organization .EXCEPT
a) Delegation and empowerment of staff.
b) The (Quality professional) is assigned to lead such cultural
transformation.
c) Increased communication.
d) Top management commitment and involvement

410-All of the following are ways through which any organization


leadership can enhance the spread of "quality culture" within the
organization EXCEPT

a) Develop mission and vision statements.

b) Develop quality initiatives.

c) Adopt flexible management styles.

77 of 268
d) Assign quality professionals to lead the process of cultural
transformation.

411-What is the right sequence for the elements of Donabedian's model?

a) Process - Structure - Outcome

b) Outcome - Process - Structure

c) Structure - Process - Outcome

d) Process - Outcome - Structure

412-All of the following are key components that guide the (Strategic
Organizational Direction), except:
a) Mission.
b) Vision.
c) Procedures.
d) Values.

412-Which of the following is not correct about (Strategic planning)?


a) May need annual development of “operational plans”.
b) Should be based on objective environmental assessment.
c) Neither long-term nor short-term objectives should be developed until the
organization completes the strategic planning process.
d) Can be prepared for a single department within the organization.
413-Which of the following statements is not correct concerning (Goals
and Objectives)?
a) Goals are more specific than Objectives.
b) A single Goal may have many Objectives.
c) Objective should be challenging yet achievable.
d) A Goal statement should mention the excepted time for achieving the goal.

414-All of the following criteria are considered by the hospital


leaders when selecting a (Strategic Quality Initiative), except:

78 of 268
a) Has organization-wide impact
b) Is linked to one or more strategic goals.
c) Should addresses clinical issues only.
d) Should focus on the improvement of systems and processes.

415-The CEO of hospital (X) sake an area for improvement, he asked


each member of the senior management group to propose him a list of
potential improvement opportunities in the next meeting; what is the
most reliable source from which the managers can get their ideas?
a} Assessment of internal and external customer needs.
b) Assessment of needs of external customers only.
c) Comparative information pertinent to competitors.
d) The work experience each manager has.

416- All of the following about the using (Metrics) or (Measurement


methods) in an organization is correct, except:
a) They have no role in linking the organizational processes to the
achievement of the corporate plan.
b) They are the diagnostics which show progress in meeting corporate goals
and objectives. ·
c) They may take various forms according to the process to be measured and
the type of data to be collected.
d) Clear understanding of the organizational mission and vision, would
ultimately facilitate the development of the appropriate metrics.

417-All of the following statements concerning the "Goals" and the


"Objectives" are correct, except:
a) Objectives are more specific than goals.
b) A single goal may have many objectives.
c) Objectives can be measured with qualitative and quantitative criteria
e) Objectives are developed before goals.

418-Risk management in an organization is most effective when it is:

79 of 268
a) Responsible for sentinel event root cause analysis
b) Incorporated into safety management
c) Integrated with organizationalwide performance Improvement
d) The responsibility of the clinical performance improvement teams.

419-One fundamental difference between monitoring product quality and


service quality is based upon the fact that

a. a service is easier to measure and verify in advance


b. a service is not perishable
c. a service is more heterogeneous than an object
d. there are more service delays than product delays

420-The perception of quality by a patient receiving care in an


ambulatory healthcare center is influenced most by

a. the physical environment.


b. caring staff and physician.
c. new technology.
d. the physician's technical competence.

421-What is the most important relationship between structure, process,


and outcome as types of indicators of quality?

a. Interdependent: Structure directly affects both process and outcome.


b. Causal: Structure leads to process and process leads to outcome.
c. Relational: Useful for comparisons, but not causal
d. There is no relationship; they are categories used to group indicators

421-Which of the following best describes the successful outcome of


the quality improvement process?

a. Customer satisfaction
b. Enhanced communication
c. Employee empowerment
d. Improved statistical data

422-Monitoring phlebitis associated with IV insertions by nurses in the


Surgical Intensive Care Unit addresses which focus?

a. Outcome of care
b. Process of care
c. Structure of care
d. Administrative procedure

423-Organizational "culture" most often refers to

80 of 268
a. the ethnicity of the organization's employees and licensed
independent practitioners
b. assumptions about people and how work gets done
c. the efforts to reach out to the diverse groups in the community.
d. the scheduled social and cultural events within the organization.

424-The leadership style that is said to motivate employees, and that


optimizes the introduction of change, is

a. autocratic.
b. consultative.
c. participatory.
d. democratic

425-In participative management the manager

a. relinquishes decision-making responsibility to the staff.


b. retains the final decision-making responsibility.
c. presents a final decision to the staff
d. permits staff participation only with noncritical issues

426-Which of the following is most important to the successful


implementation of quality improvement activities?

a. Financial commitment and written quality management plan


b. Leadership commitment and organizationwide collaboration
c. Leadership commitment and financial commitment
d. Information management system and department collaboration

427-The best way to facilitate leadership education about the role of


ethics in the organization is to understand that

a. each leader's personal value system drives decision making.


b. the organization's written Code of Ethics drives decision making.
c. the organization can have both good and bad ethics.
d. accountability for organizational ethics is primarily internal, not
public.

428-In any quality management approach, how can you best evaluate
the effectiveness of action taken?

a. Use the same performance measures to remonitor the process.


b. Formulate a new special study to monitor the action.
c. Interview the staff involved in implementing the action plan.
d. Do nothing. Effectiveness is expected with well-planned action

429-According to QI process theory and quality/performance


81 of 268
improvement standards, it is best to select a quality improvement
project that

a. is the chief executive officer's ongoing quality or cost concern.


b. is limited in scope and time to provide quick feedback.
c. has the greatest potential to improve patient outcome.
d. has the greatest potential to save the organization money

430-All quality improvement approaches or models include the following


mechanisms except

a. developing strategic goals.


b. prioritizing problems/projects.
c. collecting and analyzing data
d. taking action to improve

431-The main goal of measurement in performance improvement is to

a. provide specifications for processes needing redesign.


b. keep track of process and practitioner variances.
c. collect accurate data reflecting actual performance
d. establish benchmarks for the improvement process

432-The integrated delivery system is undergoing a major reengineering


effort, with corporate goals to complete projects timely and within
budget. Of the following, the most appropriate approach or model is

a. Failure Mode and Effects Analysis (FM EA).


b. rapid cycle.
c. FOCUS-PDCA.
d. Balanced Scorecard

433-The primary goal of quality/performance improvement is to improve

a. patient care processes.


b. patient safety.
c. patient outcomes.
d. patient satisfaction

434-Failure mode and effects analysis (FM EA) is what type of review or
improvement tool?

a. Concurrent
b. Focused
c. Prospective
d. Retrospective
82 of 268
435-The basic philosophy of benchmarking is

a. eliminating the competition


b. finding best practice and incorporating it.
c. getting all processes under statistical control
d. eliminating process deficiencies.
.

436-in developing a performance improvement plan you will implement the


following steps in which order:
1. collect basaline data.
2.evaluate the effectivness of action taken.
3.make the commitment.
4.implement the plan.

a.1,2,3,4
b.1,3,4,2
c.3,1,4,2
d.3,2,1,4

437-in postoperative assessment the nurse discovered that the surgeon has
replaced the wrong hip for a patient.this is considered:
a.sentinel event.
b.malpractice.

438-insurance company would require which of the following data in renewing


a contract with a hospital:
a. Hospital revenue,LOS, mortality rate.
b. Complication rate,readmission rate.
c. Custmer satisfaction,LOS

LOS is very generic and cannot judge the real performance of Hospitals.
Revenue is confidential data
Complication and readmission both represent care provided and are the
most annoying part between Hospitals and insurance companies

438-FMEA is considered:

a.proactive.

83 of 268
b.retrospective.

c.concurrent.

439-which of the following should be presented to GB in the annual review of


performance improvement plan?

a.team minutes.

b.team achievments.

c.occurance and inciedent reports.

440-when using mortality data in benchmarking ,it should be :

a.DRG

b.severity adjusted.

c.LOS adjusted.

441-method of data collection allowing direct contact and immediate feed


back?

a.focus group.

b.quesstionare.

c.surveys.

442-the best word explaining empowerment:

a.delgate tasks.

b.mutal trust.

c. open discussion but the leader perform all important tasks.

443-the team go through all of the following except:

a.norming.

b.forming.

c.storming.

d.conforming.
84 of 268
444-the best evidence of pateint safety culutre:

a.audits standards & medical record review.

b.anynomous reporting &audits standerds.

445-to facilitate change in the organization:

a.publish newsletters.

b.involve people who are working in the process.

c- get approval of governing body

446-when hiring a specialist from consulting firm to evaulate performance


improvement program ,as a quality professional what u should do?

a.schedule the program for the cosultaing firm.

b.identify areas with problems to the consulting firm.

c- identify outcomes

447-concentrated electrolytes should be removed from pt care unit to :

a.prevent medication errors.

b.improve resource utilization.

c.check expiration date.

448-which of the following is example of outecome measure:

a.mortality rate.

b.average LOS.

c.medication dispensing rate.

d.lab speciemen.

 YES....actually we have 3 aspects of patient care that


outcome indicators should monitor:1- clinical outcome (mortality, morbidity,

85 of 268
readmission,adverse reactions....) 2- patient functionality (long term health,
ADL....) AND 3- Perceived outcome (pt satisfaction, peer acceptability....)So
the only outcome measure in this question is A; Clinical outcome.

449-type and volume of pateints admitted in the hospital:

a.daily census.

b.case mix .

c.DRG

d.acuity index.

450- all of the following is positive patient outecome except:

a.decreased complication.

b.improved clinical &health status.

c.reduced infection rate.

d.decreased LOS

451-the ultimate authority and responsibilty for continous quality


improvement through out the organization are upon:

a.GB

b.cheif executive officer.

c.cheif financial officer.

d.quality professional.

452-the purpose for reappointment of a physician:

a.ensure clinical comptencey .

b.ensure that every physician has malpractice insurance coverage.

c.used in peer review activities.

453-to increase the effectivness of reenginering program ,the quality


professional should :

a.review polices and procedures.


86 of 268
b.plan carefully,communicate widely and lead effectively.

c.make a lecture for employees to define reenginering.

454-in root cause analysis the first thing to do:

a.flow chart.

b.fishbone diagram.

c. 5 Whys.

455-which of the following diseases is best for beginning of implementation of clinical pathway?

Volume Rate of complication


Physician LOS variation
champion
Heart failure 80 0.2 Yes Yes
Gastroenteritis 40 0.01 Yes No
Diabetes 50 0.05 No No
a.Diabetes.

b.heart failure.

c.gastroenteritis.

456-the main goal of performance improvement in patient safety culture is to:

a.reduce harm.

b.form a just culture.

c.conduct gap analysis.

457-in a culture of patient safety error is considered:

a.normal.

b.misconduct.

c.purposality.

error is not normal, it said than to err is human, only to confess that we may
err but it's not accepted to deal with it as a normal practice

Error is not a common cause to consider it normal

87 of 268
457-use the following data to answer the following 2 Q:

Number of discharges 142

Number of procedures 100

Orthroscopies 20

Hip replacement 40

Surgical wound infections 32

Incomplete medical records 40

457-A.the rate of overall surgical wound infections:

a.32%.

b.23%.

c.30%

d.40%.

457-B.the rate of overall delinquent medical record:

a.40%.

b.28%.

c.30%.

d.20%.

458-performance improvement focus on:

a.process and systems.

b. individuals.

459-as a result of custmer surveys, a new service is suggested to be


delivered. It should be written in :

a.financial plan.

b.strategic plan.

c.quality management plan.

88 of 268
460-sharing in developing SBAR with quality professional:

a.nurse.

b.medical record director.

c.financial officer.

461-multiple regression analysis means:

a.measure the relationship between 1 independent variable &multiple dependent


variables.

b.to exclude the main causes of the problem .

462- to show the relationship between 2 process characterstics:

a.pareto chart.

b.fishbone.

c.regression analysis.

463-to evaluate the effectiveness of the pharmacy unit, you should review:

a. timeliness of delivery of prescribed drugs.

b.length of Ab use.

c.the effectiveness of the drug prescribed.

464- when you compare between the performance of your organization


and the performance of industry leader,this is called:

a.best practice.

b.benchmarking.

c.setting objectives.

465-After making a brainstorm session for a, the next tool to be used is:

a. flow chart

89 of 268
b. multivoting

c. Delphi technique

d. affinity diagram

:A Performance measure/indicator is -467

.A statement of expectation to perform as expected .a

.A set of specifications of care .b

.Used to assess an outcome or a process of care .c

.Describes appropriate and expected courses of action .d

In six sigma model, the goal is-468

elimination of defects.a

Zero defects.b

defects per million 0.34.c

defects per million 3.4.d

:Six sigma strategy involves the following consecutive steps -469

.Define, analyze, aggregate, control , improve and replicate .a

.Define, measure, analyze , improve, control, and replicate .b

.Define, organize, measure, analyze , control, improve and replicate .c

.Define, measure, improve, analyze, control and replicate .d

:In the concept of performance improvement, its definition is -470

.Performance improvement is the degree of excellence .a

Performance improvement is simply the degree to which .b


.healthcare is efficacious

Performance improvement is what is done and how well it is .c


.done to provide healthcare

.Performance improvement is doing the right things right .d

Healthcare organizations need to consider three issues in building -471


:the framework for P.I

90 of 268
The state of the art professional knowledge, the integrated, coordinated .a
.efforts, and competent technical skills

Measuring, assessing and improving important functions and work .b


.processes

The leader’s ability to anticipate, understand, proactively and flexibly .c


respond to healthcare changes

The organization's relationships to the external environment, its .d


internal characteristic and functions, its methodology for systematically
.improving important functions

:The function of performance improvement includes -472

.Three processes linked to the quality management cycle .a

.Four processes linked to the quality management cycle .b

Design, monitor and analyze processes not linked to planning ,control .c


.and improvement cycle

.Planning, control and improvement cycle linked to PDCA cycle .d

The quantifiable process and outcome indicators used to monitor -473


:performance are

.Process indicators.a

.Performance measures.b

.Monitors.c

.Threshold measures.d

:Monitoring process includes-474

Ongoing prioritization of measurement efforts based on strategic goals, .a


.data collected and available resources

Validation that selected performance measures actually measures .b


.what it is intended to measure

Linkage to outcome objectives, reliability and validity checks, timely .c


.collection of data, and use of acceptable data bases

All of the above .d

:Those who determine and prioritize data collection are-475

91 of 268
.CEO.a

.Leaders.b

.Process owners.c

.Medical staff.d

Anytime the performance of an individual practitioner becomes the -476


:focus

The quality professional should assume responsibility for analysis and .a


.action

.The CEO should assume responsibility for the analysis and action .b

The leader of the organization should assume responsibility for the .c


.analysis and action

The appropriate peer review body must assume responsibility for .d


.the analysis and action

:Aggregation of data is-477

.The separation of substantial whole into its constituent parts .a

The translation of data collected during the monitoring process into .b


.information

Combining standardized data; gathering into a mass, sum, or .c


.whole

.None of the previous .d

:The analysis process includes-478

The separation of a substantial whole into its constituent parts for individual .a
.study

The comparison of aggregate level of actual performance for each indicator .b


.with the designated triggers

The translation of data collected during the monitoring process into .c


information

.All of the above.d

479-What is a process?

A. It assures continuous improvement and employee participation.

B. It converts inputs into outputs.


92 of 268
C. Anything that satisfies customers.

D. Anything that causes change.

a. A only.

b. B only.

c. C and D.

d. B and D.

480-How do you know if a process is operating normally (i.e., in


statistical control)?

a. Customers are satisfied.

b. Performance measures display variation consistent with a Normal curve.

c. Performance exceeds standards.

d. Performance measures are within the 3 sigma limits.

481-Characteristics of effective brainstorming include:

A. Generate lots of ideas quickly.

B. Understand the root cause of the problem.

C. People contribute without fear of ridicule.

D. Everyone agrees on the topic or issue.

a. A, B, and C.

b. A, B, and D

c. A, C, and D

d. B, C, and D

482-A Pareto Chart is used to:

a. Document the steps and material used in a process.

b. Identify the most important items.

c. Determine which alternative is best.

d. Track performance over time.

483-A Histogram is used to:


93 of 268
a. Provide a picture of process variation.

b. Track performance over time.

c. Identify the most important cause of a problem.

d. Segment customers and customer needs.

484-To facilitate the identification, exploration, and graphical display of


the POSSIBLE causes of an effect you would use a:

a. Force Field analysis.

b. Flowchart.

c. Corrective Action process.

d. Fishbone diagram.

485-A process can be documented by using:

A. Flowcharts and Standard Operating Procedures.

B. Pictures and video.

C. Histograms and Cause-Effect diagrams.

D. Capability studies.

a. A and B only.

b. A and D only.

c. B and C only.

d. C and D only.

486-A team has identified a process for improvement , selected


examples of best practice performers ,visited those sites ,gathered all
necessary data & compiled results. The most effective next step for the
team is to:

a)identify the next process for benchmark

b)implement change back at the team site

c)compare results to historical data

d)make the results public for others to use in benchmarking

94 of 268
:In special cause variation, the source of variation is -487

.Intermittent, unpredictable, chronic , extrinsic and assignable .a

.Intermittent, unpredictable, unstable, extrinsic and assignable .b

.Intermittent, inliers, unstable, extrinsic and assignable .c

.Intermittent, unpredictable, unstable, and intrinsic and assignable .d

: Intensive analysis is mainly performed-489

.At specified time intervals )a

.To identify opportunities to improve )b

.When significant undesirable variation in performance occurs )c

All of the above )d

When no problems or opportunities to improve care are found after -490


:sufficient time

.Do nothing ,this is the ideal situation .a

.Wait for a few weeks and reevaluate ..b

.Wait for a sufficient time(6 months-1 year),then celebrate .c

Wait for a sufficient time (6 months-1 year),then reevaluate the indicators, .d


.data collection methods and analysis processes

:A common cause variation is-491

.An intrinsic, inliers, unpredictable, chronic variation .a

.The responsibility of the process owners .b

.Correctable by top management and the team .c

.An intrinsic, outlier, unpredictable, acute variation .d

:Selecting improvement opportunities depend on -492

Deficient systems .a

Insufficient knowledge .b

.Deficient individual performance or behavior .c

All of the above .d


95 of 268
The most important value of pilot testing during the improvement -493
:process is to

Establish time frames ,sample size,and location most representative of the .A


.whole population

.Determine if the improvement is viable (will have the desired results) .B

.Test alternative improvement action if pilot test is unsuccessful .C

All of the above.D

:Pareto rule means that-494

.of the problems will have 20% of the impact 80% .a

.of the activities will bring 80% of the results 20% .b

.Focus on high-volume, high-risk, and problem prone issues .c

.Ignore problems with low potential for adverse impact .d

Based on most quality improvement standards, the quality council -495


:is all of the following except

.Delegated to plan strategically for all improvement plans .a

.Delegated by the governing body, administration , and medical staff .b


Comprised of representatives from administration ,governing body, medical .c
.staff and key departments of the organization

Delegated to prioritize, and coordinate all organization-wide quality PI .d


.activities

:The following are some of the responsibilities of the quality council -496

.Select, and charter teams for designated QI/PI projects .a

Develop, modify and approve the PI plan prior to approval by the .b


.governing body

.Approve strategic quality initiatives based on strategic plan goals .c

All of the above .d

96 of 268
:The following are among the responsibilities of quality council -497

.Reporting to the governing body and providing summary reports .a

.Establishing and overseeing a confidential peer review policy .b

.Meeting at least monthly or more on the call of the chair .c

All of the above .d

:All of the following is a true function of the quality council except -498

Determines and supports the education and training needs of the organization .a
.related to quality/PI

b. Replaces the peer review committees and handles practitioner-specific .b


.issues

.Review aggregated data feedback from customer satisfaction surveys .c

d. Determine the budget needs and implications of organization wide PI .d


.activities

:The only unique step in the 10-step benchmarking model is -499

.Identify what is to be benchmarked .a

.Identify comparative providers .b

.Communicate benchmark findings and gain acceptance .c

.Project future performance levels .d

In the Benchmarking process, future performance goals answer -500


:which question

?What is our current performance .a

?What was our previous performance .b

?Where do we want to be .c

?Where will our competitors be .d

:The sequence of events in the PDCA cycle is -501

.Plan change, gather data, implement on small scale then act .a


97 of 268
Plan, implement plan on small scale, educate, gather data, implement on full .b
.scale

Plan, implement change on small scale, then full scale,and gather data to to .c
.evaluate results of change

.Find process to improve, plan, implement on large scale, study,act .d

502-Patient satisfaction scores for a community hospital demonstrate


multiple areas for improvement including a need to improve
attractiveness of the facility responsiveness to patient needs &
physicians & nursing communication. Which of the following should the
healthcare quality professional also expect to find?

A) administration prioritization & leading units to achieve organizational goals

B) unit manager who openly discuss patient satisfaction scores

C)units operating independently with little communication between units

D)employee satisfaction scores in the 80th percentile compared to other peer


organizations

Outcomes as used as indicators of quality are defined as any of the -503


: following except

Changes in health states .a

.Changes in knowledge or behavior pertinent to future health states .b

.Provide report of what is the organization is doing now .c

.Satisfaction with healthcare .d

Outcomes measures enable us to measure and assess which of the -504


:following

What is the right thing to do(the quality of technology) .a

.Whether what is already known to be best practice is being implemented .b

.Whether what is being done is acceptable (the quality of performance) .c


98 of 268
All of the above.d

Outcomes measurement include the following -506

.Control of results of processes .a

.Patient perception of outcome .b

.Ability to function and impact on quality of life .c

All of the above .d

The following term is synonymous with clinical practice guidelines -507


:except

Standards of care .a

Standards of Practice .b

Clinical Algorithms .c

Practice parameters .d

:Clinical pathways are developed as a way to -508

.Track significant variations case by case .a

.Predict preadmission / preprocedure visits in managed care .b

.Establish a clear mechanism focused on the patient .c

.All of the above.d

509-An emergency room tracks wait times from patient arrival to


physician assessment. Data are reported using run chart which of the
following demonstrate a true statistical increase in treatment delays:

A) 6 consecutive ascending data points

B) 7 consecutive descending data points

C) zigzag pattern of 10 data points

D) data points are close to the mean line

510-standards of care based to the knowledge & experience of


recognized experts & healthcare research are known as:

A) benchmark data

B) generic screen

99 of 268
C) pre-established criteria

D) evidence based guidelines

511-which is the following is an essential component in performance


improvement report:

A) governing body approval

B) data analysis & display

C) individual performance review

D) team composition and attendance

512-the evaluation of quality & appropriateness of patient care in the


radiology department is the responsibility of:

A) medical director of radiology

B) chief medical officer

C) medical director of quality department

D) administrator of clinical service

513-Meauring the time it takes a nurse to perform a procedure address


which of the following:

A) monitoring

B) process

C) outcome

D) structure

the TIMELINEES of any procedure is a PROCESS INDICATOR :)

514-The concept of organizational liability is important to the field of


healthcare quality because it holds the organization responsible for:

A) ensure confidentiality of all documents

B) requiring physician to carry adequate malpractice insurance

C) maintain a process to identify deficiencies in the provision of care

D) assure that peer review physicians have no interest in cases being review

100 of 268
516-A healthcare quality professional is reviewing data with a wide
range of values between the highest & the lowest points. The best way
to rank order using a:

A) line graph

B) simple frequency distribution

C) grouped frequency distribution

D) bar graph

517-Which of the following tools should be used FIRST when developing


a performance improvement action plan:

A) story board

B) cause & effect diagram

C) interrelationship diagram

D) statistical process control chart

518-Analysis of post-operative surgical infection rates over the past year


demonstrate an increase in infections. Which of the following BEST
provides display of aggregated data compared to data from a local
facility?

A) bar graph

B) control chart

C) pareto chart

D) histogram

519-Team performance is BEST evaluated by:

A) the team leader

B) senior leadership

C) the PDCA process

D) the nominal group technique

520-Replacing retrospective review with concurrent review is an


example of:
101 of 268
A) a paradigm shift

B) a process improvement

C) an empowerment process

D) productivity enhancement

521-Which of the following should be appointed to quality improvement


council to deal effectively with conflict?

A) facilitator

B) chief operating officer

C) risk management

D) senior leader

522-Meaningful quality process measures must be:

A) relevant & valid

B) feasible&explainable

C) relevant& explainable

D) valid& identifiable

523-The BEST way to facilitate change within a healthcare organization


is to:

A) involve the individual directly affected by the change

B) communicate through group meeting

C) arrange presentation by senior leaders

D) communicate through group e-mail

524-A healthcare quality professional hypothesizes that a sample mean


is different from population mean. The standard deviation is not known.
The estimate must be calculated from a sample .which of the following
statistical technique should be used:

A) regression analysis

B) chi square test

C) t-test

D) analysis of variance

102 of 268
525-Which of the followings are measures of central tendency?

A) mean,mode,median

B) standard deviation,variance,standard error

C) grouped data,bell currve & distribution

D) correlation,regression & t-test

526-The following is the FIRST step in facilitating change in an


organization?

A) review customer satisfaction survey

B) get feedback from staff on problems to be addressed

C) identify key people in the organization that should be involved

D) develop a performance improvement plan

527-Private insurer has contracts with 2 area hospitals. Hospital A is a


250 bed community facility & hospital B ia a 900-bed tertiary care centre.
Costs were less at the tertiary care centre. From a financial stand points,
the BEST option to private insurer is to:

A) maintain both contract with no further action

B) monitor the expenses of both institution for the next year

C) negotiate an exclusive arrangement with hospital B

D) recommend that hospital B acquire hospital A

528-Which of the following is the BEST way for a healthcare quality


professional to involve nursing staff in the restructuring of a patient
careunit:

A) present at a department meeting

B) ask the nurse leader for recommendation

C) conduct a focus group

D) survey the nursing staff

530-A surgery department's monthly case review revealed 10 records


meeting criteria.6 records did not meet the criteria. When calculating the
incidence rate,the denominator is:

103 of 268
A) 4

B) 6

C) 10

D) 16

531-Quality improvement requires healthcare quality professional to


recognize that:

A) quality improvement generates its own change

B) the process is an ongoing continuous & dynamic change

C) the process require radical change in a short period of time

D) quality improvement is managed by senior leaders

532-For which of the following is process capability BEST used?

A) identifying if a process is having the intended effect

B) focusing a team on the best thing to do

C) narrowing down options through a systematic approach of comparison

D) determinig if a process meets established specifications

533-Which of the following is essential to an effective quality council?

A) involvement of leadership

B) consultation of the legal advisors

C) participation of the strategic planning committee

D) direction from the organization's quality department

534-The most effective way for healthcare quality professional to


communicate quality improvement activities to the medical staff is by:

A)developing professional relationships

B)inviting medical staff to an in service on quality tools

C)evaluating physician participation on quality teams


104 of 268
D)providing outcome data at medical staff meeting

535-An utilization management department of a hospital has collected


data on length of stay & readmission rates. Compared to benchmarks.
The length of stay rates are higher & readmission rates are lower. Which
if the following is the next step?

A) identify additional benchmarks to compare the data

B)conduct a cost-benefit analysis

C)analyze readmission rates with a run chart

D) continue to monitor length of stay rates

534-Benchmarking is based on identifying:

A)best practice

B)competition

C)deficiencies

D)statistical control

535-Which of the following BEST describe an organization vision


statement?

A)it's used as a marketing strategy

B)it define the structure of the institution

C)it describe the organization strategic plan

D)it reflect the organization culture

536-In deciding to submit an application for an external quality award.


The FIRST step is to determine if the award criteria:

A)are aligned with the organization strategic plan

B)are well written

C)demonstrate excellence in quality

D)are approved by the chief executive officer

105 of 268
537-A reengineering effort occurred at a facility. Activities particularly
those regarding staff layoffs ,were carefully planned ,communicated
&impliminted according to the plan. One year later, the business is
stable, but staff morale is very low. A healthcare quality professional
has been asked to consult in determining where the effort went wrong.
Based on the concepts of change theory, the cause is most likely:

A) that the reengineering decision was a mistake

B)a failure to address the need of the staff who were retained

C)that leadership was not probably trained in the change process

D) that a few disgruntled staff are instigating dissension in the ranks

538-The master patient index is considered :

A. A permanent file of all patients seen in the organization


B. A file of cases attended by individual physicians
C. A file of principal diagnosis codes.
D. A file of principal procedures.

539-The physician index is considered :

a. A permanent file of all patients seen in the organization


b. A file of cases attended by individual physicians
c. A file of principal diagnosis codes.
d. A file of principal procedures.

540-The disease index is considered :

a. A permanent file of all patients seen in the organization


b. A file of cases attended by individual physicians
c. A file of principal diagnosis codes.
d. A file of principal procedures.

541-The surgery index is considered :

a. A permanent file of all patients seen in the organization


b. A file of cases attended by individual physicians

106 of 268
c. A file of principal diagnosis codes.
d. A file of principal procedures.

542-Among registers which are permanent chronological listings for


maintaining certain statistics :

a. Surgical log
b. Surgery index
c. A and B
d. None of the above.

543-Regression analysis is a statistical technique that :

a. Compares two sets of like things using means.


b. Compares ratios and rates using tally data.
c. Compares distribution of observations of one variable with
the distribution of another.
d. Evaluates kinds of data found in pareto charts.

544-In comparing rates of nosocomial infection rates between two


hospitals the following test is commonly used:

a. The t-test
b. Linear regression analysis test.
c. The Chi-square test.
d. Standard deviation.

545-The separate services of Pharmacy and Nursing are having difficulty


developing an action plan for medication errors. Pharmacy Services states
that Nursing Services causes the majority of the problems related to errors,
while Nursing Services states the opposite. The quality professional’s role in
resolving this problem is to?

A. Provide them with directives on how to solve the problem

B. Facilitate discussion between the groups to enable them to

assume ownership of their portions of the problem

C. Assign the task to an uninvolved manager

D. Refer the problem to the facility-wide quality council

546-A quality manager needs to assign a staff member to assist a medical


director in the development of a quality program for a newly established
107 of 268
service. Which of the following staff members is most appropriate for this
project?

A. A newly hired staff member who has demonstrated competence


and time to complete the task

B. A knowledgeable staff member who works best on defined tasks

C. A motivated staff member who is actively seeking promotion

D. A competent staff member who has good interpersonal skills

547-A social service department regularly monitors

• the number of inappropriate referrals,


• the timeliness of discharge planning, and
• the number of days of discharge delays.
What additional monitor should be added to evaluate the appropriateness of
social serviceinterventions?

A. Inadequacy of documentation in progress notes

B. Attainment of social service goals

C. Timeliness of referrals to social services

D. Number of social service referrals from nursing

effectiveness = how the care or service would achieve z OUTCOMES not the
Goals..
appropriateness = how z care or service would be RELEVANT to z individual
s NEEDS or in accordance wz z PURPOSE WHICH = ur GOALS from the
start r they achieved ?!

548-A patient was in the operating room when a piece of a surgical instrument
broke off and was left in the patient’s body. The patient was readmitted for
removal of the foreign object. Which of the following would most likely apply
in this situation?

A. Res ipsa loquitur

B. Contributory negligence

C. Contractual liability

D. Tort liability

549-Which of the following is most likely to be a benefit of concurrent


ambulatory surgical case review?

Decreased medical record review at discharge .A


108 of 268
B. An increase in the number of cases failing screeningcriteria
C. An increase in reviewer competence
D. Decreased employee turnover

550-In order to perform a task for which one is held accountable, there must
be an equal balance between responsibility and

A. Authority

B. Education

C. Delegation

D. Specialization

551-The primary purpose of an emergency preparedness program is to

A.Conduct evaluations of emergency training

B.Provide evaluations of semiannual evacuation drills

C. Prevent internal disasters that disrupt the facility’s ability to provide care
and treatment
D. Manage the consequences of disasters that disrupt the facility’s ability to
provide care

T MAGIC

552-“Underuse is evidence by the fact that many scientifically sound


practices are not used as often they should be, For example, biannual
mammography screening in woman ages 40 to 69 has been proven
beneficial and yet is performed less than 75 percent of the time.” This is
the categorization of:

A. Defects

B. La of professionalism in Medical field

C. La of care

D. Healthcare practice

553-__________ is a term applied when the proper clinical car process is not
executed appropriately, such as giving the wrong drug to a patient or
incorrectly administering the correct drug.

109 of 268
A. Underuse

B. Overuse

C. Misuse

D. Illegal use

554-which of the following is example of outcome measure:


a.mortality rate.
b.average LOS.
c.medication dispensing rate.
d.labspeciemen.

555- Which one of the following BEST describes organizational culture


and change?

A. It is best to move quickly with change.


B. Professionals are relatively easy to convince of the need for change.
C. The factor most often misjudged in the change process is the
magnitude of change required and the staff response to change.
D. Ingrained mindsets are incapable of change; thus, they should simply be
eliminated from the change process.

556-Which of the following is the MOST common reason for failure of


quality improvement initiatives?

A. The expectation that managers will become part of the change process
B. The expectation that staff will become part of the change process
C. Viewing quality improvement as a long-term process
D. Viewing quality improvement as a short-term fix

557-The best view of outcomes is that they should be ____ focused,

A. patientB. physician
C. staff D. statistically

558-Which of the following is the MOST common mistake in the use of


consultants?

A. Following their recommendations blindly


B. Not following their recommendations at all
C. Hiring based on cost
D. Not checking the references and track record of the consultant

559-From the standpoint of the hospital, which one of the following best
fits all three roles of customer, processor, and supplier?

A. CEO
B. Employee
C.Patient
110 of 268
D. Physician

560-The capitation rate paid for a Medicare member is ____% of the


adjusted average per capita cost.

A. 100
B. 95
C. 75
D. 66

561-Which of the following are considered to be coordinated care plans


under Medicare Choice?

I. Health Maintenance Organizations


II. Preferred Provider Organizations
III. Provider Sponsored Organizations

The CORRECT answer is:

A. I, II
B. I, III
C. II, III
D. I, II, III

562-Which one of the following is NOT true regarding the JCAHO's


interpretation of environment of care?

A. All employees must be determined competent before care is rendered.


B. Building evacuation is not required during quarterly fire drills,
C. Physicians are not included in this function.
D. Waste and utility systems management is a must know item.

563-Which of the following is NOT true regarding the JCAHO's no


smoking policy?

A. Exceptions to the prohibitions are usually determined by licensed,


independent practitioners.
B. Social rehabilitation settings may not require a licensed independent
practitioner’s order.
C. Exceptions are allowed in hospital-sponsored ambulatory care areas.
D. Exceptions are not allowed in adolescent and child patient care areas,

564-A histogram with a twin peaks appearance is also called (n) ___
distribution.

A. bimodal
B. high variability
C. isolated peak
D. minimal variability

565-Which of the following Is NOT a source of information for the


National Practitioner's Data Bank?
111 of 268
A. American Medical Association
B. Healthcare entities
C. Insurance companies
D. Medical boards

566-A laboratory can receive a waiver from meeting expanded


proficiency testing standards under the Clinical Laboratory
Improvement Act of 19b8 provided

A. all of its technologists and physicians are appropriately licensed/certified


B. the lab is in a state-recognized physician's office
C. the lab is hospital-based
D. the lab only performs simple tests

567-Which one of the following best describes the intent of OSHA


regulations?

A. The maintenance of hazardous materials


B. The maintenance of safe, healthful work conditions
C. The maintenance of medical devices
D. Conformance to federal labor law

568-Which one of the following is not part of the Patient Self-


Determination Act?

A. Informing requirements for hospitals


B. Informing requirements for physicians
C. Durable power of attorney
D. Living wills

569-Which one of the following is NOT true regarding peer review


immunity under the Healthcare Quality Improvement Act?

A. Clinical psychologists are included as possible plaintiffs


B. It applies only to hospitals using formal peer review processes.
C. It does not prevent federal antitrust actions.
D. It is considered to be a conditional legislation.

570-Rejecting the null hypothesis and concluding that population means


are not equal when they in fact are, is

A. known as a Type I error


B, known as a Type II error
C. less likely if the alpha is set at .05 (as opposed to .01)
D. less likely if the alpha is set at .10 (as opposed to .05)

112 of 268
571-The National Practitioner Databank was established by the ____ and
became operational in ____.

A. Health Care Quality Improvement Act of 19:86; 1986


B. Health Care Quality Improvement Act of 19i86; 1990
C. Joint Commission; 1986
D. Joint Commission; 1990

572-Which one of the following is NOT true regarding the National


Practitioner Databank?

A. Health care practitioners may self-query at any time.


B. Hospitals must query when a practitioner applies for privileges and. every
two years on practitioners on the medical staff or holding privileges.
C. Medical malpractice payers may query at any time.
D. State Licensing Boards may query at any time.

573-Nurse Practice Acts are examples of____ law.

A. administrative- B. common
C. public D.statutory

574-State Boards of Nursing are examples of ____ law.

A. administrativeB. common
C. public D. statutory

575- Informed consent i^not needed when

I. there is an emergency if there is an immediate threat to life 01^ health


II. experts agree that it is an emergency
III. the client is unable to consent and the ^legally authorized person cannot
be reached

The CORRECT answer is:

A. I, II B. I, III C. II, III D. I, II, III

576-Which of the following would NOT satisfy the disclosure element of


informed consent?

A. The patient has been informed of current course of medical status and
treatment
B. That the patient has been informed of the risks and benefits of various
treatment alternatives
C. The patient has been told that outcomes can be guaranteed
D. The patient has been given a professional opinion as to the best alternative

113 of 268
577-Which of the following would constitute voluntary standards used
as a guideline for peer review?

A. ANA Standards of Practice


B. Credentialing
C. Licensure
D. Nurse Practice Acts

578-A person who____ would NOT be covered under the Americans with
Disabilities Act of 1990.

A. has a communicable disease such as AIDS or HIV


B. has committed a felony
C. is recovering from drug or alcohol addiction
D. is regarded as disabled, whether or not he/she is in fact disabled

579-The error in reasoning made by American business regarding.


Japanese competition in the 1960s and 1970s was tha.1

A. no one -will buy Japanese products


B. jobs will never be exported to Japan
C. Japanese competition is primarily. cost competition, not quality
competition
D. statistics are worthless in the business world

580- A manager who ____ best shows leadership in action.

A. identifies several new markets for the company based on strategic planning
B. advocates a one right way of doing things
C. is always pleasant with her employees
D. is well-viewed in the community

581-To ____ customers is the best delineation of a goal, as it is the least


subject to interpretation.

A. identify
B. know
C. appreciate
D. understand

582-In the hospital service delivery process, recent emphasis has been
on

A. admissions criteria
B. aftercare
C. continued stay criteria
D. preadmission processes

583-The composition and qualifications of staff BEST describes staff

A. allocation
114 of 268
B. FTE
C. mix
D. roles and responsibilities

584-Which of the following is the best description of the JCAHO's view


of model adoption for CQI?

A. The ten-step process is a mandatory component.


B. Outdated models, such as Shewhart's PDCA cycle, should be avoided,
C. Multiple models are preferred.
D. The institution must adopt whichever model best fits its needs.

585-JoEllen Smith is determining how many clients still became sick


from influenza after receiving flu shots at her facility. Under the
JCAHO's Dimensions of Performance, she is evaluating

A. appropriateness
B. effectiveness
C. efficacy
D. safety

586-When JoEllen evaluates how many people in her facility's managed


care plan were able to receive the flu shot. She is evaluating, under
Dimensions of Performance

A. appropriateness
B. availability
C. effectiveness
D. efficacy

587-When JoEllen evaluates how many people in her facility's managed


care plan were happy with the service received while receiving the flu
shot, she is evaluating, under Dimensions of Performance,

A. effectiveness
B. efficacy
C. respect and caring
D. safety

588- Robert Smith determines that the laboratory is the cause of the
most waiting in his hospital, followed by radiology, and admissions.
When he decides to place most of his efforts to decrease wait times by
starting with the lab, he is

A. monitoring quality costs


B. Pareto planning
C. rank-ordering
D. stratifying data

589-The fewest number of measures in performance monitoring should


be in the area of

115 of 268
A. process
B. outcome
C. structure
D. utilization

590-An integral part in the process of determining medical necessity in


the delivery of quality medical care including weighing the potential for
undesirable outcomes and side effects against the potential for positive
outcomes of a treatment best describes

A. cost-effective medical care


B. efficacious medical care
C. efficient medical care
D. risk-benefit analysis

591- ____ is correlated to. the timeliness of delivered medically


necessary standard of care services and supplies which result in the
least cost.

A. Cost-effective medical care


B. Efficacious medical care
C. Efficient medical care
D. Risk-benefit analysis

592-The selection of the least expensive medically necessary treatment


from two or more that are equally efficacious in achieving a desired
health care outcome is termed

A. cost-effective medical care


B. efficacious medical care
C. efficient medical care
D. risk-benefit analysis

593-Quality improvement and risk management are ____ in the long run,
____ risk translates into ____ quality.

A. not related; reduced; no change


B. interrelated; higher; higher
C. interrelated; reduced; reduced
D. interrelated; reduced; higher

594- ____ risk is best described as the risk of doing business.

A. Claims
B. Managed care
C. Marketing
D. Operational

595-Health economic research that focuses on personal satisfaction


after retirement would best be described as ____ research

A. cost utilization
116 of 268
B. cost minimization
C. outcomes
D. quality of life

596-Which one of the following is NOT a typical means by which


managed care organizations try to reduce costs?

A. Denial of non-emergency ER visits


B. Denying all services
C. Increased use of PAs and NPs
D. Increased emphasis on prevention and self-care

597-The data shared between institutions to see how each one is doing
is BEST defined as ____ data

A. comparative
B. definitive
C. normative
D. relational

598-If a study has 40 true positives and 10 false negatives, then its
sensitivity is____%.

A. 8
B. 80
C. 88
D. 90

599-Our Lady Hospital, finding that 80% of its patients are poor over 65
years of age, decides to develop programs specifically for that group.
This is called

A. assessment
B. improvement
C. reiteration
D. statistical analysis

600-Nancy Smith designs a study to look at two individuals working in


the hospital; RNs and CNAs. This is an example of a(n) ____ variable.

A. acentric
B. causal
C. dichotomous
D. twin

601-The documentation of basic treatment or action sequence in order


to eliminate unnecessary variation best describes

A. benchmarking
B. critical path
C. flow charting
117 of 268
D. variation reduction

602-Under JCAHO's Cycle for Improving Performance, historical


patterns of performance in the organization is also called

A. baseline performance
B. benchmarking
C. designed performance limits
D. practice guidelines and parameters

603-The steps involved in JCAHO's Cycle for improving performance are

A. design, measure, assess, and improve


B. find, organize, clarify, understand, and select
C. focus, adapt, develop, and evaluate
D. plan, do, check, and act

604-Which of the following would be the BEST means of charting


continuous data?

A. Fishbone diagram
B. Flowchart
C. Histogram
D. Pie chart

605-(1) Call patient by name; (2) Walk patient back to radiographic room;
(3)Perform x-ray exam; is an example of

A. flowcharting
B. improvement cycle
C. planning
D. process

606-Joe Smith wants to study patient satisfaction in his institution but


wants to get the largest group possible so he conducts his study in the
local mall. His study might be criticized not only for reaching individuals
who are not patients, but also that it is

A. capitated
B. nonrandomized
C. randomized
D. variated

607-Your 6 units receive patient satisfaction scores of 56, 66, 68, 70, 78,
89. What is the mean score for the group?

A. 56
B. 70
C. 71
D, 89

118 of 268
608-Your 6 units receive patient satisfaction scores of 56, 66, 68, 70. 78,
89. Which of the. following are accurate representations of range?

I. 56-89 II. 33 III. 71/33

The CORRECT answer is:

A. I, II
B. I, III C. II, III
D. I, II, III

609-Death rate as an indicator for the quality of care

I. often ignores case mix


II. often ignores regional differences
III, is the best indicator of physician performance

The CORRECT answer is:

A. I, II
B. I, III C. II, III
D. I, II, III

610-Jane Smith develops a patient survey and tests it on a number of


patients, who can understand it. She receives fairly identical comments
and results. She can consider that this survey meets standards of ____
and ____, but perhaps not ____,

A. readability; reliability; validity


B. validity; readability; reliability
C. reliability; validity; readability
D. accuracy; validity? Readability

611-The method of case management with the longest history is ____


case management.

A. institution based
B. private based
C. reimbursement
D. social welfare

612-The three displays of a scattergram show

A. large relation, small relation, narrow relation


B. normally distributed, left skewed, right skewed
C. positive relation, neutral relation, negative relation
D. skewed relation, normal relation, inter-related

613-An affinity diagram groups

A. identical ideas
119 of 268
B. people by skill
C. people by job
D. related ideas

614-The Process Decision Program Chart (PDPC) is a combination of


____ and____ diagrams.

A. Fishbone; flow
B. Fishbone; GANTT
C. Fishbone; tree
D. GANTT; affinity

615-Whereas strategic planning focuses on ____, tactical planning


focuses on ____.

A. immediate; long range


B. long range; immediate
C. overview; the process
D. the process; overview

616-If step one in a quality improvement program is to clarify goals, then


the next logical step is to

A. analyze data
B. educate and build the –team?
C. investigate the process?
D. take appropriate actions

617-The best description of a specification is

A. measurable aspects of a producer service that match customer needs


B. specific aspects or attributes of a product or service
C. written documents
D. written protocol or policy for action

618-The ideal flow of information in a business or organization is from

A. customer to worker to manager


B. manager to supervisors to workers
C. manager to workers
D. vendor to manager to worker

619-Which of the following would most likely NOT represent a typical


CQI team?

A. Mammographic image quality: radiologist, technologist, physicist


B. Patient satisfaction: admissions clerk, staff RN, MD, patient representative
C. Patient transport: transporter, radiology representative, lab representative,
staff RN
D. Product supply: staff technologist, staff RN, medical records clerk

120 of 268
620-The most realistic outcome of Continuous Quality Improvement is
an

A. improvement may not work, but it is a useful learning tool


B. improvement will always work if properly carried out
C. improvement will work most of the time but people usually mess them up
D. outcome is a stepping stone to the next higher step

621-Symptomatic treatment is considered inadequate in CQI and


performance improvement; instead, the goal must be to

A. determine the root causes of a problem


B. find all problems
C. fix all problems
D. fix problems at the lowest cost

622-GANTT charts define ____ and ____.

A. internal processes; external processes


B. management responsibilities : employee responsibilities
C. problems; who fixes them
D. who does what; when

623-In a systems approach, the most important step to avoid frustration


and keep working toward future excellence is

A. converting customer needs to expectations


B. output or outcomes identification
C. selection of appropriate measures
D. the ability to go back to a prior step if a barrier occurs in the process

624-Critical pathways would most likely be developed first for diagnoses


and procedures that are high

A. cost
B. risk
C. volume
D. all of the above

625-When should patients be advised of their rights?

A. Prior to entry into a healthcare setting


B, At entry into a healthcare setting
C. Following entry into a healthcare setting
D. Patients must ask for explanations of their basic rights

626-The Mammography Quality Standards Act requires direct reporting


to patients in which form?

I. Oral II. Written III. In lay terms


121 of 268
The CORRECT answer is:

A. I. II
B. I, III
C. II, III
D. I, II, III

627-In development performance improvement action plan, this of the


following tools should be used first

a-control chart
b-cause and effect diagram
c-interrelation ship diagram
D-pareto chart
628-quality improvement team are beneficial because they

a-maximize expertise and perspective

b-promote competition and pride among members

d-authorize solutions to problems

629-The first step in the design process of a QI plan is

a. determine the scope of the organization. . . . . . . . . “it is not a strategic plan”

b. make a cost-benefit analysis

c. establish performance objectives

d. establish the project goals

630-The strategic plan is used to

Achieve the mission

Achieve tactical objectives

631- "Occurrence reporting" is a type of

a. risk reduction

b. risk evaluation
122 of 268
c. risk identification

d. risk prevention

632- 60 years female spend 2 more days in hospital due to medication


reaction This is

Medication error

Unexpected adverse occurrence

633- which is the best tool used in "generic screening"?

a. medical record

b. claims data

c. incident report

d. performance indicators

634- Which of the following is NOT included in a utilization management


plan

a. procedures for discharge

b. organization's reporting structure

c. confidentiality agreement

d. annual budget

635-Morbidity rate for obstetric measured by

Septisemis after delivery

Ceaserin rate

Normal delivery rate

636- A level of cohesion between team members is reached in

a. forming stage

b. norming stage
123 of 268
c. storming stage

d. performing stage

637-During performing a re-appointment for a physician, which of the


following can be

a. blood utilization review

b surgical case mix

c. Medical Record Completion Review

d. fall rate review

638- In a behavioral healthcare facility, CQI team was working for one
year to decrease chemical and physical restraint. After applying the
program, the falls with subsequent injuries increased for one standard
deviation. The following action is:

a. return to the physical restraint and increase monitoring


b. continue the program and seek opportunities to decrease falls
c. stop the program and re-engineer the process
d. use more chemical restraint and educate physician

639- Stability Of Infection Control Rate Over Time

run chart

control chart

640-Successful development of guidelines

A-physician involvement

b-staff education

c-QI tools

d-pt education

641-organizational committed to improving patient safety key area to


influence change are:

124 of 268
A-staff willingness to change , policies . procedure,
equipment ,technology ,evaluate process ,redesign the structure , improving
of morale

b-structure ,environment , equipment , process ,application ,


leadership ,culture

c-medication delivery , structure , staff willingness to change ,non-slip


floor ,improvement of wards environment

d-leadership , culture , policies &procedure ,staff incentives , better


lightening , evaluate process &EMR

642- which is attributes to culture of safety

a-transparency & increase pt acuity level

b-error-proof environment of empowered staff

c-empower staff &transparency

d-increase pt acuity level& error proof environment

643-which best enhance patient safety program

a-online staff survey

b-bar code of supplies

c-video pt monitoring

d-EMR

644-under conducting a sentinell event review a RCA is

a-provide judjment of staff behavior

b-require team consensus

c-identify gap in pt care behavior

d-proctively identifies couses & effects

645-One of the team members that keep team members on track & focus
on the process is:

A.Leader

B. Facilitator

C. Time keeper

125 of 268
D. Recorder

646-When implementing a breakthrough improvement to an admission


process, the hospital can benchmark with the practice of:

a. Other health care organizations than a hospital

b. Evidence from literature

c. Hotels and resorts

d. Neighbor hospitals

647-aproprietness of appendectomy

preadmission test

pathology test

age

clinical test

648-alarm on iv infusion was broken nurse evaluate the patient she


should

call physician

call to fix the alarm

call the head nurse

649- the physical difference between traditional QA & QIthat QI is:

a- focus on process& systems while QA focus on individual faults

b- stress peer review while QA focus on customer

650-Managing data and assisting with mathematical calculations is the


best description of
A. database
B. graphics
C. spreadsheet
D. word processor

651-Which individual can be seen as serving as an internal consultant for the


organization in an accreditation survey?
A. Chief executive officer
B. Chief financial officers
C. Chief operating officer
D. Quality manager

126 of 268
652-The least preferred outcome of QI:
a- empowerment
b- increased statistical data
c- increased communication between staff
d- customer satisfaction

653-A Team has been tasked with developing a program to prevent patient
falls which of the following data elements from an incident\ occurrence report
would provide the most useful information for the team in evaluating the pro -
gram success?
A-patient demographic
B-record of the time of the fall
C-nursing assessment
D-staffing ratio at the time of the fall

654-Physician asks the nurse to cancel EKG for a patiThe nurse forget
to record this cancell ent. ation & then the patient die .the physician
should
Do nothing since the cancellation is the cause of death-A
Ask the nurse to write cancellation in the medical record-B
Add an addendum to the record that the EKG had been cancelled-C
The physician add the cancelled order to the record-D

655-External benchmarking can be


A. useful because all institutions are different.
B. misleading because all institutions are different.
C. meaningful when different populations are sampled internally.
D. misinterpreted because the data are so vast.

B. misleading because all institutions are different.


External benchmarkinginvolves analyzing data from outside an institution,
such as monitoring national rates of
nosocomial (hospital-acquired) infection and comparing them to internal rates.
To make this data meaningful, use
the same definitions and the same populations for effective risk stratification.
Using national data is informative,
but each institution is different, so relying on external benchmarking to select
indicators for Infection Control or
other processes can be misleading. Benchmarking is a compilation of data
that may vary considerably if analyzed
individually, and its anonymity makes comparisons difficult. Internal
trendinginvolves comparing internal rates
of one area or population with another, such as infection rates in ICU and
General Surgery. Trending can help to
pinpoint areas of concern within an institution, but making comparisons is still
problematical because of inherent
differences. Use a combination of external and internal data to identify and
select reliable indicators.

127 of 268
656-which of the following is most commonly omitted from the Q. assessment
& improving:
a-reporting results of studies in a timely manner
b-determining the effectiveness of actions taken
c-defining criteria
d-delegating data collection activities

657-a report that helps customers to evaluate Q. of care & services of


health plans or providers is required to:
a-show comparative performance in specific areas
b-identify areas for performance improvement

658-Which one of the following is not a way to promote organizational


values and commitment among the staff?
A. Establish departmental goals as mandated by the governing agencies.
B. Assist teams to develop good listening and collaborative skills.
C. Build upon the beliefs of the staff to establish a mindset for continuous
improvement.
D. Encourage staff to create personal goals within the organizational Mission
and Vision.

659-comparison of surgical wound infection rate is an eg. of:


a-surgery profiling
b-practitioner credentialing

660-who is responsible for providing organizational direction for a


facility's CQI ?
a-facilitator

b-quality council

c-leader

d-teams

661-a facility decide to implement standard precautions 1year ago,but


compliance of the staff has beeb poor,to assess the causes of poor
compliance& improve it,the most effective way of the organization is to:
a-initiate testing as a part of staff competency
b-show a videotape on standard precautions quarterly
c-review & revise policies & procedures

662-Healthcare associated infections / injuries is regarded as: .


a. Sentinel event
b. Incompliance with standards.
c. Mal-practice.
d. Negligence

663-The organization.s strategic goals are best linked to its performance


improvement activities by management
128 of 268
A. offering many performance improvement models from which each area can
choose.
B. assigning improvement models to areas as deemed appropriate by the key
leaders.
C. obtaining organizational consensus for continuous improvement activities.
D. analyzing the goals and improvement activities of other similar
organizations

664-design of piece of device make error this type of error is


a.active
b.negligence
c. latent
d . organizational

665-healthcare quality program had prepared a balanced score card that


displayed patient satisfaction was 98%, financial target has been met ,
medication error had been increased by 30% and heart surgery rate
decreased 3% , what additional information the governing body may ask
for?
a) type of medication error.
b)heart surgery case.
c)patient satisfaction data.
d)review patient compliant .

666-brainstorming is best used to:


a-subdivide or organize a large no. of ideas
b-identify, analyze ,or plan solutions to problems
c- identify cause & effect relationships

667-to display the best reimbursement/cost ratios for procedures


performed in 3 ambulatory surgery centers in the last calender year, use
a:
a-scatter diagram
b-pie chart
c-control chart
d-Pareto chart

668-to reduce incidence of ventilator associated pneumonia in ICU,who


should b included in QI team:
a-ICU nurse,respiratory therapist & internist
b-pharmacist,nurse manager,infection control nurse
c-ICU manager,respiratory therapist,pharmacist
d-1ry care physician,infection control nurse

669-The quality of service of care refers to characterisitcs of the setting


in which encounter between patient and clinician takes place ,,such as :
a-comfort
b-comfort,care and access
c-comfort ,convenience and privacy

129 of 268
670-Staff education in organizational change include all of the following
except:
a- negotiating process
b- project & time mangement
c- conflict resolution
d- budjeting techniques

671-when considering use of external consultant,which of the follow.


characteristics is important:
a-cost of consultant's service
b-references of the consultant
c-leadership's personal preferences

672-A seamless continuum of care is a:


a. System oriented process.
b. Client oriented process.
c. Practitioner oriented process
d. Managerial oriented process

673-the First Key to determine when u could evaluate the current status
of A Quality Improvement Program :
a-Climate for change in each department and service
b-Extent of the leadership Knowledge and Improvement in Quality Activities.

674-The key goal of Re-engineering is to :


a-improve care process
b-satisfy the customer
c-position to change
d-redesign the organization

675-a nursery home 60% of residents complained of food that was


delivered cold as a quality professional you should do:
a.Call dietercian and ask for explanation.
b.Review previous Results & trend data.
c.do nothing

676-Which of the following information would be most useful


in performing SWOT analysis?
(A) The experiences of cross-functional teams
(B) Historical data on accounts receivable
(C) A report on the turnover rates of competitors
(D) Trends in market growth

130 of 268
678-All of these are considered barriers for communication except :
a-lack of interest
b-semantics
c-assumption
d-active listening

679-Evaluating medication administration to reduce medical errors is an


example of
A. risk management.
B. utilization management.
C. quality management.
D. financial management

680-Clinical performance measures in disease management programs


are based on
a. standards of practice.
b.clinical privilege criteria.
c.clinical pathways.
d.practice guidelines

681-One standard element of organization-wide strategic planning is


A. determining the future implications of the services offered by other
healthcaren facilities.
B. analyzing internal services and community needs.
C. adopting the fiscal and ethical standards of the corporate model.
D. adopting the strategic plans of successful organizations.

682-In a managed care organization (MCO), an appeal following a denial


of care or benefits :
a-may be reviewed by an independent external review process.
b-is a formal grievance filed by a patient.
c-is limited to insurance coverage issues.
d-may be reviewed by a patient advocacy group process.

683-As quality management director at Sunshine Community Medical


Center, you are conducting comparative analysis of the surgical wound
infection rate data between two surgical units. The "p-value" of the chi-
square test you run will help you draw what conclusion about the
relationship between the two sets of data?

a-Ratio of the two rates


b-Standard deviation of the difference from the mean
c- Proportion of the relationship
d-Significance of the relationship

131 of 268
684-The appraisal of individual practitioner performance in health care,
beyond minimum standards and criteria, is known as
A.continuous quality improvement. B.intensive analysis.
C.perceptive quality.
D.peer review.

685- Measuring the time it takes a nurse to perform a procedure address


which of the following:

A) monitoring

B) process

C) outcome

D) structure

686-In any hospital, developing (Continuous Education Programs) is


usually a function of which of the following departments?

a) Human Resources department.

b) Quality Management department.

c) Risk Management department.

d) Nursing department

687- When hospital (X) is a provider in the network of providers which


has contracts with the Managed Care Organization (Y) in order to
provide clinical services for the enrollees of the plans offered by this
MCO. If the contract between hospital (X) and the MCO (Y) will be
renewed; what type of information will be of high importance to the MCO
to know before renewal of the contract?

a) Admission rate, charges, and resource utilization.

b) Admission rate, costs, and resource utilization.

c) Charges and revenues.

d) None of the above.

688-The middle manager should be involved in any CQI program under


direct supervision from:

a) Governing body.

b) CEO.

c) Senior managers.

d) The Quality Council.


132 of 268
689-______________ can be measured by how well evidence-based
practices are followed, such as the percentage of time diabetic
patientsreceive all recommended care at eachdoctor visit, the
percentage of hospital-acquired infections, or the percentage of
patientswho develop pressure ulcers (bed sores) while in the nursing
home.

A. Safe care

B. Equitable care

C. Effective care

D. Timely care

690-healthcare quality professional can best communicate


organizational value and commitment through
A-leading by example
b-Disseminating monthly newsletters
C-establishing multidisciplinary task force
D-creating mission statement

691-Crossing the Quality Chasm provided a blueprint for the future that
classified and unified the components of quality through six aims for
improvement, chain of effects, and simple rules for redesign of
healthcare. The six aims for improvement, viewed also six dimensions
of quality. Which of the following is NOT out of those dimensions?

A. Safe
B. Care centered
C. Efficient
D. Effective

692-Today’s patients’ perception of the quality of our healthcare system


is not favorable. In healthcare,quality is household word that evokes
great emotion, including:
A. Frustration and despair, exhibited by patients who experience healthcare
services firsthand or family members who observe the care of their loved ones
B. Anxiety over the ever-increasing costs and complexities of care
C. Patient centered measures
D. Timely care that may be experienced in terms of performance of services
Answer:A, B

693-There is a story of an intensive care unit (ICU) at Dominican Hospital


in Santa Cruz Country, California. Dominican, a 379-bed community

133 of 268
hospital, is part of the 41-hospital Catholic Healthcare West system. “We
used to replace ventilator circuit for incubated patients daily because we
thought this helped to prevent pneumonia,” explained Lee Vanderpool,
vice president. “”But the evidence shows that the more you interfere
with that device, the more often you risk introducing infection. It turns
out it is often better to leave it alone until it begins to become cloudy, or
‘gunky,’ as the nonclinicians say.” The hospital staff learned an
important lesson from this experience that:

A. Evidence is more powerful than intuition

B. Intuition is more powerful than evidence

C. Efforts improve mortality rate

D. Introduction f a new protocol, or any new idea, involves education

694-A number of attributes can characterize the quality of healthcare


services. As, there are different groups involved in healthcare, such as
physicians, patients and health insurers, tend to attach different levels
of importance to particular attributes and as a result define quality care
differently. Which of the following is/are NOT out of those attributes?

A. Technical performance
B. Responsiveness to patient preferences
C. Excess staff
D. Amenities

695-Quality and technical performance refers to how well current


scientific medical knowledge and technology are applied in a given
situation. It is usually assessed in terms of:

A. Timeliness and accuracy of the diagnosis


B. Appropriateness of therapy and other medical interventions are performed
C. The quality of interpersonal relationships
D. Both A & B

696-The quality of amenities of care refers to the characteristics of the


setting in which the encounter between patient and clinician takes place,
such as:
A. Comfort

B. Comfort, care and access


C. Comfort, convenience and privacy
D. Responsive to patient preferences

key word is Amenities of care =


the quality of being pleasing or agreeable in situation, prospect or any feature
134 of 268
that provides comfort, convenience, or pleasure e.g bathroom,,parking
areas..an so on,,so ( sure privacy ll b needed in bathroom ) and it should b
convenient ,,,simply :

so he asked about the Quality of amenities Not Quality of Care here ,,this the
difference between B and C
697-Amenities may cover areas as mentioned below EXCEPT:

A. Ample and convenient parking


B. Good directional signs
C. Comfortable waiting rooms
D. Vast and facilitated food providing area

698-as a result of customer surveys, a new service is suggested to be


delivered. It should be written in :
a.financial plan.
b.strategic plan.
c.quality management plan.
699-to increase the effectiveness of reengineering program ,the quality
professional should :
a.reviewpolices and procedures.
b.plancarefully,communicate widely and lead effectively.
c.make a lecture for employees to define reenginering.

700-The roles and responsibilities of a process improvement team


should be established by which of the following hospital's authorities?

(A) The board of directors ..


(B) .(B) The human resources department

(C) (C) The team members’ supervisors

(D) (D) The quality steering committee

701- One of the most important follow-up activities for root cause
analysis (RCA) is to review the database of previous findings internally
and compare with related external databases, if available. The purpose
is to also look for

a. common cause.

135 of 268
b. special cause.

c. positive outcomes.

702-_________________ refers to the “degree to which individuals and


groups are able toobtain needed services.”
A. Responsiveness to patient preferences
B. Amenities
C. Equity
D. Access

703-In earlier formulations, responsiveness to patients’ preferences was


just one of the factorsseen as determining the quality of patient clinician
interpersonal relationship. But, now itis translated into many factors.
Which of the following is out of such factors?
A. Respect for patients’ values
B. Respect for patients’ preferences
C. Respect for patients’ expressed needs
D. Respect for Respect for patient’s convenience
Answer: A, B, C
BeITCertified.com
704-Efficiency refers how well resources are used in achieving a given
result. Efficiencywhenever the resources used to produce a given
output are ______
A. Reduces, reduced
B. Increases, increased
C. Improves, reduced
D. It is truly situation dependent

705-In general, as the amounts spent on providing services for a


particular condition grow,diminishing returns set in meaning that each
unit of expenditure yield ever-smallerbenefits until a point where _____
A. No additional benefits accrue from adding more care
B. Additional benefits are too small to justify the added costs
C. There is displacement of more useful care
D. perfection is within the reach of all individuals

706-“Quality is the degree to which health services for individuals and


populations increasethe likelihood of desired health outcomes and are
consistent with current professionalknowledge.” This is the definition of
Quality care often quoted by:
A. IOM
B. IHI
C. HQCB
D. OCHP

BeITCertified.com
707-“Likelihood of desired health outcomes” corresponds to clinicians’
view that, withrespect to outcomes, there are only probabilities, not
certainties, owing to factors-such aspatients’ genetically determined
physiological reliance-that influence:
136 of 268
A. The primary concerns of patients
B. Outcomes of care and yet are beyond clinicians’ control
C. Outcomes of care and now are within clinicians’ control
D. High cost interventions
708-In fact, because patients’ satisfaction is so influenced by
__________________ ratherthan to the more indiscernible technical
ones-health maintenance organizations, hospitalsand other health care
delivery organizations have come to view the quality of nontechnical
aspects of care as crucial to attractions and retaining patients.
A. Their reactions to interpersonal and amenity aspect of care
B. Patients recognize that they do not possess the wherewithal to evaluate all
technicalelements of care
C. Every patient has definite preference in every clinical situation
D. Their likelihood of desires outcomes

709-Payers are more likely to embrace the optimization definition of care


which can put themat odds with:
A. Clinicians
B. Health administrators
C. Physicians
D. Both A & B
BeITCertified.com
710-The manager’s perspective on quality differs markedly from that of
clinicians andpatients on:
A. Efficiency, effectiveness and access
B. Efficiency, cost effectiveness and equity
C. Responsiveness to patient preferences
D. Equity, access and technical performance

711-Strong disagreement do arise, among the five parties’ definitions


(i.e. the clinician’s, thepatient’s the payers, the manager’s and the
society’s), even outside the realm of costeffectiveness. Conflicts
typically arise when:
A. Practitioners who are highly skilled in trauma and other emergency care
B. Each group emphasizes a particular aspect of care
C. One party holds that a particular practitioner or clinic is a high quality
provider byvirtue of having high ratings on single aspect of care
D. The facility receives top marks from a team of expert clinicians whose
primary focusis on technical performance

712-All the evaluations of quality of care can be classified in terms of


one three aspects ofcare giving they measure. Which of the following
is/are NOT out of these measures?
A. Structure
B. Process
C. Output
D. Cutbas
BeITCertified.com
713-When quality is measured in terms of structure the focus is on the
relatively static characteristics of the individuals who provide care and
of the settings where the care isdelivered. These characteristics include
137 of 268
____________ of professionals who provide careand the adequacy of
the facility’s equipment, and overall organization.
A. Education
B. Training
C. Certification
D. A, B and C

714-Licensing and accrediting bodies have relied heavily on structural


measures of quality notonly because the measures are relatively stable
and thus easier to capture but:
A. They reliably indentify providers who are cheap
B. They reliably identify providers who demonstrably la means to deliver
high qualitycare
C. They can never la the means to deliver high quality care
D. They reliably identify physicians

715-Ordering the correct diagnostic procedure for a patient is a measure


of _________. Whenevaluating the process of care, however,
appropriateness is only half the story. The otherhalf is in how well and
how promptly (i.e. skillfully) the procedure was carried out.
A. Consciousness
B. Appropriateness
C. Care assessment
D. Equity
BeITCertified.com
716-Because of the goals of care can be defined broadly, outcome
measures have come toinclude the costs of care as well as patients’
satisfaction with care. In formulations thatstress the technical aspects
of care, however outcome typically refers to:
A. Health status-related indicators such as whether the pain subsided
B. Desired results
C. Appropriate and potentially harmless care
D. Special set of clinical activities

717-Knowledge about _______ is crucial to making valid judgments about


quality of careusing either process or outcome measures. If we know that a
given clinical interventionwas undertaken in circumstances that match those,
under which the intervention has beenshown to be efficacious, we can be
confident, that the care was appropriate and, to theextent of good quality.
A. Outcomes
B. Structure
C. Efficacy
D. Processes

718-Universities often evaluate applicants for admission on the basis of,


among other things,the applicants’ scores on standardized tests. The
scores are thus one of the criteria bywhich program judge the Quality of
their applicants. However, although two programsmay use the same
criterion – scores on a specific standardized examination-to
evaluateapplicants, the programs may differ markedly on standards:
One program may considerapplicants acceptable if they have scores
above the 50th percentile, whereas the scoreabove the 90th percentile
138 of 268
may be the standard of acceptability for the other program. Thisexample
clearly defines the difference between:
A. Sources and structure
B. Criteria and standards
C. Processes and outcomes
D. Efficacy and equity
719-For cheking the outcomes our focus of attention is blood pressure
of patients with diabetes.Its criteria and standard can be respectively:

A. Criterion: Percentage of post heart atta patients prescribed beta-bloers on


dischargeand Standard: At least 96% of heart atta patients receive a beta-
bloer prescription ondischarge
B. Criterion: Percentage of patients with diabetes whose blood pressure
is at or below130/85 and Standard: At least 50% of patients with diabetes
have blood pressure at orbelow 130/85
C. Criterion: Sugar level in blood on daily basis and Standard: How many
times sugarlevel rises and how many times it declines in a week
D. None of these

720-When formulating medical standards, a critical decision that must


be made is the _____at which the standard should be set.
A. Depth
B. Clarity
C. Level
D. utility of measurement

721-Which of the following is a value-added activity?

A.Inspection
B.Just-In-Time Inventory
C.Defect Correction
D.Waiting

722-For a normal distribution, two standard deviations on each side of


the mean would include what percentage of the total population?
a. 47%
b. 68%
c. 95%
d. 99%

723-The roles and responsibilities of a Champion include all of the


following EXCEPT:
A.A Champion selects the Team Leader.
B.A Champion reviews team progress.
C.A Champion coordinates team logistics.
D.A Champion assures the use of Six Sigma methods and tools.

724-The objective of the define phase of a six sigma project is to define:

139 of 268
A.The quality policy
B.The customer, core business process involved and CTQ business issues
C.The statistics work instructions
D.The procedures manual for control charting

725-What is the Goal of Lean Manufacturing?

A.Reduction of defects
B.Elimination of waste
C.Increased profits
D.All of the above

726-An early step in any project must be seeking the voice of the
internal and external customers of a project. This statement is:
A.False, because projects should only be concerned with external customers.
B.True, because both internal and external customers can be impacted
by the project.
C.True, because a project team wants to maintain good will with all
customers.
D.False, because internal projects only impact internal customers.

727-What is the best way to select six sigma projects when addressing
customer satisfaction issues?

A.Problem Focus
B.Product Focus
C.Project Cost Savings Focus
D.Process Focus

728-The primary purpose of a project charter is to


(A) subdivide the project into smaller, more manageable
components
(B) provide management with a tool for selecting a project
that addresses business needs
(C) provide management with a tool to ensure that project
deadlines are met
(D) provide the project manager with authority to apply
organizational resources to project activities

In project management, a project charter or project definition is a statement


of the scope, objectives and participants in a project. It provides a preliminary
delineation of roles and responsibilities, outlines the project objectives,
identifies the main stakeholders, and defines the authority of the project
manager. It serves as a "reference of authority" for the future of the project.
The terms of reference is usually part of the project charter

140 of 268
729-Which of the following is NOT a key element of the define phase?
A.Measure
B.Process mapping
C.Project charter

730-the key dimensions of quality that are related to utilization


management are:

1. appropriateness
2. availability
3. continuity
4. effectiveness
5. Timeliness

731-In the community health clinic, at least four complaints have been received
per month for the past four months, compared to an average of one per month
for the six prior months. The average number of patients seen per month is
2000. The trigger is >0.2%. What is the appropriate response?
a. Select a more useful indicator.
b. Perform intensive analysis now.
c. Trend for at least three more months.
d. Reward the entire staff.

732-Who is ultimately responsible for the effective implementation of the


quality program:
a. Governing Body
b. CEO
c. All staff
d. The CFO
733-Mission statement means:
1. what the organization strives to be?
2. what to do to be successful?
3. purpose of the organization
4. where are we going?
734-Which part of a job description should be used in a criteria-based
performance evaluation?
A. salary grade
B. duties and responsibilities
C. working conditions
D. qualifications

141 of 268
735-based on your knowledge of motivational theory ,which of the
following would most attract a job candidate
a. job description ,salary ,benefits
b.job description ,salary ,excellent working relationships
c.job description ,salary ,opportunities to advance
d. salary ,benefits ,retirement plan
736-teaching the use of QI ''process tools '' is more effective when
a.all possible tool options are covered
b. statistical process control is covered first
c. the team needing the tool is meeting together
d.watching a videotape

737-in general accrediting organizations expect to see conclusions


drawn from quality improvement used in:
a-utilization review
b-risk management decisions
c-productivity management
d-recredentialing LIPs

738-during the team meeting,the facilitator should do 1st:


a-choose homogenous group
b-make ground rules
c-make rapport to the group
d- instruct orders

739-A medication is ordered for a diabetic patient. Its capacity to


improve health status, as a dimension of quality of performance, is its:
A. Effectiveness
B. Potential
C. Appropriateness
D. Efficacy

740-If, in the continuous improvement process, we increase our


emphasis on customer satisfaction and outcomes of care, which two
dimensions of quality/performance must be incorporated into all quality
management activities?
A. Availability and respect/caring
B. Respect/caring and competency
C. Effectiveness and respect/caring
D. Continuity and competency

741-Review of the timeliness of high risk screening for diabetes


addresses which focus?
A. Outcome of care
B. Processes of care
C. Structure of care

142 of 268
D. Administrative procedure

742-The centerpiece is of “outcomes management” in healthcare is:


A. The measurement of the patient ‘s functionality and quality of life
B. Morbidity and mortality
C. Data reliability
D. Financial impact

743-Within the context of total quality management philosophy,


communication of quality is:
A.The responsibility of top management leaders
B.Delegated to the quality management department
C.An internal organizational, not community, issue
D.Independent of processes budgets or costs

744-Which of the following key healthcare issues is more problematic


for ambulatory care than for inpatient care?
A. Reimbursement of care
B. Access to specialty care
C. Appropriateness of treatment setting
D. Quality of care provided

745-One fundamental differences between monitoring product quality


and service quality is based upon the fact that:
A.A service is easier to measure and verify in advance
B.A service is not perishable
C.A service is more heterogeneous than an object
D.There are more service delays than product delays

746-The task of setting up an ambulatory care setting QM/PI program


that focuses on “outcomes” as a measure of treatment effectiveness is
difficult because:
A. The patient remains in control of treatment
B. Patient care outcomes are determined by the payer
C. There are no required medical records
D. Expected outcomes for ambulatory conditions are too obvious
747-The function in the Juran Quality Management Cycle that includes
the initial analysis of data/information is:
A. Quality planning
B. Quality initiative
C. Quality control/measurement
D. Quality improvement

143 of 268
748-When common cause process variation is identified, the goal of
quality improvement is to:
A. Promote compliance with established procedure or protocol
B. Eliminate the variation
C. Improve practitioner competency
D. Reduce variation sufficiently to produce stability

749-Use the following list of values to answer questions 1- 5:

4 8 12

5 8 12

5 9 15

6 10 18

8 11 19

1. What is the total "N"?

a.11
b. 8
c.19
d.15

2. What is the range of values?


a. 11
b. 8
c. 19
d. 15

3. What is the arithmetic mean?


a. 11
b. 10
c. 9
d. 8

4. What is the mode?


a. 11
b. 10
c. 9
d. 8

144 of 268
5. What is the median?
a. 11
b. 10
c. 9
d. 8

750-You are setting up the data collection methodology for a study of


certain inpatient cardiac procedures, of the following, where might you
look for the most concise
e. Master patient index
f. Disease index
g. Surgical index
h. Surgical log

751-Management information systems are designed to be


a.Separate data source
b.Primarily manual
c.Used in decision making
d.Used primarily for long-term data storage

752-When comparing averaged immunization data from two pediatric


medical groups, it is appropriate to use
A.Standard deviation
B. A t-test
C.A chi-square test
D.Variance

752-The use of regression analysis to help determine relationships


between groups of numbers is most closely associated with which
graphic display technique?
a. Frequency distribution
d. Scatter diagram
c. Line graph
d. Histogram

753-The ability of a data measurement tool to produce the same results


over a period of time is known as:
a. Sensitivity
b. Specificity
c. Validity
d. Reliability

754-Every communication carries which two messages?


a. Surface and hidden
b. Truth and untruth
145 of 268
c. Mixed and contradictory
d. Content and relationship

755- The strategy for conflict resolution that emphasize facts and
finding an appropriate alternative solution, is called
a. Smoothing
b. Negation
c. Forcing
d. Discussion

756-According to Total Management principles, mangers must


a. Lead with autocratic decision making
b. Lead with participative decision making
c. Communicate successes or failures only to the boss
d. Concentrate on short-term financial impact of quality improvement
techniques

757-Which of the following concerns would be best solved by a QI


team?
a. A system issue
b. A discipline problem
c. A customer complaint
d. A financial variance

758-What is the best way to deal with conflict in a group?


a. A mandate
b. Assertiveness
c. Smoothing
d. Negotiation

759-Which of the following is a typical size for a customer focus group?


a. 1 to 3
b. 3 to 5
c. 8 to 12
d. 15 to 20

760-The success of a customer focus group rests principally on the:


a. Group size.
b. Length of meeting.
c. Opinions of the group.
d. Skill of the moderator.

761-Which of the following is an example of nominal data?


A. Pre-op, post-op.
B. AS, BS. MS, PhD.

146 of 268
C. Temperature.
D. Mass.

762 .Which of the following is an example of ordinal data?


e. Pre-op, post-op.
f. AS, BS. MS, PhD.
g. Temperature.
h. Mass.

763- Which of the following is an example of ratio data?


a. Pre-op, post-op.
b) AS, BS. MS, PhD.
c) Temperature.
d) Mass.

764-Which of the following is an example of interval data?


a. Pre-op, post-op.
b. AS, BS. MS, PhD.
c. temperature.
d. Mass.

765-Which one of the following is a synonym for count data?


a) Analog.
b) Continuous.
c) Discrete.
d) Variable.

766- be the least powerful statistically?


a) Interval.
b) Nominal.
c) Ordinal.
d) Ratio

767-The sampling of 5% or 30 cases, whichever is greater, is an example


of ____ sampling.
a) Convenience.
b) Expert.
c) Purposive.
d) Quota.

768-Comparing scores on one-half of a test with scores on the other half


provides ____ reliability.
a) Equivalence.
b) Internal.
c) Paired.
d) Test-retest.

769-The degree to which an instrument adequately measures the


universe of content is ____ validity:
e) Construct.
f) Criterion.
147 of 268
g) Critical.
h) Face.

770-Which type of validity uses the opinions of experts to determine


whether the instrument meets the standards required?
a) Construct.
b) Criterion.
c) Critical.
d) Face.

771-If the score on an instrument can be related to the behavior that the
instrument was supposed to predict then it possesses ____ validity.
a) Construct.
b) Criterion.
c) Critical.
d) Face.

772-The test used to analyze the difference between 2 means is the:


a) Chi-square.
b) Multiple regression analysis.
c) Regression analysis.
d) T-test.

773-If you want to predict IV site infiltration based on osmolarity of the


IV solution and the addition of potassium to the IV, you should use:
a) Chi-square.
b) Multiple regression analysis.
c) Regression analysis.
d) T-test.

774-Which test is often used by quality managers when data is counted,


not measured?
a) Chi-square.
b) Multiple regression analysis.
c) Regression analysis.
d) T-test.

775-Which of the following is the best confidence interval?


a) 99%
b) 95%
c) 90%
d) 85%

776-Which of the following is the best level of significance?


a) 0.01
b) 0.02
c) 0.05
d) 0.10

148 of 268
777-Historically, when the level of significance has been below ____
researchers have rejected the null and declared their results statistically
significant.
a) 0.01
b) 0.02
c) 0.05
d) 0.10

778-The Delphi technique is a form of ____ sampling:


a) Convenience.
b) Expert.
c) Purposive.
d) Quota.

779-A freeform generation of ideas best describes:


a) Brainstorming.
b) Delphi technique
c) Motivating.
d) Nominal group technique.

780-An arrow-type diagram or PERT chart is a type of:


a) Activity network diagram.
b) Deployment chart.
c) Pareto chart.

781-Organizational profitability is the difference between:


a) Reimbursement minus cost of service.
b) Reimbursement minus charges.

c) Fees minus cost.


d) Cost minus reimbursement.

782-To increase reliability of care process it is advisable to:


a.Design tasks around people 's professional skills
b. Design tasks to meet benchmarking. standards.
c. Design tasks to ensure a fair market share.
d. Design tasks to meet the organization goals.

783-Prospective analysis approaches detect limitations in:

a) Reporting process.
b) Designing process.
c) Implementing process.
d) Monitoring process.
784-Patient focused care deliver care services through:

a. Unit-based teams.
149 of 268
b. Specialized individuals.
c. Departments.
d. Independent practitioners.

785-The goal of an integrated service approach is to:

a. Reduce the cost of services


b. Increase the organization financial return
c. Involve top management, leaders, and department managers in the
process
d. Involve all working personnel in the process

786-Safety problems are regarded as:

a. Managerial problems
b. Clinical problems
c. Quality problems
d. Administrative problems

787-It is difficult to predict outcomes of care in:


a. Ambulatory care.
b. Hospice care.
c. Intensive care.
d. Emergency care.

788-Balance between cost, benefit and risk of under investigation,


constitutes a major problem in the management of:
a. Hospice care.
b. Home care.
c. Ambulator care.
d. Long-term care.

789-A patient was admitted to the chest out-patient clinic of St. Mark
hospital suffering from chronic lung insufficiency, to which level of care
should the patient be referred to be placed on a ventilator

a. Emergency care.

b. Intensive care.

c. Sub-acute care.

d. Long-term care.

150 of 268
790-Choosing the practitioner whether from inside or outside the
contracted network, is a privilege provided to customers of

a. HMOs.

b. PPOs.

c. Point of service.

d. Consumer directed health plan

791-A patient filed a law suit against a healthcare org. for removing his
spleen without his consent. The court stated a monetary compensation
for the patient. This action is considered:

a. Criminal liability.

b. Expanded liability.

c. Breach of duty.

d. Tort liability.

:Pre-authorization is required for all of the following except -792

.Inpatient acute care a.

.Ambulatory care b.

.Rehabilitation c.

.Psychiatric care d.

All of the following can be utilized as measures of utilization -793


:performance except

.Number of encounters in out-patient clinic a.

.ALOS in acute inpatient care b.

.Denials of service c.

.LIPs' appraisal d.

:Case management is a comprehensive process that involves -794

.Individual assessment a.

.Assessment of cohorts of similar patient groups b.


151 of 268
.Assessment of each DRG c.

.Assessment of the entire community d.

A patient was admitted to the surgical department for elective hip -795
replacement, following surgery, the patient suffered from diabetic
coma, and stated that he is diabetic since 5 years. The investigation
revealed that the surgeon never ordered a blood analysis for the patient
:prior to surgery, this is considered

.Negligence a.

.Breach of contract b.

.Incompetence c.

.Lack of professionalism d.

:Compliance with policies and procedures can be a useful tool for -796

.Risk prediction a.

.Risk prevention b.

.Risk avoidance c.

.Risk financing d.

:Evaluation of financial performance is based on -797

.Cost per unit of service a.

.Direct cost of service b.

.Indirect cost of service c.

.Reimbursement rate d.

:Patient safety is the responsibility of-798

.Top management a.

.Organization leaders b.

.All working personnel c.

.Patient safety officer / con bb b vvgfmmittee d.

:A safe environment can be best achieved by involving -799


152 of 268
Leaders and top management a.

Delegating the responsibility to a cross-functional team b.

Involving staff members organizationwide in the safety initiatives c.

Establishing a specified committee to review safety issues d.


organization-wide

800-Patient safety program must include all of the following, but the
most crucial is:
a. Identified individual or group to manage the program.
b. Defined mechanisms for support of staff responsible for the occurrence
of a sentinel event.
c. Proactive risk reduction activities.
d. Reporting mechanism.

surgery department reported that the review of nursing instrument -801


sheet reveals the absence of clamp, an x-ray done with negative results
and the patient is asymptomatic this is considered as

a. Potential compensable event


b. near miss
c. malpractice
d. nurse incompetence

The three models for case management are-802

A. Type of patient care, Focus of patient needs, and Professional discipline

B. Type of provider care, Focus of care, and Professional discipline

C. Type of patient care, Focus of provider, and Professional discipline

D. Type of provider care, Focus of patient needs, and Professional discipline

Answer is D
Professional discipline is common, and He has to cover patient and provider
for patient he will cover: needs For provider he will cover: type of
care

153 of 268
An Incremental Cost-Effectiveness Ratio is -803

.A. the equivalent ratio of mandatory costs and projected costs

.B. the result of a cost-utility analysis, but stated in ratio increments

.C. the same as a cost-effective analysis, but stated in ratio increments

D. the difference between a cost change and an outcome change

Cost analysis involves cost allocation, in which costs are -804


determined as direct or indirect. Direct costs are to indirect costs as

A. departmental budget is to the organization.s mission

B. the organization.s mission is to departmental budget

C. out-patient costs are to in-patient costs

D. in-patient costs are to out-patient costs

When developing a departmental budget, which one of the following -805


?need not be considered

A. Employee Satisfaction

B. Strategic Mission and Vision

C. Controlling costs

D. Developing Quantitative Plan Records

Data definition is necessary for performance improvement. When -806


measuring the frequency and type of medication error, you must first

.A. establish what is considered to be a medication error

.B. determine the need for measuring medication errors

.C. assemble a team familiar with those who make medication errors

.D. request data regarding medication errors committed recently

154 of 268
?What are the four primary types of events related to medical error -807

A. Near-error, Unusual Activity, Sentinel, Adverse

B. Near-error, Unusual Activity, Sentinel, Anticipated Outcome

C. Near-error, Unsafe Activity, Sentinel, Adverse

D. Near-error, Unsafe Activity, Sentinel, Anticipated Outcome

Risk Management assessment is a primary concern during -808

.A. day-to-day patient care activities

.B. evaluation of practitioners for credentialing and privileging

.C. invasive procedures because of the high-risk equipment

.D. process evaluation and process improvement

Answer: C

Cause you have to calculate the risk for such procedures it will be your main
concern

To develop the organization.s patient safety culture, the CPHQ will -809
not

.A. establish a commitment for allocation of funds, personnel and resources

.B. develop strategic plans that promote patient safety

.C. evaluate patient safety scenarios based on data-stream priorities

D. identify systemic processes that provide opportunities to improve patient


.safety

Answer B; Strategic plan is a preceding step

Facilitating the development of a patient safety program requires a -810


CPHQ to

.A. identify an interdisciplinary group to manage the safety program

.B. define the scope of the team and administrative oversight

.C. establish procedures for rapidly disseminating medical errors reports


155 of 268
D. outlining mechanisms to support the findings of sentinel event
.investigation

The most important components of a patient safety program are -811

A. links to external customers and suppliers based on outward data-


.structures

B. a functional infrastructure with a leader, safety officer, teams and tracking

.mechanisms

C. procedures for data collection that use data-streaming and vertical


.stratification

.D. reporting systems that allow for organizational transparency with all events

812-In the past, the Surgery Department at Sunshine Community


Hospital received a quarterly report with year-to-date information
concerning the hospitalwide attack rate of nosocomial infections, based
on 100% surveillance data from concurrent chart review. Now 100%
surveillance is no longer performed; the Surgery Department wants to
focus on the surgical site infection CDC definitions and wants targeted
studies performed for four procedures that relate to high-cost DRGs.
The infection control practitioner has similar requests from six other
departments or section (e.g., focus on pneumonia for Medicine and
Family Practice Departments, urinary tract infection for Urology Section,
etc.). She cannot do everything and is frustrated at their requests. In a
QI environment, her best solution is to:

a. argue for a return to 100% concurrent surveillance with appropriate staffing.


b. request QI teams to perform all targeted studies and feed the data to the
appropriate department.
c. request a QI team to prioritize the surveillance process and assure accurate
data collection.
d. send a memo to Utilization Management to do the studies through
concurrent review

813-At Sunshine Community Hospital, the quality professional is asked


to help the Respiratory Department establish indicators to measure their
performance in the treatment of ventilator-dependent patients. This
clinical condition has been identified by organization leaders as a
156 of 268
Strategic Quality Initiative and representatives from all appropriate
departments are on the chartered QI team. The Respiratory Department
currently views this "study" as added work and a "cost issue," not a part
of their departmental "quality management plan." Use this information
to answer Questions 2 through 5:

2- The quality professional, acting as facilitator, meets with the


Respiratory Department QI Task-Team and identifies as the team's first
clinical task:

a. describing the scope of the problem and possible reasons.


b. identifying all current Respiratory Department indicators and criteria.
c. defining the Respiratory Department scope of service and ventilator care
process.
d. reviewing all data collected in past monitoring of ventilator-dependent
patients.

3- Involving all appropriate departments/services in organizationwide


"Strategic Quality Initiatives" is consistent with which aspect of the
performance improvement function?

a. top-level involvement.
b. collaboration.
c. prioritization.
d. competency review.

4- What can the quality professional do to best facilitate "buy-in" on the


part of the Respiratory Department?

a. restate the mandate of the leaders to involve all appropriate departments.


b. offer to do all the data collection and initial analysis.
c. provide background data/information concerning the selection of the
initiative.
d. provide all available cost data on ventilator dependent patients, with
breakdown by department.

5- Once the Respiratory Department has gathered and aggregated their


data, the Department 01 Task-Team should :

157 of 268
a. provide only summary findings; all data collected remains confidential to the
department.
b. provide information only to the Medicine Department of the medical staff, to
whom the Respiratory Department reports quarterly.
c. provide the ventilator data and all ongoing monitoring activity data related to
oxygen use.
d. provide the ventilator data and initial findings to the quality professional and
the QI team.

814-A bundled payment, as described in the Accountable Care Act, for


an inpatient hospitalization would include:

A. All costs for hospital-based service


B. All costs for ancillary services while hospitalized
C. Costs incurred for 30 days after hospitalization
D. All of the above

815-Which of the following is a likely process measure for a patient with


diabetes mellitus?

A. Average value of HbA1C testing


B. hospitalization rates
C. percentage of patients developing foot ulcers
D. rates of foot examination?

816-healthcare organization is in a region with population that has a


high affection with G6PD deficiency in their children. As screening is
expensive, the organization screen samples only, but the organization
found that the treatment of undetected cases is more costly than
screening of whole population. The organization should:

A) Continue to screen samples


B) Screen all the population.
C) Stop screening
D) Increase the size of sample

817-Attempts to align financial incentives of purchasers,payers


&providers with provider performance on clinical process &outcome
meadures encourages ......

a.under utilization
b.community backlash
158 of 268
c.overutilization
d.reengineering..

818-the long-run objective of finanical management is to :

a. maximize the value of hospital common stock .


b. maximize market share.
c. maximize return on investement .
d. maximize earnings per share.

819-Comparing organization expenses with patient days is a measure


of:

a, Organization performance.
b. Staff accountability.
c. Compliance of the organization with benchmarking standards.
d. Productivity.

820- Written structure, process and outcome criteria against which


actual patient chart are screened to assess the adequacy of intervention
efforts is known as:

A. audit
B. critical pathways
C. diagnosis related groups
D. utilization review

821-A primary purpose of an information management system is to


allow an organization to

A . save time .
B . centralize demographics .
C . reduce cost .
D . evaluate data

822-Which of the following monitors provides patient outcome


information?

A . healthcare-acquired infection rate


B . nursing care documentation compliance
C . antibiotic therapy discontinuation compliance
D . equipment malfunction rate

159 of 268
823-As the Director of Quality at Hospital X, you have been appointed to
lead a team to improve patient flow through the hospital system.
At your first team meeting, some people expressed their excitement
over the new project while others were unsure of their rôles and
responsibilities. After several meetings, team members disagreed on
various issues, sometimes leading to heated debates. Cliques began to
form within the group, and some members resisted taking on more
tasks. Collective decisions were difficult to make. Over the next few
weeks, the team gradually began to respect your authority as the team
leader. As team members knew one another better, they began to work
more closely and socialize together. It is evident that the team has
developed a stronger commitment to the team goal. Meaningful
progress is finally being made but your participation is still required.
What is the term commonly used to describe the current stage of team
development?

a. Norming
b. Performing
c. Forming
d. Storming

824-A quality professional needs to assign a staff member to assist a


medical director in the development of a quality program for a newly
established service . Which of the following staff members is MOST
appropriate for this project?

A . a newly hired staff member who has demonstrated competence and has
time to complete the task
B . a knowledgeable staff member who works best on defined tasks
C . a motivated staff member who is actively seeking promotion
D . a competent staff member who has good interpersonal skill

825-Discharge planners regularly monitor the number of inappropriate


referrals, the timeliness of discharge planning, and the number
of days of discharge delays . What additional monitor should be added
to evaluate the appropriateness of discharge planning
interventions?

A . adequacy of documentation in progress notes


B . attainment of discharge planning goals
C . timeliness of referrals to discharge planning
D . number of discharge planning referrals from nursing

160 of 268
826-A well-designed patient safety program should include all of the
following EXCEPT

A . an annual patient safety committee meeting .


B . planned response to adverse events .
C . orientation and continuing education on patient safety issues .
D . review of patient safety policies and procedures for all departments

827-A healthcare organization must have a Risk Management plan to


obtain liabilityinsurance. Which of the following lists is best for a Risk
Management plan?

A. Statement of Purpose, Goals, Program Scope, Procedures, Design


Mechanisms
B. Statement of Purpose, Goals, Design Mechanisms, Policies, Data Sources
C. Statement of Purpose, Goals, Program Scope, Policies, Data Sources and
Evaluation
D. Statement of Purpose, Goals, Team Member Evaluations, Procedures,
Data Sources

828-When an expert consultant is hired to assist with the Performance


Improvementproject, the CPHQ should

A. allow the consultant to establish the necessary goals for the project.
B. ask the consultant for an itemized job description that s/he will follow.
C. defer to the consultant regarding time frames and deadlines.
D. provide the consultant with an organizational chart indicating lines of
authority.

829-The champions of the Performance Improvement Team

A. monitor of all of the team’s day-to-day activities.


B. review the team’s overall efforts and provide guidance and direction.
C. counsel, but are not accountable for the team’s efforts.
D. support, but require the team to keep all interested parties informed.

830-To facilitate change within an organization, a CPHQ should

A. obtain reference information from other organizations.


B. maintain every aspect of the plan as long as it is in place.

161 of 268
C. participate in developing and improving a common vision of care.
D. keep the plan for each department confidential from the others.

831-Which one of the following is not an issue that must be resolved


when developing aPerformance Improvement Plan?

A. Assign key leaders at all levels of responsibility within the organization


B. Determine consistent terminology for documenting planned activities
C. Allocate personnel to the Quality Council to oversee activities required in
the plan
D. Create an Oversight Group of the Governing Board to supervise the
Quality Council

832-Your freestanding Radiology Center did 200 outpatient CT scans


each of the last two years. The average reimbursement rate has been
decreased from 200$ to 100$. The scanner and room need repairs
estimated at 100.000$. there are two other CT scanners in your
immediate vicinity. The most likely decision resulting from a cost-benefit
analysis would be to:

.A. Quit doing CT scans


B. Repair the scanner
C. Contract with a competitor for referral fees
D. Market heavily and postpone the repairs for 6 months

833-managed competition in healthcare most often refers to:

a.healthcare providers administered by competing management companies.


b. healthcare providers competing by Diagnosis Related Group (DRG).
c. grouped healthcare providers competing within a geographic region.
d. managed care organizations.

834-The Quality Management cycle, based on Juran's Quality Trilogy


(quality planning, quality control, quality improvement)

a. excludes the lab's activities to monitor equipment.


b. requires a departmentalized approach to quality management.
c. encompasses only the nonclinical aspects of OM.
d. incorporates information from strategic planning.

835-The most basic components of managed care include all except

162 of 268
a. prepaid financing.
b. comprehensive services at multiple levels and settings.
c. controlled access to services.
d. broad choice of providers.

836-In managed care, the most common form of mimbursement for


primary care physicians is
a. straight salary.
b. capitation without withholds.
c. capitation with withholds.
d. discounted fee-for-service.

837-A hospital utilization management plan generally includes provision


for:
A. Disaster planning
B. Transition planning
C. Quality planning
D. Financial planning

838-It is important to identify customer-supplier relationships to improve


methods, to meet customers’ needs, and to increase internal awareness.
Which of the following is the best example of a horizontal internal
customer-supplier relationship?

A. Administrators-Board of Directors
B. Administrators-Nursing Executives
C. Directors of Nursing-Charge Nurses
D. Charge Nurses-Registered Nurses with BSNs

839-After defining "internal" and "external" customers, your


organization is making a master list of each type of customer before
initiating a major change process. Of the following, which is the best
next question to ask of staff?

a. Who do you receive services from?


b. Who in your work day do you serve?
c. Which patients receive your services?
d. How do you know a customer from a supplier?

163 of 268
840-Being immediately responsive and attentive to a family’s concerns
following a patient’s fall in the subacute care facility is:

A. Loss reduction activity


B. Loss prevention activity
C. Risk shifting activity
D. Risk avoidance activity

841-In your organization, Quality management (QM) and Risk


Management (RM) are separate departments. As QM Director, you
recognize the importance of linking with RM to prevent or reduce risk
and maximize patient safety. Use this information to answer questions 1
&2

1- How can Quality Management link with Risk Management on peer


review cases

A. Provide information about peer review actions


B. Provide information about patient occurrences
C. Provide aggregate occurrence data
D. Meet with RM Director regularly in confidence

2-Of the following, sharing which data supports risk prevention?

A. Annual practitioner profiling


B. Monthly event/occurrence reporting
C. Root cause analysis
D. Failure mode and effects analysis

842-even with good process,errors may occur,,knowing this ..leaders


should depend on..:

a-strict policy
b-performance feedback
c-customer survey

843-most appropriate educational programme during merger:

a-rapid cycle
b-organizational change

164 of 268
844-In the transition from quality assurance to quality
management/quality improvement, which of the following emphases has
resulted in the most significant benefit?

a. Focusing primarily on process rather than individual performance


b. Focusing on organizationwide rather than clinical processes
c. Organizing activities around patient flow rather than department or
discipline
d. Initiating more prospective rather than retrospective improvement efforts

845-strategic planning is:

a-mission,vision,values,goals,objectives
b-master plan,customer,quality initiatives

846-Reasons for using pre-certification as a prospective review method


include all of the following, except:

A. It enables the UM staff at the MCO to perform a pre-admission review on


the case
B. It enables the MCO to divert the case to the provider with the lowest
prices.
C. It enables the MCO to capture data for more accurate estimation of
financial accruals rather than waiting for claims to come in.
D. The MCO uses the pre-certification process to verify the member's
eligibility for coverage

BBB.....MCO concern with effectiveness & efficiency of the care delivered......


A...UM review..promote effeciency & medical necessity & this is important
C.... estimate financial accurals...promote risk management & this is important
D...member eligibility to coverage ....also promote efficiency
while..BB...lowest price of provider ..not by itself a goal
So, my answer is BBBB

847-standard of care is:

a-based on what an ordinary prudent person of like training and expertise


would do for a specific condition
b-an expression of the ideal care that the pt needs and expects to receive for
a specific condition
c-based on locale where the individual receives care
d-not acceptable in a court of law for a malpractice case

165 of 268
848- patients are key customer in PI..of the following what is the most
accurate way to measure patient perception of care after completion of
treatment:

a-log and analyze expressed pt outcome


b-collect data on returns to emergency departement,revisits to primary
careand re admission to acute care
c-utilize pt satisfaction surveys,sampling each quarter
d-utilize pt health out come questionaires for specific illnesses

849-The Organizational Culture comprises the attitudes, beliefs, and


behaviors of thoseinvolved in the organization. The four basic types of
Organizational Cultures areStable-Learning, Group, Independent, and

A. Dependent.
B. Secure.
C. Insecure.
D. Environmental

850-Timely intervention is MOST likely possible with a ____ review


system.

A. concurrent B. focused
C. prospective D. retrospective

851- hoshin planning most valuable step is

A. Roll down of strategies to departmental level


B. Annual reporting
C. Use of PDCA
D. Prioritization of goals

852-In the "language" of strategic planning, the "strategies" of the


organization can also be called

a. objectives.
b. critical success factors.
c. goals.
d. the dashboard.

166 of 268
853-The capability of the indicator to describe the feature of interest,
expresses its:

a. Credibility.
b. Reliability.
c. Validity.
d. Clarity.

854-An organization asks a Q professional to help in preparedness to a


survey of accreditation body,the Q manager will first

a.assign a team for the project


b.arrange for a mock survey
c.educate the staff about the Q that they may be asked
d.review the adherernce of org to accreditation standards

855-of the following the most effective quality -based management of


human resources involves

a.hiring,training,appraising,firing
b.hiring,training,appraising,promoting
c.hiring,training,appraising,employee relations
d.hiring ,retaining,promoting ,appraising

856-healthcare quality programe has prepared a balanced score card


that displayed pt satisfaction was 98%..financial targets has been
met,medication error increased by 30 %..and heart surgeries decreased
by 3%..what addidtional information should the GB ask for..:

a-type of medication error


b-heart surgery case
c-pt satisfaction data
d-review pt complaints

857- in a health care organization ,CEO requested the most effective


recommendations to aseess the organization readiness,,which of the
following methods should the healthcare Q proffesional recommend
first..:

167 of 268
a-administer survey to evaluate org culture
b-review of performance up result
c-contract a Q.consultant to conduct review
d-walk through the org

858-a descriptive measure derived from a population is known as:

a-parameter
b-statitsic
c-data
d-precision

859-provision of safty services must be consistent with :

a_patient perespective
b_practitioner credentialing
c_best practice
d_organization initiatives

860-The hospital management asked you, as a social worker, to conduct


a survey that measure caregiver attitudes about six patient safety-
related domains, to compare themselves with other organizations, to
promote interventions to improve safety attitudes, and to measure the
effectiveness of these interventions. What these domain scales?

A. Teamwork climate, job satisfaction, perceptions on risk management,


safety climate, optimal personal factors, and stress recognition.
B. Safe, timely, effective, efficient, equitable, and patient-centered.
C. Leadership competencies, culture of work, shared leadership, perceptions
on management, policies, and external partnerships.
D. Effective communication, creative people, high-alert leaders, external
partnerships, timely effective care, and punitive reaction.

861-In one of the first class high quality hospitals in Newcastle, Ministry
of Health yield a survey about what is the impact of safe culture of work
on the caregivers? As a quality man, What is not expected to find in the
final report of the survey?

A. The staff did not worry that their mistakes will be reported in their personnel
file.
B. The staff belief that their weakness points will not be used against them.
C. The highest percentage of staff reported that no significant adverse events
168 of 268
had occurred in their setting during the past 12 months.
D. Low average composite score involved questions related to nonpunitive
response to error.

862-In an integrated delivery system, the success of the quality strategy


is most dependent on the effectiveness of the

a. information system.
b. Ql team process.
c. case management process.
d. patient care management system.

863-Clinical decision support systems can support medication safety by


alerting prescribers to

A. patient compliance and allergies.


B. the need for dose adjustments and patient weight changes.
C. drug interactions and patient weight changes.
D. allergies and drug interactions.

864 - A key physician /licensed independent practitioner QM function is


a.Researching criteria options for specialty- specific peer review
b.Determination of what constitutes a deviation from accepted standard of
care
c.Determination of data collection methodology for non-physician clinical
reviewers
d.Tabulation of peer review data for periodic committee reporting

865-Which of the following is NOT requirement for an organizationwide


QM program?
a. Quality management activities include the use of performance measures
in peer review activities.
b. Peer review problems are resolved and opportunities for improvement are
taken
c. Reports to the governing body include the findings from peer review
activities
d. The effectiveness of the program, including peer review is evaluated

866-The Wellness Medical and Healthcare Center uses a multi-level


medical record review system to monitor clinical care that cannot be
evaluated through their electronic stat systems. Nurses, other clinical
staff, health information management staff, and physicians participate.
physicians usually do all except:
a. Review /confirm variations in trend data
169 of 268
b. Review selected cases to confirm noncompliance with criteria
c. Provide oversight monitoring of non-physician clinical reviewers
d. Screen cases for peer review

867-What of the following is the greatest benefit of concurrent clinical


review?
a. Ability to focus review on prioritized performance measures
b. Ability to review outcomes of care and process
c. Timely assessment at the onset of care for continuity
d. Timely intervention to reduce risk of adverse outcomes

868-In conjunction with hospital credentialing, clinical privileges are


granted
a. Only to members of the medical/professional staff
b. To all employees performing clinical procedures
c. To all licensed independent practitioners
d. Only to active members of the medical/professional staff

869-In the large healthy Community Medical Group, one general surgeon
has an 8% rate for both superficial and deep incisional surgical site
infections for cases performed from October through March , 60 %
higher than the average for the other general surgeons in the group . in
conjunction with the medical director, what should the quality
professional do next?
a. Compare with local and national average in infection rates
b. Determine the surgeon risk-adjusted case mix and practice patterns
c. Compare with the rates of general surgeons in other surgical groups
d. Take cases to the peer review body

870-Operative/ other procedure review is the responsibility of


a. The medical staff
b. The organization leaders
c. The Quality council
d. Those providing the care and service

871-Most commonly the primary purpose for incident/occurrence


reporting is to
a. Record infection rates
b. Identify medication errors
c. Identify adverse patient events
d. Identify patient grievances

872-Failure mode and effects analyses (FMEA) is what type of review of


improvement tool?
a. Concurrent
b. Focused
c. Prospective
170 of 268
d. Retrospective

873-Which of the following four types of processes listed that are


associated with review of medication use, blood/blood product use, or
operative and other procedures use (hospital or ambulatory care), which
would most likely fall under the purview of utilization management?
a. Indications/appropriateness
b. Preparation/dispensing
c. Administration/ performance
d. Monitoring effects

874-The appropriateness of care is:


a) Primarily a focus of utilization management.
b) A key dimension of quality care.
c) Equivalent to case management.
d) The degree to which healthcare services are coherent and
unbroken.

875-Total quality management philosophy assumes that:


a. Most problems with services delivery result from systems
difficulties.
b. Frequent inspection is necessary to improve quality.
c. Most problems with services delivery result from difficulties with
individuals.
d. Top management leadership in quality activities disenfranchises
employees.

876-The major difference between traditional “quality assurance”


activities and the expanded quality improvement/performance
improvement activities is the QI/PI focus on:
e) People and competency
f) Analysis of data
g) Performance measures
h) Systems and processes

877-Organizational “culture” most often refers to:


a. The ethnicity of the organization’s employees and licensed
independent practitioners
b. Assumptions about people and how work gets done
c. The efforts to reach the diverse groups in the community
d. The scheduled social and cultural events within the organization

878-Applying the Pareto Principle in quality improvement is:


a. Prioritizing process issues
b. Tracking and measuring process effectiveness
c. Providing meaningful data to support strategic objectives
d. Prioritizing patient outcome issues

171 of 268
879-A hospital generally has a unique structure comprised of a
“triangle” which three entities make up the triangle?
a. Governing body, administration, finance
b. Administration, department managers, medical staff
c. Governing body, administration/management, medical staff
d. Administration, medical staff, nursing

880-The best evidence of the incorporation of quality planning into the


organization wide strategic planning process might be:
a. Successful quality initiatives
b. The organization wide plan for provision of patient care
c. The quality management/quality improvement plan
d. The quality management/quality improvement budget

881-Strategic planning is best described as:


a. A long-term focus, projecting the present into the future
b. A set of top-level performance measures
c. A statement of mission, vision, and values
d. An ongoing look into the future

882-In an organization wide QI model, the person or group usually


accountable for continuously assessing and improving performance at
the department level is the
a. Cross-functional QI team
b. Quality council
c. Department director
d. Department team

883-A large emergency department (ED) reduced its average length of


stay for discharged patients from 130 minutes to 1 hour with a goal to
improve patient satisfaction. How best might ED know the changes were
also effective financially, as part of a cost-benefit analysis?
a. Decreased staffing and decreased costs
b. Increased staffing and increased net revenue
c. Increased patient volume and increased net revenue
d. Increased patient volume and increased staffing

884-A quality professional in a home health care agency is charged to


develop a quality management/quality improvement strategy. Of the
following steps, which should be done first?
a. Develop strategic quality initiatives
b. Determine the roles of leaders in implementation
c. Draft the QM/QI plan for review by leaders
d. Review the organization’s scope of care and service

885-Which of the following is not relevant to include in both utilization


management and quality management plans?
a. Confidentiality policy
172 of 268
b. Process for appealing treatment denials
c. Conflict of interest policy
d. Provision for annual program evaluation

886-An 85-year-woman is admitted through the emergency department


with a fractured right hip. When should discharge planning begin?
a. After surgery, once the physical therapist has done an
assessment
b. When the physician writes a discharge planning order
c. At time of admission to the acute hospital
d. When the decision is made concerning the next level of care

887-Negligence means a lack of proper care, in medical malpractice


“proper care” is determined by:
a. JCAHO standards
b. Jury of civilian peers
c. Tort law
d. Medical peers

888-The utilization management committee for a large medical group is


concerned about underutilization. Which data supports the concern?
a. Lab report delays
b. Reduced pediatric hospitalization rates
c. Increased incidence of C-Sections
d. Reduced pediatric immunizations rates

889-The term “performance” as used in healthcare quality improvement


activities, refers to:
a. The effective execution of functions and processes
b. An interactive series of process steps
c. A statement of expectations
d. A demonstration during accreditation survey

890- The most effective way to ensure patient safety as a dimension of


performance is to:
a) Sponsor a “hotline” for reporting problems
b) Focus on processes and minimize individual blame
c) Have leaders who commit to and foster a safe culture
d) Encourage patients and families to identify risks

2. 891- The responsibility to reduce risks of endemic and epidemic


nosocomial infection is vested in:
a) The organization
b) An interdisciplinary committee
c) A qualified infection control practitioner
d) The attending physician

173 of 268
892 In any quality management approach, how can best evaluate the
effectiveness of action taken?
a) Use the same performance measures to re-monitor the process
b) Formulate a new special study to monitor the action
c) Interview the staff involved in implementing the action plan
d) Do nothing. Effectiveness is expected with well-planned action

893- An orthopedic surgeon in a surgical group refuses to accept high


postoperative site infection rate for joint cases over the last year. What
could the QM professional try next to convince him?
a) Present the data to all the orthopedic surgeons using
practitioner names
b) Do nothing with the surgeon, continue to measure
c) Have peers outside the group review all the surgeon’s cases
d) With the medical director, show the surgeon the data compared
to peers

894-Root cause analysis is the most the most appropriate PI process


for:
a) Determining costs/benefits
b) Evaluating dental care
c) Analysis sentinel events
d) Performing peer review

All the questions in the quiz along with their answers are shown below. Your
answers are bolded. The correct answers have a green tick while the incorrect
ones have a red cross.

Question 1 of 105
895- The benefits of studying a process include all of the following
EXCEPT

A. arriving at a common understanding.


B. eliminating errors.
C. eliminating inconsistencies.
D. highlighting obvious problems.
Question was not answered.

Answer: B
Benefits of studying a process include the following:

174 of 268
 Arriving at a common understanding. When Team Members work
through the recommended strategies, they gain a common understanding of
the process, start using the same terminology, and don't waste time pulling in
different directions.
 Eliminating inconsistencies. When comparing how various people
associated with the process carry out their work, inconsistencies will surface.
Many of these can be traced to a lack of documentation and inadequate
training about the best way to run the process. Quality and productivity often
increase dramatically once employees who do the same job start sharing and
using a "best-known way" to do their work.
 Highlighting obvious problems. Looking closely at a process almost
always highlights glaring problems that have gone unnoticed but can be easily
fixed. This is particularly true of administrative processes.
Studying a process does not directly lead to the elimination of errors.

Content Category: Performance Measurement and Improvement


Cognitive level required for a response: Recall
Tasks on the CPHQ exam content outline to which the question is
linked: Coordinate or participate in quality improvement projects

Question 2 of 105
896-The second sponge count at the end of a hernia repair operation on
an obese patient was incorrect. This was confirmed by repeat sponge
counts, and the surgeon eventually located and retrieved the missing
sponge. The patient's recovery was uneventful.

The healthcare professional should

A. conduct root cause analysis.


B. perform failure mode and effects analysis.
C. continue to monitor incident reports of inaccurate sponge counts from
the Operating Room.
D. recommend retraining of Operating Room staff to better track surgical
instruments and sponges during surgical procedures.
Question was not answered.

Answer: A
This question consists of two parts. Firstly, to answer the question correctly,
you need to recognize that the incident in the Operating Room is a sentinel
event. As discussed in my article on sentinel events, this incident meets the
definition of a sentinel event because there was a significant risk that the

175 of 268
sponge (a foreign body) was retained in the patient. Secondly, you are
expected to know that any sentinel event requires a thorough and credible
root cause analysis.
Content Category: Patient Safety
Cognitive level required for a response: Analysis
Tasks on the CPHQ exam content outline to which the question is
linked: Perform or coordinate risk management

Question 3 of 105
897- A point prevalence survey in 2010 showed that the overall
prevalence proportion of healthcare-associated infections in a hospital
system was 7.3%. In 1990, the prevalence proportion was 8.1%. A
hypothesis test for the difference between the two prevalence
proportions gave a P-value of 0.029.

Which of the following is the most accurate interpretation of the results?

A. The prevalence of healthcare-associated infections in 2010 is


significantly lower than that in 1990.
B. The prevalence of healthcare-associated infections in 2010 is lower
than that in 1990, but there is a small chance that, in reality, there was no
difference in the prevalences.
C. The prevalence of healthcare-associated infections in 2010 is higher
than that in 1990 but there is a moderate chance that, in reality, there was no
difference in the prevalences.
D. The prevalence of healthcare-associated infections in 2010 is slightly
lower than that in 1990.
Question was not answered.

Answer: B
The prevalence of healthcare-associated infections is lower in 2010 than in
1990. Therefore, the third answer option (C ) should be eliminated. The final
answer option (D) is ambiguous -the term "slight" carries no meaning in
statistics or in healthcare quality. Some candidates might have chosen the
first answer option because the P-value was lower than 0.05. However, the
level of significance for the hypothesis test was not stated. It could have been
0.01 (in which case it would not have been a "significant result") or 0.10 (in
which case it would have been significant). The most accurate interpretation
of the result is this: the prevalence of healthcare-associated infections in 2010
is lower than that in 1990, but there is a small (2.9% probability) that the result

176 of 268
would have been observed, due to chance, if there was in truth no difference
between the two prevalences. The correct answer for this question is B.
Content Category: Information Management
Cognitive level required for a response: Application
Tasks on the CPHQ exam content outline to which the question is
linked: Use the results of statistical techniques to evaluate data (e.g. t-test,
regression)

Question 4 of 105
898- Which of the following is most effective in communicating
instructions to patients before their discharge from hospital?

A. In addition to clear verbal instructions, the caregiver conveys all


instructions in written form.
B. The caregiver gives verbal instructions on more than one occasion
before the patient's discharge from hospital.
C. The caregiver communicates the same instructions to a member of the
patient's family.
D. The caregiver communicates instructions to the patient and then asks
the patient to explain what he/she has just been informed to do.
Question was not answered.

Answer: D
Each of the four communication strategies above could be applied. However,
"teach back" (answer option D) is probably the most effective technique to
communicate instructions to patients. According to the Agency for Healthcare
Research and Quality (2001) report Making Health Care Safer, "Asking that
patients recall and restate what they have been told" is one of 11 top patient
safety practices based on strength of scientific evidence.
Content Category: Performance Measurement and Improvement
Cognitive level required for a response: Application
Tasks on the CPHQ exam content outline to which the question is
linked: Facilitate evaluation or selection of evidence-based practice
guidelines (e.g. for standing orders or as guidelines for physician ordering
practice)

Question 5 of 105
899- Among the following factors, competency assessment of staff is
LEAST influenced by data related to

A. productivity.

177 of 268
B. feedback from patients, families, and staff.
C. performance improvement findings relative to performance standards
in the job description.
D. knowledge of administrative policies and procedures.
Question was not answered.

Answer: D
The first three answer options support staff competency assessment. Whilst
knowledge of administrative policies and procedures may be required for
some jobs, the other factors are stronger in evaluating an employee's
competency and skill gaps.

Content Category: Performance Measurement and Improvement


Cognitive level required for a response: Recall
Tasks on the CPHQ exam content outline to which the question is
linked: Incorporate performance improvement into the employee performance
appraisal system

Question 6 of 105
900-The senior leaders of a managed care organization have consulted a
healthcare quality professional on the purchase of a clinical data
management software system to support performance improvement.

Which of the following is the most important issue in identifying the


system requirements?

A. Users' need for customized graphs and tables.


B. The number of existing computer terminals.
C. The organization's goals for the data management system.
D. Integration with existing information systems.
Question was not answered.

Answer: C
All the answer choices play a part in identifying the system requirements but
the primary issue, at least at the beginning of the software selection process,
is identifying the goals and objectives of the new data management system.

Content Category: Information Management


Cognitive level required for a response: Application
Tasks on the CPHQ exam content outline to which the question is
linked: Assist with the evaluation of computer software applications
178 of 268
Question 7 of 105

901-A team in a healthcare facility is working on a project to improve


access to primary care. The average length of time to the "third next
available" appointment was chosen as an outcome measure. Which of
the following is the most appropriate balancing measure?

A. Number of new patient visits.


B. Percentage of patients satisfied with phone access.
C. Individual panel size.
D. Office visit cycle time.
Question was not answered.

Answer: B
For this project, the number of new patient visits and individual panel size (i.e.
number of unique patients for which a physician is responsible) are process
measures. Office visit cycle time, i.e. the length of time that a patient spends
at an office visit, is possibly another outcome measure for this project.

A "balancing measure" is one used to make sure that any changes introduced
during the project to improve one part of the system are not causing new
problems in other parts of the same system. In this case, by reducing the time
to the "third next available" appointment, staff could possibly be stretched too
thin and their response to telephone calls may become less satisfactory.

As another example, if you were involved in a project to prevent venous


thromboembolism (clots) after pelvic surgery, you might select, as balancing
measures, the incidence of bleeding episodes and heparin-induced
thrombocytopenia (low platelet count).

Content Category: Performance Measurement and Improvement


Cognitive level required for a response: Application
Tasks on the CPHQ exam content outline to which the question is
linked: Facilitate development or selection of process and outcome measures

Question 8 of 105

179 of 268
902-A licensed independent practitioner with admitting privilege at a
hospital must

A. be a fully licensed physician.


B. be eligible for medical staff membership.
C. be a member of the medical staff.
D. have completed proctoring for any clinical privileges previously
requested.
Question was not answered.

Answer: C
A licensed independent practitioner with admitting privilege MUST be a
member of the medical staff. In some organizations, the medical staff may
include professionals other than medical doctors (MD, MBBS, DO, etc.).
Examples of the latter are dentists, clinical psychologists, and podiatrists.
Therefore, the first answer option (A) is not the best answer. Eligibility for
medical staff membership implies that the individual has not yet been
appointed. Proctoring is usually required for a predefined period after the
initial appointment. In many organizations, completion of proctoring is not
required for admitting privilege.

Content Category: Performance Measurement and Improvement


Cognitive level required for a response: Recall
Tasks on the CPHQ exam content outline to which the question is
linked: Participate in the credentialing and privileging process

Question 9 of 105
903- In your capacity as the Director of Quality and Patient Safety at a
1600-bed tertiary referral center, you are consulted to assist in the
development of a balanced scorecard.

In selecting measures for the scorecard, the "perspectives" commonly used


include all of the following EXCEPT

A. Financial.
B. External Business Processes.
C. Customer.
D. Learning and Growth.
Question was not answered.

Answer: B
180 of 268
The 4 classic "perspectives" of the balanced score card are: "Financial",
"Customer", "Internal Business Processes", and "Learning and Growth". See
this article on the Balanced Scorecard Framework for more information.
Content Category: Management and Leadership
Cognitive level required for a response: Application
Tasks on the CPHQ exam content outline to which the question is
linked: Develop and use performance measures (e.g. balanced scorecards,
dashboards, core measures)

Question 10 of 105
904-To assess their job satisfaction, 32 nurses on a Med-Surg ward were
given a self-administered questionnaire. One of the items on the
questionnaire was a self-rating of job satisfaction on a Likert scale (1 =
Very Dissatisfied, 10 = Very Satisfied).

What is the healthcare quality professional's interpretation of the results from


the graph above?

A. There is a strong linear positive association between job satisfaction


and years of employment.
B. There is a moderate linear positive association between job satisfaction
and years of employment.
C. There is a strong linear negative association between job satisfaction
and years of employment.
D. There is a moderate linear negative association between job
satisfaction and years of employment.
Question was not answered.

Answer: D

181 of 268
The scatter plot (one of the 7 Basic Tools of Quality) describes the association
between two variables. This is a summary of scatter plots, which also explains
the correct answer.
Content Category: Information Management
Cognitive level required for a response: Application
Tasks on the CPHQ exam content outline to which the question is
linked: Use basic statistical techniques to describe data (e.g. mean, standard
deviation)

Question 11 of 105
905-The number of productive hours worked by nursing staff with direct
patient care responsibilities per patient day is a

A. structural measure.
B. process measure.
C. outcome measure.
D. composite measure.
Question was not answered.

Answer: A
Structural measures reflect the conditions in which providers care for patients.
Process measures show the degree to which evidence-based steps in care
processes are followed. Outcome measures look at the results of care.
Composite measures combine the result of multiple performance measures to
provide a more comprehensive picture of quality care. You might like to read
our article on assessing the quality of care.
Content Category: Performance Measurement and Improvement
Cognitive level required for a response: Application
Tasks on the CPHQ exam content outline to which the question is
linked: Facilitate development or selection of process and outcome measures

Question 12 of 105
906-In your capacity as the Director of Quality at an 800-bed
multidisciplinary hospital, you are consulted on how best to reduce
complication rates while reducing length of stay and cutting overall
costs for total hip replacement.

Provided none of the following has already been attempted, the best
option is

182 of 268
A. to identify the causes for the unacceptable complication rates, high
lengths of stay and rising costs related to total hip replacement.
B. a clinical pathway.
C. to analyse the data and confirm that total hip replacement is associated
with unreasonable rates of complications, and higher-than-expected lengths
of stay and costs.
D. to review, and revise if necessary, all existing policies and procedures
for total hip replacement.
Question was not answered.

Answer: B
There is no indication that the complication rates, length or stay or overall
costs are unacceptable or above the average. Therefore, answer options A
and C are not appropriate. Between the other two choices, B is the better
answer. Merely reviewing policies and procedures rarely improves the
outcomes considered in this question. See our article on clinical pathways for
an explanation.
Content Category: Performance Measurement and Improvement
Cognitive level required for a response: Application
Tasks on the CPHQ exam content outline to which the question is
linked: Participate in the development of clinical/critical pathways or
guidelines

Question 13 of 105
907-The senior leaders of a hospital are prioritizing performance
improvement initiatives for the coming year.

Which of the following tools will be most useful for this purpose?

A. Pareto chart
B. Cause-and-effect diagram
C. Affinity diagram
D. Stratification
Answer: A
The most useful tool would likely be a Pareto chart. A cause-and-effect
diagram is useful for identifying possible causes of a problem. An affinity
diagram is useful for brainstorming and therefore not relevant in this
situation. Stratification is unlikely to be appropriate in this scenario.
Content Category: Performance Measurement and Improvement
Cognitive level required for a response: Application

183 of 268
Tasks on the CPHQ exam content outline to which the question is
linked: Facilitate establishment of priorities for process improvement activities

Question 14 of 105
908-Which of the following graphs is most appropriate in displaying the
root causes of adverse events that have occurred in a hospital system
over the past 10 years?

A. Histogram
B. Frequency polygon
C. Line chart
D. Bar chart
Question was not answered.

Answer: D
Root causes are qualitative data. Therefore, a correct way to summarize them
is by using a bar chart. Another acceptable way of displaying the data would
be via a pie chart (but a bar chart is preferable). A line chart, histogram or
frequency polygon may be used for quantitative data. See our article
ongraphing frequency distributions for more details. Page 46 of Improving
America’s Hospitals: The Joint Commission’s Annual Report on Quality and
Safety 2007 illustrates how bar charts can effectively display data on root
causes of sentinel events.
Content Category: Information Management
Cognitive level required for a response: Application
Tasks on the CPHQ exam content outline to which the question is
linked: Compile and write performance improvement reports

Question 15 of 105
909-A series of poor surgical outcomes at a small community hospital
led to an investigation which eventually found that the vast majority of
recent failed surgeries were conducted by only one surgeon. The
"surgeon" was subsequently discovered to have forged his medical
qualifications and had been impersonating a doctor for the previous 8
months. He is currently awaiting trial on charges in connection with the
surgeries he performed at the hospital.

The primary role of the healthcare quality professional in this case is

184 of 268
A. to assist in the criminal prosecution of the alleged perpetrator.
B. to identify other licensed independent practitioners whose qualifications
had been forged.
C. to facilitate a review of the credentialing and privileging process.
D. to review policies and procedures related to surgical procedures so that
patient safety may be improved.
Question was not answered.

Answer: C
The healthcare quality professional's primary role in such a case is to help
improve processes. A credentials committee is usually responsible for
reviewing the qualifications of medical staff applicants/members, not the
healthcare quality professional. A review of policies and procedures related to
surgery is unlikely to prevent unqualified practitioners from operating in the
future.

Content Category: Patient Safety


Cognitive level required for a response: Analysis
Tasks on the CPHQ exam content outline to which the question is
linked: Integrate patient safety concepts within the organization

Question 16 of 105
910-The chart below shows the overall inpatient mortality at a hospital.

Based on the data, the healthcare quality professional should

A. report that overall inpatient mortality has improved and the


improvement is statistically significant.
B. conclude that inpatient mortality has increased overall.
C. conduct drill-down analysis.
185 of 268
D. continue to monitor inpatient mortality.
Question was not answered.

Answer: D
As explained in our article on using run charts, identification of special cause
variation - both good and bad - follows the criteria listed. In this case, there is
no clear run/shift, trend or pattern in the run chart. In the absence of evidence
of special cause variation, it is best to continue tracking the measure. The
actions in the other answer options are not appropriate without any further
information. However, if the next data point is below the median line, I would
call it a run/shift and therefore try to understand why this has occurred. It is
possible that something significant occurred around or before January 2010 to
account for the start of this run, e.g. change of management, introduction of
strategic improvement initiatives, etc.
Content Category: Information Management
Cognitive level required for a response: Application
Tasks on the CPHQ exam content outline to which the question is
linked: Use or coordinate the use of statistical process control components
(e.g. common and special cause variation, random variation, trend analysis)

Question 17 of 105
911-The senior leaders of a hospital system were interested to learn
whether inpatient deaths were associated with after hours admission.
Therefore, a random sample of patients who had died in hospital and a
representative group of patients who did not die were selected.
Subsequently, the times of their hospital admission were analysed.

Which of the following measures is most appropriate to determine


whether there is an association between inpatient mortality and after
hours admission?

A. Rate ratio
B. Odds ratio
C. Risk ratio
D. Risk incidence
Question was not answered.

Answer: B
This is an example of a case-control study. As discussed in our article
on overview of study design, we start by identifying individual cases of the
outcome of interest. We then identify a representative group of individuals

186 of 268
who do not have the outcome. These individuals act as controls. We then
compare cases and controls to assess whether there were any differences in
their past exposure to one or more possible risk factors. The most appropriate
measure of effect for case-control studies is the odds ratio.
Content Category: Information Management
Cognitive level required for a response: Application
Tasks on the CPHQ exam content outline to which the question is
linked: Use epidemiological theory in data collection and analysis

Question 18 of 105
912-Which of the following sampling methods should a healthcare
quality professional use to obtain the most precise estimate of the
prevalence of pressure ulcers in a 900-bed multi-disciplinary tertiary
care facility?

A. Simple random sampling


B. Systematic sampling
C. Stratified simple random sampling
D. Multi-stage sampling
Question was not answered.

Answer: C
For this heterogeneous hospital population, the most precise estimate of
pressure ulcer prevalence will be obtained by stratified simple random
sampling (unless the sample is very large). See our article on complex
sampling methods for more information.
Content Category: Information Management
Cognitive level required for a response: Application
Tasks on the CPHQ exam content outline to which the question is
linked: Use epidemiological theory in data collection and analysis

Question 19 of 105
913-In examining the association between inpatient mortality and after
hours admission, the healthcare quality professional was interested to
find out whether the distance between patients' place of residence and
the hospital might be a confounding variable.

Which of the following methods can she use to determine whether the
association between inpatient mortality and after hours admission was
confounded?

187 of 268
A. ANOVA
B. Stratification
C. Chi-squared test
D. Student's t-test
Question was not answered.

Answer: B
This article on confounding explains the use of stratification to control for
confounding.
Content Category: Information Management
Cognitive level required for a response: Application
Tasks on the CPHQ exam content outline to which the question is
linked: Use epidemiological theory in data collection and analysis

Question 20 of 105
914-One of the aims in the treatment of severe community-acquired
pneumonia is to maintain an oxygen saturation of >94% (or 88 - 92% in
patients with chronic obstructive airway disease). Ensuring adequate
oxygenation for this condition is a

A. process and outcome measure.


B. structure measure.
C. process measure.
D. outcome measure.
Question was not answered.

Answer: C
Please refer to our article on structure, process and outcomes for an
explanation of the answer.
Content Category: Performance Measurement and Improvement
Cognitive level required for a response: Application
Tasks on the CPHQ exam content outline to which the question is
linked: Facilitate development or selection of process and outcome measures

Question 21 of 105
915-Senior leaders of an organization can promote quality by

A. executive walkarounds.
B. micromanagement to demonstrate a hands-on approach.
C. encouraging staff to set their own expectations so that they can meet
them.

188 of 268
D. focusing on the financial position, reputation and lay management of
the hospital.
Question was not answered.

Answer: A
Senior leaders can improve quality, especially patient safety, by executive
walkarounds. Micromanagement is generally frowned upon. Leaders should
set expectations for staff. Traditionally, senior leaders focused on finance,
reputation and lay (non-clinical) management but modern quality
management requires their active participation.

Content Category: Management and Leadership


Cognitive level required for a response: Application
Tasks on the CPHQ exam content outline to which the question is
linked: Facilitate assessment and development of the organization’s quality
culture

Question 22 of 105
916-The chart below shows the rate of Cesarean Sections in a hospital.

The healthcare quality professional should

A. continue monitoring the monthly rates of Cesarean Sections.


B. recommend a Cesarean Section audit by peer review.
C. review the policies and procedures for Cesarean Section.
D. review the antenatal care of women who had Cesarean Sections.
Question was not answered.

Answer: B

189 of 268
There is a rise in the rate of Cesarean Sections and the final 7 consecutive
data points form a run/shift. A rise in the Cesarean Section rate is not
desirable and should be investigated. The healthcare quality professional is
not responsible for determining whether the procedure was medically
indicated - this is done by peer review. Reviewing policies and procedures
alone rarely helps in situations like this. Antenatal care may be a contributing
factor - this is evaluated during peer review.

Content Category: Patient Safety


Cognitive level required for a response: Analysis
Tasks on the CPHQ exam content outline to which the question is
linked: Interpret outcome data

Question 23 of 105
917-A randomized controlled trial was conducted to assess the
effectiveness of a multimedia patient education program designed to
help prevent falls in a hospital. The following results were obtained:

What is the rate ratio?

A. 2.3
B. 1.3
C. 0.77
D. -2.3
Answer: C
The rate ratio is the ratio of the incidence rate in the exposed (intervention)
population to the incidence rate in the unexposed (control) population. In this
case, the rate ratio = 7.5/9.8 = 0.77. See our article on difference and ratio
measures for more information.
Content Category: Information Management
Cognitive level required for a response: Application
Tasks on the CPHQ exam content outline to which the question is
linked: Aggregate/summarize data for analysis

Question 24 of 105
918-In participative management,

190 of 268
A. the leader attempts to sell his/her decisions to the group.
B. the leader makes decisions and announces them to the group with
minimal participation from group members.
C. few decisions are made.
D. the leader presents a tentative decision, elicits suggestions from
group members, and then makes the final decision.
Answer: D
See this brief description on participatory management style for an
explanation of the answer.
Content Category: Management and Leadership
Cognitive level required for a response: Recall
Tasks on the CPHQ exam content outline to which the question is
linked: Facilitate change within the organization

Question 25 of 105
919-The Process Management area of the Baldrige Health Care Criteria
for Performance Excellence addresses each of the following EXCEPT

A. healthcare processes.
B. support processes.
C. operational planning.
D. strategic planning.
Question was not answered.

Answer: D
The Process Management category includes healthcare processes, support
processes and operational planning, but not specifically strategic planning
(though the latter is somewhat related to the first three areas).

Content Category: Performance Measurement and Improvement


Cognitive level required for a response: Recall
Tasks on the CPHQ exam content outline to which the question is
linked: Aid in evaluating the feasibility to apply for external quality awards
(e.g. Malcolm Baldrige, Magnet)

Question 26 of 105
920-A healthcare quality professional examined the relationship
between the rate of adverse patient occurrences and duration of overall
medical practice among physicians in a hospital. In the analysis, the
computed value of r was 0.8139.
The healthcare quality professional concluded that there is a

191 of 268
A. moderate positive relationship between the rate of adverse patient
occurrences and duration of medical practice.
B. strong positive relationship between the rate of adverse patient
occurrences and duration of medical practice.
C. moderate negative relationship between the rate of adverse patient
occurrences and duration of medical practice.
D. strong negative relationship between the rate of adverse patient
occurrences and duration of medical practice.
Question was not answered.

Answer: B
The letter r stands for the correlation coefficient. A value of 0.8 or above is
generally considered strong and a positive value indicates a positive
relationship. See our article on scatter plots and correlation for more details.
Content Category: Information Management
Cognitive level required for a response: Application
Tasks on the CPHQ exam content outline to which the question is
linked: Use the results of statistical techniques to evaluate data (e.g. t-test,
regression)

Question 27 of 105
921-Clinical decision support software is an example of

A. external memory.
B. a server.
C. artificial intelligence.
D. mass storage.
Question was not answered.

Answer: C
You should be familiar with some common terms used in computerized
systems as they are necessary for evaluating such systems and using them.
See this brief description of Clinical Decision Support.
Content Category: Patient Safety
Cognitive level required for a response: Application
Tasks on the CPHQ exam content outline to which the question is
linked: Evaluate computerized systems for data collection and analysis

Question 28 of 105
921-The main purpose of performance improvement in healthcare is to

192 of 268
A. improve patient outcomes.
B. enhance patient and family satisfaction.
C. improve processes of care.
D. meet accreditation standards.
Question was not answered.

Answer: A
A general definition of performance improvement is "a planned, systematic,
organizationwide approach to the monitoring, analysis, and improvement of
organization performance, thereby continually improving the quality of patient
care and services provided and the likelihood of desired patient outcomes."
Content Category: Management and Leadership
Cognitive level required for a response: Recall
Tasks on the CPHQ exam content outline to which the question is
linked: Develop a performance improvement plan

Question 29 of 105
922-For a patient with insulin-dependent diabetes mellitus, which of the
following programs is the most appropriate to administer?

A. Disease management
B. Utilization management
C. Demand management
D. Risk management
Question was not answered.

Answer: A
Disease management is defined as "a system of coordinated health care
interventions and communications for populations with conditions in which
patient self-care efforts are significant. It is the process of reducing healthcare
costs and/or improving quality of life for individuals by preventing or
minimizing the effects of a disease, usually a chronic condition, through
integrative care."

Utilization management is concerned with "the planning, organizing, directing,


and controlling of the healthcare product in a cost-effective manner while
maintaining quality of patient care and contributing to the overall goals of the
institution."

193 of 268
Demand management involves the use of decision and behaviour support
systems to appropriately influence individual patients' decisions about
whether, when, where, and how to access medical services.

Risk management is a formal attempt to control liability, prevent financial loss,


and protect the financial assets of the organization. Note that in the CPHQ
Exam Candidate Handbook's "Terminology Crosswalk of Terms", case
management = case/care/disease management.

Content Category: Management and Leadership


Cognitive level required for a response: Application
Tasks on the CPHQ exam content outline to which the question is
linked: Contribute to development and revision of a written plan for a
case/care/disease/utilization management program

Question 30 of 105
923-Impressed by what he saw at a healthcare conference, the Chief
Executive Officer decided to adopt Lean Six Sigma as the hospital's new
approach to process improvement.

If the desired results are not achieved, which of the following is the most
likely reason for this?

A. Lack of understanding of Lean Six Sigma.


B. Lack of top management support.
C. Projects not linked to organizational goals and objectives.
D. Inadequate focus on behavioural change to support process change.
Question was not answered.

Answer: B
All of the above contribute to failure of quality improvement initiatives.
However, the commonest problem is lack of senior management support. It's
common for a senior member of staff liking the idea of introducing something
they had heard about but not understanding what it takes to achieve success.
This often leads to disillusionment later and subsequent failure of projects. All
improvement requires investment in resources - time, people, skills
development, money, etc. - and leaders need to be aware of this before
committing to new methods.

Content Category: Management and Leadership


Cognitive level required for a response: Application
194 of 268
Tasks on the CPHQ exam content outline to which the question is
linked: Facilitate development of leadership values and commitment

Question 31 of 105
924-The prevalence of pressure ulcers in an acute-care facility is 18.3%.
The healthcare quality professional obtained information that the
prevalence of pressure ulcers across 420 acute care hospitals in the
state was 14.8%. For the purpose of improvement in her hospital, the
latter figure (14.8%) is best regarded as

A. a benchmark.
B. a goal.
C. a comparison.
D. an estimate.
Question was not answered.

Answer: C
The figure of 14.8% is an "average" of the measure across the hospitals. It's
not a "benchmark", which relates to best practice. The figure may be a "goal"
and is certainly an "estimate" but the BEST description is a "comparison".

Content Category: Information Management


Cognitive level required for a response: Application
Tasks on the CPHQ exam content outline to which the question is
linked: Use comparative data to measure or analyze performance

Question 32 of 105
925-The Body Mass Index (a measure of body fat) was measured in a
group of women attending a primary care clinic. The graph below
summarizes the results.

195 of 268
Which of the following measures best summarizes the data?

A. Mean
B. Mode
C. Median
D. Range
Question was not answered.

Answer: C
The relative frequency distribution is skewed (to the right). The median is the
most appropriate summary measure for skewed distributions. Read our article
on measures of location for more details.
Content Category: Information Management
Cognitive level required for a response: Application
Tasks on the CPHQ exam content outline to which the question is
linked: Use basic statistical techniques to describe data (e.g. mean, standard
deviation)

Question 33 of 105
926-Which of the following is NOT a key concept of Total Quality
Management?

A. Waste elimination
B. Customer focus
C. Process centered
D. Continual improvement
Question was not answered.
196 of 268
Answer: A
See this resource on Total Quality Management for more information.
Content Category: Performance Measurement and Improvement
Cognitive level required for a response: Recall
Tasks on the CPHQ exam content outline to which the question is
linked: Develop organizational performance improvement training (e.g.
quality, patient safety)

Question 34 of 105
927-Which of the following is the first step in implementing lean
management effectively in a hospital?

A. Create an organizational culture that is receptive to lean thinking.


B. Distinguish between value-added steps from non-value-added steps in
any given process.
C. Eliminate waste.
D. Identify key processes for kaizen projects.
Question was not answered.

Answer: A
The first step in successful lean implementation in an organization is senior
leadership creating an organizational culture that is receptive to lean thinking.

Content Category: Management and Leadership


Cognitive level required for a response: Application
Tasks on the CPHQ exam content outline to which the question is
linked: Facilitate assessment and development of the organization’s quality
culture

Question 35 of 105
928-When a healthcare quality professional plotted data on patients' age
in a frequency histogram, she found a negatively skewed distribution.
Therefore,

A. the median is greater than the mean and the mode is greater than the
median.
B. the mean is greater than the median and the median is greater than the
mode.
C. the mode is greater than the mean and the median is greater than the
mode.

197 of 268
D. the mean is greater than the mode and the mode is greater than the
median.
Question was not answered.

Answer: A
For a negatively skewed distribution, the mode is greater than the median,
which is, in turn, greater than the mean. Below are illustrations of the relative
values of the mean, median and mode for both negatively skewed and
positively skewed distributions.

Content Category: Information Management


Cognitive level required for a response: Application

198 of 268
Tasks on the CPHQ exam content outline to which the question is
linked: Use basic statistical techniques to describe data (e.g. mean, standard
deviation)

Question 36 of 105
929-Which of the following is the most likely cause of medication errors
in an acute care facility?

A. Illegible physician handwriting.


B. Systems failure.
C. Careless nurses.
D. Look-alike, sound-alike drugs.
Question was not answered.

Answer: B
Medication errors, like other patient safety incidents, are thought to be due to
systems failure. James Reason's Swiss cheese model of system accidents is
discussed in the following article:
http://www.bmj.com/content/320/7237/768.full
The other answer options give factors that contribute to medication errors but
"systems failure" is the best answer.

Content Category: Patient Safety


Cognitive level required for a response: Application
Tasks on the CPHQ exam content outline to which the question is
linked: Integrate patient safety concepts within the organization

Question 37 of 105
930-Which of the following tools is most useful in identifying ways to
shorten nurses' walking time from one activity to another in a hospital
ward?

A. Pareto chart
B. Ishikawa diagram
C. Spaghetti diagram
D. Control chart
Question was not answered.

Answer: C
A Pareto chart is used to identify the most frequent or impactful problems or
causes of problems.

199 of 268
An Ishikawa diagram, or cause-and-effect diagram, is a tool for identifying and
organizing the possible causes of a problem.

"The spaghetti diagram is a tool to help you establish the optimum layout for a
department or ward based on observations of the distances travelled by
patients, staff or products, e.g. x-rays. Spaghetti diagrams expose inefficient
layouts and identify large distances travelled between key steps."

A control chart is a time plot that indicates the rage of variation built into the
system.

Of these four tools, a spaghetti diagram would be the most useful in this case.
The spaghetti diagram is often used with other tools and techniques, e.g.
process mapping. Below is an example of a spaghetti diagram.

Content Category: Information Management


Cognitive level required for a response: Application
Tasks on the CPHQ exam content outline to which the question is
linked: Use or coordinate the use of process analysis tools to display data
(e.g. fishbone, Pareto chart, run chart, scattergram, control chart)

Question 38 of 105
931-A healthcare quality professional can best display the distribution of
48 data points on waiting times in an ambulatory care clinic using a

200 of 268
A. stem-and-leaf plot.
B. bar chart.
C. scatter diagram.
D. run chart.
Question was not answered.

Answer: A
A stem-and-leaf plot, or stem-plot, is a tool for presenting quantitative data in
a graphical format. Like a histogram, it provides information on the shape of a
distribution. A bar chart, scatter diagram, and run chart are not appropriate in
presenting the distribution of data points.

Content Category: Information Management


Cognitive level required for a response: Application
Tasks on the CPHQ exam content outline to which the question is
linked: Aggregate/summarize data for analysis

Question 40 of 105
932-A 37 year old woman who underwent mechanical aortic valve
replacement was discharged on warfarin (coumadin). The target
International Normalized Ratio (INR) was 3.0–3.5. She developed a
severe pneumonia two weeks later and was readmitted to hospital, at
which time her INR was found to be 8.3.

Upon receiving a report on the elevated INR, the healthcare quality


professional should first

A. inform the reporting staff member that cases of elevated INR levels
need not be reported.
B. conduct root cause analysis.
C. check if the patient received predischarge medication counseling.
D. continue monitoring reports of elevated INR levels.
Answer: C
This patient's elevated INR was most likely related to her pneumonia or the
medications used to treat it before her readmission. However, a high INR is a
potentially fatal and preventable condition - it should be reported. Root cause
analysis (RCA) is not appropriate in this case - conducting RCA on all cases
of elevated INR levels would not make the best use of available resources. It
is probably useful to determine whether the patient received the necessary
counseling about her discharge medications, including warfarin. This could
uncover opportunities for improvement in the predischarge medication

201 of 268
counseling process. Data on elevated INR levels should be collected and
aggregated. However, between answer options C and D, C is the better
answer.

Content Category: Patient Safety


Cognitive level required for a response: Application
Tasks on the CPHQ exam content outline to which the question is
linked: Perform or coordinate risk management

Question 41 of 105
933-Common causes of process variation refer to causes of variation
that

A. are one-time occurrences.


B. occur over long periods of time and that are persistent.
C. are extrinsic to the process.
D. are more easily identified.
Question was not answered.

Answer: B
"Common causes" are chronic and persistent. Answer options A, C, and D are
features of "special causes".

Content Category: Information Management


Cognitive level required for a response: Recall
Tasks on the CPHQ exam content outline to which the question is
linked: Use or coordinate the use of statistical process control components
(e.g. common and special cause variation, random variation, trend analysis)

Question 42 of 105
934-Cross-sectional studies

A. can be used to study causation.


B. measure incidence rate.
C. measure incidence risk.
D. can be conducted using a questionnaire survey.
Question was not answered.

Answer: D
For more information about cross-sectional studies, read our article on study
design.
202 of 268
Content Category: Information Management
Cognitive level required for a response: Recall
Tasks on the CPHQ exam content outline to which the question is
linked: Use epidemiological theory in data collection and analysis

Question 43 of 105
935-Concerning the surgical "time-out", which of the following
statements is FALSE?

A. The surgical "time-out" reduces the risk of wrong-site surgery.


B. The surgical "time-out" reduces the risk of preventable surgical
mistakes other than wrong-site surgery.
C. The surgical "time-out" is a component of the World Health
Organization (WHO) Safe Surgery Checklist.
D. The surgical "time-out" requires involvement of the patient.
Question was not answered.

Answer: D
A "time-out" just before starting a surgical procedure, to ensure the correct
patient, procedure and body part, is part of The Joint Commission's Universal
Protocol and is a requirement for one of JCI's International Patient Safety
Goals (Eliminate Wrong-site, Wrong-patient, Wrong-procedure Surgery). The
surgical "time-out" reduces the risk of wrong-site surgery and other
preventable surgical mistakes, and is a component of the WHO Safe Surgery
Checklist. The "time-out" should involve the entire operative team, but not
necessarily the patient (who may be under general anaesthesia when "time-
out" is performed").
Content Category: Patient Safety
Cognitive level required for a response: Recall
Tasks on the CPHQ exam content outline to which the question is
linked: Integrate patient safety goals into organizational activities (e.g. Joint
Commission, JCI, NQF, IHI)

Question 44 of 105
936-Leaders of a multi-hospital system are trying to prioritize the
services to introduce in the coming year based on their impact on the
community. These leaders, who work geographically apart, can arrive at
a group consensus without meeting face to face by

A. the nominal group technique.


B. the Delphi technique.

203 of 268
C. brainstorming.
D. a focus group.
Question was not answered.

Answer: B
Among the four answer options (in their original iterations), only the Delphi
technique does not require face-to-face meetings.

Content Category: Performance Measurement & Improvement


Cognitive level required for a response: Application
Tasks on the CPHQ exam content outline to which the question is
linked: Lead performance improvement teams

Question 45 of 105
937-Following a serious adverse event, a hospital decided to pursue a
negotiated settlement. Which of the following would most likely apply in
this situation?

A. Tort liability
B. Contributory negligence
C. Contractual liability
D. Res ipsa loquitur
Question was not answered.

Answer: C
Tort liability is a sort of insurance coverage that takes effect when a court
determines that damage (in some form or other) was caused by negligence
on the part of the defendant. In this case, tort liability does not apply as it was
an out-of-court settlement, i.e. it did not involve the courts. There is no
information in the question that suggests contributory negligence. Contractual
liability for negligent treatment is likely to be applicable in this case - when a
patient is admitted for care, he/she and the hospital enter into a contractual
relationship. Res ipsa loquitor is not relevant in this case.
Content Category: Performance Measurement and Improvement
Cognitive level required for a response: Application
Tasks on the CPHQ exam content outline to which the question is
linked: Perform or coordinate risk management

Question 46 of 105

204 of 268
938-A hospital purchases additional malpractice insurance and general
tort liability insurance prior to introducing a pediatric heart surgery
program. This is an example of

A. risk transfer.
B. risk avoidance.
C. risk reduction.
D. risk retention.
Question was not answered.

Answer: A
Risk transfer, or risk sharing, means transferring the burden of loss (or the
benefit of gain) from a risk to another party (which, in this case, is an insurer).
Risk avoidance means not conducting the activity that carries the risk. Risk
reduction involves reducing the severity of the loss and/or the likelihood of the
loss occurring. Risk retention means accepting the loss, or benefit of gain,
from a risk when it occurs.

Content Category: Performance Measurement and Improvement


Cognitive level required for a response: Application
Tasks on the CPHQ exam content outline to which the question is
linked: Perform or coordinate risk management

Question 47 of 105
939-The Chief Executive Officer of an acute care facility wishes to know
the difference between Total Quality Management (TQM) and Six Sigma.

The healthcare quality professional should inform him that

A. TQM can be implemented on its own while the benefits of Six Sigma
can only be realized when it is combined with Lean methods.
B. TQM loosely monitors progress toward goals whereas Six Sigma
ensures that investment in quality produces the expected return.
C. TQM focuses on compliance with performance standards. Six Sigma
focuses on world class performance.
D. TQM is a management philosophy whereas Six Sigma is a tool to
reduce variation in a product or process.
Question was not answered.

Answer: D
Only answer option D provides a true statement.
205 of 268
Six Sigma has been used successfully without Lean methods by numerous
organizations.

TQM does not "loosely" monitor progress toward goals.

One of the basic principles of TQM is continuous improvement. Contrary to


popular belief, "compliance" is not a feature of TQM.

Content Category: Management and Leadership


Cognitive level required for a response: Recall
Tasks on the CPHQ exam content outline to which the question is
linked: Evaluate applicability of performance improvement models (e.g.
FOCUS, PDCA, Six Sigma)

Question 48 of 105
940-The following table shows the risk-adjusted Acute Myocardial
Infarction (AMI) inpatient mortality for 10 different hospitals in 2009:

What is the benchmark risk-adjusted AMI inpatient mortality?

A. 4.7%
B. 8.2%
C. 8.5%
D. 10.4%
Question was not answered.

Answer: A
Benchmarking is based on identifying best practices. In this case, the
benchmark is the lowest AMI inpatient mortality rate, i.e. 4.7%.

Content Category: Information Management


Cognitive level required for a response: Application
Tasks on the CPHQ exam content outline to which the question is
linked: Interpret benchmarking data

Question 49 of 105

206 of 268
941-A hospitalized patient died shortly after being administered with
medication intended for another patient. Which of the following tools is
most appropriate for facilitating root cause analysis?

A. Barrier analysis
B. Prioritization matrix
C. Pie chart
D. Pareto chart
Question was not answered.

Answer: A
Root cause analysis following an adverse event is commonly facilitated by a
fishbone diagram, also known as a cause-and-effect diagram or Ishikawa
diagram. However, this is not one of the answer options.

Barrier analysis is suitable for use in root cause analysis.


A prioritization matrix is often used by teams to get consensus about an issue.

Pie charts display categorical data.

Pareto charts help to identify the most frequent or impactful problems or


causes of problems but are not generally employed during root cause
analysis.
Content Category: Patient Safety
Cognitive level required for a response: Application
Tasks on the CPHQ exam content outline to which the question is
linked: Perform or coordinate risk management—root cause analysis

Question 50 of 105
942-In the second half of 2008, the inpatient fall rate at Hospital X was
above 15 falls per 1000 patient-days. A multidisciplinary team
commenced an initiative to lower the rate of inpatient falls in February
2009. The historical average in the 10 years before 2008 was 6.6 falls per
1000 patient-days and the target for this initiative was 5.0 falls per 1000
patient-days. The results of this improvement work are shown in the
graph below.

207 of 268
Which of the following is the most appropriate next step?

A. Stop monitoring patient falls.


B. Continue monitoring patient falls.
C. Continue the initiative to reduce the rate of patient falls further.
D. Lower the target.
Question was not answered.

Answer: B
Patient falls are reportable events in a hospital and should therefore be
tracked by the event reporting system. Patient falls in Hospital X should
continue to be monitored - this is the best answer. It is unlikely that the
improvement team can reduce the inpatient fall rate any further, having
achieved improvement beyond the target and well below the historical
average. Lowering the target, likewise, is unlikely to be beneficial.

Content Category: Performance Measurement and Improvement


Cognitive level required for a response: Application
Tasks on the CPHQ exam content outline to which the question is
linked: Integrate results of data analysis into the performance improvement
process

Question 51 of 105
943-The dimension of quality/performance that is addressed by
introducing a rapid response team in a hospital is

A. continuity of care.
B. efficiency.

208 of 268
C. effectiveness.
D. prevention and early detection.
Question was not answered.

Answer: D
A rapid response team, also known as a medical emergency team, "is a team
of clinicians who bring critical care expertise to the bedside". Their primary
purpose is to identify unstable patients and those patients likely to suffer
cardiac or respiratory arrest, and prevent their unnecessary deaths.

Continuity of care refers to the degree to which healthcare is delivered in a


coordinated and seamless manner from one service to another.

Efficiency refers to the amount of resources required to achieve a desired


result.

Effectiveness is "the degree to which a desired outcome is reached".

Prevention and early detection means the degree to which interventions


promote health and prevent disease. The most appropriate answer is D.

Content Category: Performance Measurement and Improvement


Cognitive level required for a response: Application
Tasks on the CPHQ exam content outline to which the question is
linked: Perform or coordinate risk management: risk prevention

Question 52 of 105
944-In analyzing data, the healthcare quality professional can minimize
the risk of interpreting noise as if it were a signal AND minimize the risk
of failing to detect a signal when it is present by using a

A. run chart.
B. control chart.
C. specifying a target.
D. comparing data to average values.
Question was not answered.

Answer: B
Both the run chart and control chart describe or define the "voice of the
process", whereas specifications (answer options C and D) define the "voice
of the customer".
209 of 268
The distinction between signals and noise is the foundation for every
meaningful analysis of data. It also defines two mistakes which can be made
when attempting to analyze data. The first mistake is that of interpreting
random variation as a meaningful departure from the past—interpreting noise
as if it were a signal. The second mistake consists of not recognizing when a
change has occurred in a process—failing to detect a signal when it is
present. This mistake is most often found in conjunction with the specification
approach to analysis. The underlying process changes, but the values are still
within the specification limits, so no one notices.

Clearly, one may avoid the first mistake by never reacting to any value as if it
were a signal, but this would guarantee many mistakes of the second kind.
Likewise, one may avoid the second type of mistake by reacting to every point
as if it were a signal, but this guarantees many mistakes of the first kind.

The control chart approach strikes a balance between the two errors. The use
of control limits to filter out the noise will minimize the occurrence of both
types of errors.

Content Category: Information Management


Cognitive level required for a response: Application
Tasks on the CPHQ exam content outline to which the question is
linked: Use or coordinate the use of statistical process control components
(e.g. common and special cause variation, random variation, trend analysis)

Question 53 of 105
945-From a quality perspective, which of the following is the BEST way
to control costs in healthcare?

A. Identifying and eliminating waste.


B. Organization-wide budget cuts.
C. Temporary hiring freeze.
D. Improving customer satisfaction.
Question was not answered.

Answer: A
Past approaches to controlling costs in healthcare have included across-the-
board budget cuts, hiring/salary freezes, and even laying off of staff. However,
from a quality perspective, the best way to cut costs is by identifying and
eliminating muda (waste), of which there is plenty. Improving customer
satisfaction will not directly lead to reduced costs.

210 of 268
Content Category: Management and Leadership
Cognitive level required for a response: Application
Tasks on the CPHQ exam content outline to which the question is
linked: Demonstrate financial benefits of a quality program

Question 54 of 105
946-The odds ratio of an outcome

A. is the ratio of the number of cases to the number of non-cases in a


defined population and time period.
B. provides an approximate estimate of the risk ratio when the outcome of
interest is rare.
C. is used in case-control studies only.
D. compares the number of cases in a population exposed to a suspected
risk factor with the number of cases in a population not exposed.
Question was not answered.

Answer: B
The odds ratio of an outcome compares the odds of having a particular
outcome (not the number of cases) in a population exposed to a suspected
risk factor with the odds in a population not exposed. When the outcome of
interest is rare, the odds ratio is approximately equal to the risk ratio (the "rare
disease assumption"). The odds ratio is also used in cross-sectional studies.

Content Category: Information Management


Cognitive level required for a response: Recall
Tasks on the CPHQ exam content outline to which the question is
linked: Aggregate/summarize data for analysis

Question 55 of 105
947-Which of the following tools is most appropriate for investigating
the relationship between two characteristics?

A. Scatter plot
B. Cause-and-effect diagram
C. Failure modes and effects analysis
D. Pareto chart
Question was not answered.

Answer: A

211 of 268
Of the four answer options, a scatter diagram is the most appropriate tool to
evaluate the relationship between two variables. Cause-and-effect diagrams
help to identify and organize the possible causes of a problem in a structured
format. Failure modes and effects and analysis is used to anticipate possible
process or product failures and the risks associated with these failures. A
Pareto chart is used to identify the most frequent or impactful problems or
causes of problems.

Content Category: Information Management


Cognitive level required for a response: Application
Tasks on the CPHQ exam content outline to which the question is
linked: Use or coordinate the use of process analysis tools to display data
(e.g. fishbone, Pareto chart, run chart, scattergram, control chart)

Question 56 of 105
948- Updated guidelines to prevent falls in the elderly state that patients
with no evidence or history of gait problems or recurrent falls do not
require a fall risk assessment. In this case, asking older patients if they
have fallen recently or if their gait is unsteady is a form of

A. surveillance.
B. screening.
C. diagnosis.
D. monitoring.
Question was not answered.

Answer: B
Surveillance, like monitoring, refers to an ongoing activity to detect changes in
trends or distribution of a disease or behaviours leading to an increased risk
of a disease/outcome, with the aim of facilitating disease/outcome control.
The activity in this case is not ongoing (for any particular individual) and
therefore cannot be called surveillance or monitoring.

Screening is a method used to detect a disease/condition in individuals


without overt signs or symptoms of that disease or condition. In this case, the
condition is an increased risk of falls.

Diagnosis is the "identification of the nature and cause of anything". In this


case, the "diagnosis" of increased falls risk has not yet been made; otherwise
there would no longer be a need for further fall risk assessment.

212 of 268
Content Category: Performance Measurement and Improvement
Cognitive level required for a response: Application
Tasks on the CPHQ exam content outline to which the question is
linked: Perform or coordinate risk management: risk prevention

Question 57 of 105
949-In an improvement project to reduce the wait times in an Emergency
Room, the time taken to be assessed by a physician is

A. a process measure.
B. an outcome measure.
C. a structure measure.
D. not a suitable measure.
Question was not answered.

Answer: A
In reducing wait times in an Emergency Room, one of the key steps is to
reduce the door-to-physician time. It is, therefore, a process measure. It is not
an outcome measure because physician assessment is only one of the steps
in the process that will lead to the final outcome - discharge from the
Emergency Room (to a bed in the hospital, home or other facility).

Content Category: Performance Measurement and Improvement


Cognitive level required for a response: Application
Tasks on the CPHQ exam content outline to which the question is
linked: Facilitate development or selection of process and outcome measures

Question 58 of 105
950-Overall responsibility for an improvement project lies with the

A. Facilitator.
B. Sponsor.
C. Team Leader.
D. Team Members.
Question was not answered.

Answer: B
The Sponsor/Champion maintains overall responsibility, authority, and
accountability for an improvement project.

213 of 268
Content Category: Performance Measurement and Improvement
Cognitive level required for a response: Application
Tasks on the CPHQ exam content outline to which the question is
linked: Identify champions (e.g. process owners, quality, patient safety)

Question 59 of 105
951-In improvement work, a constraint is

A. a factor that will prevent the team from achieving its goal.
B. a factor that can be easily changed.
C. a factor that will limit the options the team can realistically consider.
D. a factor that is not easily changed but changing it will help the team's
progress.
Question was not answered.

Answer: C
A constraint is an unchangeable factor that will limit the options the team can
realistically consider. Examples of real constraints are:
 Available budget
 Written or unwritten rules (sacred cows or taboos)
 Present technical ability of Team Members and other involved parties
 Factions, rivalries, or ongoing issues between individuals or groups
Content Category: Performance Measurement and Improvement
Cognitive level required for a response: Application
Tasks on the CPHQ exam content outline to which the question is
linked: Facilitate performance improvement teams

Question 60 of 105
952-A team was involved in an initiative to improve care for acute
myocardial infarction (AMI) in an acute care facility from January 2008 to
September 2009. Some of the data collected are shown in the two
graphs below.

214 of 268
Upon reviewing the data in October 2009, the Director of Quality should
recommend

A. closure of the initiative to improve AMI care.


B. further monitoring of the data.
C. a celebration of the team's achievement.
D. the data be reviewed.
Question was not answered.

Answer: D
From time to time, data might appear highly implausible, as in this case. The
data suggest no improvement in the early administration of aspirin from
January 2008 - July 2009, and perhaps some improvement in the AMI
inpatient mortality rate from mid 2009. However, it is very unlikely that the
early administration of aspirin reached 100% for the last two (consecutive)
months, i.e. August and September 2009, so dramatically (i.e. relative to prior

215 of 268
performance). Likewise, it is highly improbable that the AMI inpatient mortality
rate was zero for two consecutive months, given the past data.

Such data warrant a review, preferably by an independent party. I recently


came across similar data in a hospital (not one of our clients) that was
expecting its next JCI on-site survey in less than a month! The latter could
have explained the sudden and dramatic improvement in the data (which was
unlikely to reflect true performance).

Content Category: Performance Measurement and Improvement


Cognitive level required for a response: Analysis
Tasks on the CPHQ exam content outline to which the question is
linked: Analyze/interpret performance/productivity reports

Question 61 of 105
953-On a rheumatology ward of a hospital, a nurse accidentally
administered 20 mg of methotrexate to a patient instead of the
prescribed 7.5mg. When interviewed later, the nurse explained that she
was accustomed to administering 20 mg of methotrexate to patients on
the ward and that she was extremely busy on the morning that she
committed the error.

This is an example of a

A. slip.
B. lapse.
C. knowledge-based error.
D. rule-based error.
Question was not answered.

Answer: A
Slips relate to observable actions and are commonly associated with
attentional or perceptual failures. Lapses are internal events that generally
involve failures of memory. Both slips and lapses are errors in the
performance of skill-based behaviours, typically when attention is diverted.
Knowledge-based errors occur when solving novel problems—this might
occur in an inexperienced or junior professional or someone with a "biased
memory". Rule-based errors occur when the wrong rule is applied for a given
situation, e.g. choosing the wrong arm of a clinical algorithm.

216 of 268
Content Category: Patient Safety
Cognitive level required for a response: Application
Tasks on the CPHQ exam content outline to which the question is
linked: Perform or coordinate risk management

Question 62 of 105
954-According to the Law of Diffusion of Innovations, the highest rate of
rise in innovation diffusion occurs among the

A. innovators.
B. early adopters.
C. early majority.
D. late majority.
Question was not answered.

Answer: C
According to the Diffusion of Innovation theory, the adopter categories are:

 innovators
 early adopters
 early majority
 late majority
 laggards
The rates of adoption for innovations are determined by an individual’s
adopter category. As illustrated in the graph below, the rate of rise in
innovation diffusion is highest among the early majority (represented by the
steepest part of the S-shaped cumulative frequency distribution curve).

217 of 268
Content Category: Management and Leadership
Cognitive level required for a response: Recall
Tasks on the CPHQ exam content outline to which the question is
linked: Facilitate change within the organization

Question 63 of 105
955-A team is evaluating a new screening questionnaire to anticipate
delayed discharge from hospital following hip replacement surgery. The
following table shows the results of using the tool:

What is the negative predictive value of the screening tool?

A. 0.33
B. 0.50
C. 0.66
D. 0.75
Question was not answered.
218 of 268
Answer: D
The negative predictive value (NPV) is the proportion of patients with a
negative test result who do not have the outcome of interest.

NPV = 60/(20 + 60) = 60/80= 0.75

Content Category: Information Management


Cognitive level required for a response: Application
Tasks on the CPHQ exam content outline to which the question is
linked: Interpret outcome data

Question 64 of 70
956-A healthcare professional has been consulted to evaluate the
average monthly waiting time at an orthopaedic clinic over the past 15
months. Which of the following charts indicate that waiting time is NOT
potentially "out of control"?

219 of 268
A. Chart A
B. Chart B
C. Chart C
D. Chart D
Answer: C
Applying Nelson Rules, only Chart C does not meet any of the 8 rules. Chart
A has two out of three points in a row that are more than 2 standard
deviations from the mean in the same direction. Chart B has fifteen points in a
row all within 1 standard deviation of the mean on either side of the mean.
Chart D has nine points in a row on the same side of the mean.
Content Category: Information Management
Cognitive level required for a response: Analysis
Tasks on the CPHQ exam content outline to which the question is
linked: Use or coordinate the use of statistical process control components
(e.g. common and special cause variation, random variation, trend analysis)

220 of 268
Question 65 of 105
957-The Quality Manager of a hospital is using Nelson rules to
determine whether the rate of sharps injuries is out of control. After
reviewing the data, she realizes that one "rule" is missing and proceeds
to include it in her analysis. In doing so, she increases the risk of a

A. Type I error
B. Type II error
C. Standard error
D. Sampling error
Question was not answered.

Answer: A
Nelson rules are used to determine if some measured variable is out of
control. Selecting rules once the data have been reviewed increases the risk
of a “false positive”, i.e. concluding that there is special-cause variation when
in truth there is none. This is a Type I error. A Type II error, in this context,
occurs when one concludes there is no special-cause variation when in truth it
exists. "The standard error of a method of measurement or estimation is the
standard deviation of the sampling distribution associated with the estimation
method." Sampling is not relevant in this case.

Content Category: Information Management


Cognitive level required for a response: Application
Tasks on the CPHQ exam content outline to which the question is
linked: Use epidemiological theory in data collection and analysis

Question 66 of 105
958-Which of the following is an example of a continuous variable?

A. Hospital admissions in year 2009.


B. Hepatitis carrier status of women attending an antenatal clinic.
C. Rates of adverse events as detected by the IHI Global Trigger Tool.
D. Customer satisfaction as measured on a five-point rating scale.
Answer: C
A continuous variable is "a variable that can take on any of a range of values",
e.g. height, weight, time.

221 of 268
The rate of adverse events is a continuous variable—it can take on almost
any value, e.g. 40.1 adverse events/1000 patient-days, 40.2, 40.3,... 41.0,
41.1, etc.

The number of hospital admissions can take on a finite number of values, e.g.
1500, 1501, 1502, etc. (and not 1500.1, 1500.2,... ), and is therefore a
discrete variable.

Hepatitis carrier status is a binomial variable: carrier or non-carrier.

A 5-point rating scale is an example of a categorical variable because


responses can take only one of a finite number of values, e.g. Excellent,
Good, Average, Below Average, Poor.

Content Category: Information Management


Cognitive level required for a response: Application
Tasks on the CPHQ exam content outline to which the question is
linked: Perform or coordinate data definition activities

Question 67 of 105
959-Salaries are included in the

A. operating budget.
B. capital budget.
C. cash budget.
D. ongoing budget.
Question was not answered.

Answer: A
There are three kinds of budgets: operating budget, cash budget, and capital
budget. Salaries are included in the operating budget.
Content Category: Management and Leadership
Cognitive level required for a response: Recall
Tasks on the CPHQ exam content outline to which the question is
linked: Participate in developing and managing a budget for a department

Question 68 of 105
960-In strategic planning, critical success factors

A. define specific strategic end points toward which activity is directed to


achieve the organization's mission and vision.
222 of 268
B. define the organization's aspirations for the future.
C. are things that must be done for an organization to remain viable.
D. are things that must be accomplished for an organization to achieve its
goals.
Question was not answered.

Answer: D
Answer option A describes the organization's goals. Answer option B
describes the organization's vision statement. Between options C and D, the
latter is the better answer.

Content Category: Management and Leadership


Cognitive level required for a response: Recall
Tasks on the CPHQ exam content outline to which the question is
linked: Participate in organization-wide strategic planning

Question 69 of 105
961-In 2009, Hospital A reported the following figures:

What is the incidence rate?

A. 7.1 deaths/1,000 admissions


B. 7.1 deaths/1,000 patient-days
C. 24.2 deaths/1,000 admissions
D. 24.2 deaths/1,000 patient-days
Question was not answered.

Answer: B

223 of 268
The incidence rate is the number of new cases per population in a given time
period. In this case, the denominator is the sum of the person-time of the "at-
risk" population (number admissions × ALOS). Read our article "Measures of
Occurrence".
Content Category: Information Management
Cognitive level required for a response: Analysis
Tasks on the CPHQ exam content outline to which the question is
linked: Use epidemiological theory in data collection and analysis

Question 70 of 105
962-Which of the following are measures of dispersion?

A. Mean, mode, median


B. Correlation, regression, t-test
C. Distribution, analysis of variance, and dispersion factor
D. Range, standard deviation, variance
Question was not answered.

Answer: D
The range is the simplest measure of dispersion. The variance and standard
deviation are commonly used measures of dispersion.

Content Category: Information Management


Cognitive level required for a response: Recall
Tasks on the CPHQ exam content outline to which the question is
linked: Use basic statistical techniques to describe data

Question 71 of 105
963-"From a sample of 300 patients, the estimated rate of patient falls in
Hospital A is 7.2 falls per 1,000 bed-days with a 95% confidence interval
ranging from 6.75 to 7.65 falls per 1,000 bed-days."

Which of the following statements about the confidence interval is correct?

A. The 95% confidence interval will contain the true overall patient fall rate
in Hospital A 95% of the time.
B. We are 95% confident that the true overall patient fall rate in Hospital A
lies within this interval.
C. The 95% confidence interval is an interval containing 95% of the
distribution of the hospital's patient population.

224 of 268
D. The 95% confidence interval indicates the rate at which 95% of the
patients in Hospital A fall.
Question was not answered.

Answer: B
As discussed in our article on inference from a sample mean, the confidence
interval is an interval around the estimated mean which we can be confident
contains the true population mean. In this case, the population is all the
patients in Hospital A.
Content Category: Information Management
Cognitive level required for a response: Application
Tasks on the CPHQ exam content outline to which the question is
linked: Use basic statistical techniques to describe data (e.g. mean, standard
deviation)

Question 72 of 105
964-Which of the following bodies is ultimately responsible for
credentialing in a hospital?

A. Chief Executive Officer


B. Chief Medical Officer
C. Governing Body
D. Credentialing Committee
Question was not answered.

Answer: D
In almost all healthcare organizations, particularly in the USA, a Credentialing
Committee makes the final decision(s) on credentialing.
Content Category: Performance Measurement and Improvement
Cognitive level required for a response: Recall
Tasks on the CPHQ exam content outline to which the question is
linked: Participate in the credentialing and privileging process

Question 73 of 105
965-In a large tertiary hospital, 10.3% of a general surgeon's cases in the
last 3 months were associated with surgical site infections. The average
surgical site infection rate for the other general surgeon's was 4.8%.
Working closely with the Chief Medical Officer, the healthcare quality
professional should

225 of 268
A. compare the hospital's overall surgical site infection rate with local and
national data.
B. examine the surgeon's case-mix, risk-adjusted outcomes and practice
patterns.
C. refer the surgeon's cases for peer review.
D. compare the surgeon's surgical site infection rate with that of surgeons
in other specialties.
Question was not answered.

Answer: B
The most appropriate course of action is to determine if the surgeon's SSI rate
is consistent with past practice and also if his cases in the last quarter carried
an increased risk - more complex, clean/clean-contaminated, co-existing
diseases such as diabetes mellitus, etc. If the data indicate an issue in the
surgeon's practice, then his cases should be referred to a peer review body.
Comparing the hospital's data with local and national data will not help in this
case. A comparison between the surgeon's SSI rate and that of surgeons in
other specialties is not appropriate due to the likely difference in risk.

Content Category: Performance Measurement and Improvement


Cognitive level required for a response: Application
Tasks on the CPHQ exam content outline to which the question is
linked: Analyze/interpret performance/productivity reports

Question 74 of 105
966-Root cause analysis following a sentinel event will probably require a

A. flow chart.
B. Gantt chart.
C. force field analysis chart.
D. control chart.
Question was not answered.

Answer: A
All root cause analyses will need a thorough understanding of the
actions/conditions/materials in the process that led to the sentinel event. The
best way to achieve this and to document the series of steps is with a flow
chart. The other answer choices are not appropriate: Gantt charts are used in
project management, a force field analysis chart may be used in group
dynamics and action research, and the control chart is a popular statistical
tool.

226 of 268
Content Category: Patient Safety
Cognitive level required for a response: Recall
Tasks on the CPHQ exam content outline to which the question is
linked: Perform or coordinate risk management (root cause analysis)

Question 75 of 105
967-Compared with its previous version, the Procedure Coding System
(PCS) of the tenth edition of the International Classification of Diseases
(ICD) standards describes procedures in greater detail and thus
improves

A. sensitivity.
B. specificity.
C. reliability.
D. precision.
Question was not answered.

Answer: B
For an explanation of the answer, read our article on statistics for diagnostic
tests.
Content Category: Information Management
Cognitive level required for a response: Application
Tasks on the CPHQ exam content outline to which the question is
linked: Interpret data to support decision making

Question 76 of 105
968-In a facility which allows verbal/telephone orders, a nurse is asked
to take a telephone order for the sedative medication Zoplicone to help a
patient with insomnia. The nurse should

A. ask the physician to write the order himself before she administers the
medication as it is not an emergency.
B. record the order word-for-word on the medication order sheet, read
back the order and get confirmation from the physician who gave the order.
C. ask another nurse to take the order.
D. record the order word-for-word on the order sheet, ask another nurse
to verify it is correct, and then administer the medication to the patient.
Question was not answered.

Answer: B

227 of 268
In general, the correct procedure for taking a verbal/telephone order is
outlined in the second answer choice (B). If the nurse has difficulty
understanding the order, she should ask another nurse to listen in as the first
nurse takes the order, after which the second nurse should read it back and
sign the order as well. This was a National Patient Safety Goal of The Joint
Commission (USA) and is one of JCI's International Patient Safety Goals.

Content Category: Performance Measurement and Improvement


Cognitive level required for a response: Application
Tasks on the CPHQ exam content outline to which the question is
linked: Integrate accreditation and regulatory recommendations into the
organization

Question 77 of 105
969-The rate of sharps injuries in Hospital X is shown in the chart below:

Which of the following steps should the healthcare quality professional take?

A. Educate the staff in Hospital X on sharps injuries prevention.


B. Review the policies and procedures for the handling of sharps injuries
in Hospital X and revise them if necessary.
C. Continue to monitor the rate of sharps injuries.
D. Conduct drill-down analysis of sharps injuries in Hospital X.
Question was not answered.

Answer: C
The chart above is a run chart, commonly used in healthcare organizations.
Following the "trending rules" described in our article on using run charts, no
real trend is present. Therefore, the most appropriate action is to continue
monitoring sharps injuries in the hospital.
228 of 268
Content Category: Information Management
Cognitive level required for a response: Application
Tasks on the CPHQ exam content outline to which the question is
linked: Use or coordinate the use of statistical process control components
(e.g. common and special cause variation, random variation, trend analysis)

Question 78 of 105
970-Which of the following strategies is MOST effective in achieving
widespread adoption of a new electronic medical record system?

A. Convincing respected members of the medical staff to adopt the


system.
B. Require all salaried staff to use the system first.
C. Institute financial penalties for not using the system.
D. Implementing the system in phases, starting with the most essential
features.
Question was not answered.

Answer: A
Of the four answer options, widespread adoption of the new EMR system is
most likely to occur with its early use by respected members of the medical
staff. Read our article on diffusion of innovation for more tips on how to
spread new technologies.
Content Category: Management and Leadership
Cognitive level required for a response: Application
Tasks on the CPHQ exam content outline to which the question is
linked: Facilitate change within the organization

Question 79 of 105
971-In developing a patient safety training program, the healthcare
quality professional must first

A. develop instructional objectives.


B. assess trainees' needs.
C. prepare training materials.
D. develop a pre- and post-test to assess knowledge of the subject.
Question was not answered.

Answer: B

229 of 268
The first step in most models for training development is an analysis of
trainees' needs.

Content Category: Performance Measurement and Improvement


Cognitive level required for a response: Application
Tasks on the CPHQ exam content outline to which the question is
linked: Develop organizational performance improvement training (e.g.
quality, patient safety)
Question 80 of 105
972-In your capacity as Director of Quality and Patient Safety, you are
asked to develop a budget for the quality management department.
Because you anticipate growth of this department in the coming year,
you included the cost of new furniture and equipment, estimated at
$5000. Under which type of budget would your department's new
furniture and equipment fall?

A. Operating Budget
B. Capital Budget
C. Fixed Budget
D. Master Budget
Question was not answered.

Answer: B
In general, furniture and equipment that cost more than $1000 - $2000 are
classified under the organization's Capital Budget. Costs of about $5000 in
most healthcare organizations will be treated as a capital budget expense.
Content Category: Management and Leadership
Cognitive level required for a response: Application
Tasks on the CPHQ exam content outline to which the question is
linked: Participate in developing and managing a budget for a department

Question 81 of 105
973-Concerning control charts, each of the following statements is true
EXCEPT:

A. The standard error is calculated.


B. Control charts are used to evaluate whether or not a process is in a
state of statistical control.
C. The upper and lower control limits are drawn only at 3 standard errors
from the center line.
D. Control charts may be divided into zones.

230 of 268
Question was not answered.

Answer: C
Our article on control charts describes their key features. The control limits
need not be drawn at 3 standard errors; they could (for example) be drawn at
2 standard errors from the center line.
Content Category: Information Management
Cognitive level required for a response: Recall
Tasks on the CPHQ exam content outline to which the question is
linked: Use or coordinate the use of process analysis tools to display data
(e.g. fishbone, Pareto chart, run chart, scattergram, control chart)

Question 82 of 105
974-A suspicious death after surgery occurred in a prestigious hospital.
Initial reports suggested that post-operative nurses might have misread
physician orders for intravenous fluids. Senior management had
concerns about litigation and adverse publicity. In making rapid
decisions, the Chief Executive Officer of the hospital should adopt a
leadership style that is

A. participatory.
B. consultative.
C. autocratic.
D. democratic.
Question was not answered.

Answer: C
This question describes a crisis. An autocratic leadership style is the most
appropriate in crises.
Content Category: Management and Leadership
Cognitive level required for a response: Application
Tasks on the CPHQ exam content outline to which the question is
linked: Facilitate assessment and development of the organization’s quality
culture

Question 83 of 105
975-A 35 year old man presented to the Emergency Room with diabetic
ketoacidosis, a life-threatening complication of diabetes mellitus. The
patient's diabetes had been undiagnosed previously. Discharge
planning should begin

231 of 268
A. at the time of admission to the hospital.
B. after the patient's medical condition stabilizes and he is transferred
from the Intensive Care Unit to a medical ward.
C. after the physician writes the discharge planning order.
D. two days before the expected date of discharge.
Question was not answered.

Answer: A
In general, discharge planning should commence as soon as the patient is
admitted.

Content Category: Performance Measurement and Improvement


Cognitive level required for a response: Application
Tasks on the CPHQ exam content outline to which the question is
linked: Coordinate the performance improvement process

Question 84 of 105
976-A patient with no prior history of major medical problems was
admitted for an elective cholecystectomy. On the second postoperative
day, the patient started to experience pain at the operative site and high
fevers. Blood cultures were positive for Escherichia coli and other
investigations confirmed the presence of a surgical site infection. The
patient died of overwhelming septicaemia in the Intensive Care Unit 7
days after his operation. From a quality standpoint, this case is best
classified as a
A. clinical mishap.
B. adverse event.
C. never event.
D. sentinel event.
Question was not answered.

Answer: D
In general, The Joint Commission, JCI and many other agencies consider
healthcare-associated infection associated with a death or permanent
disability as a sentinel event.

A healthcare-associated infection is an "adverse event" but the CPHQ exam


requires you to select the best answer choice. In this case, the best answer is
"sentinel event."
This is not a case of medical or clinical mishap, which occurs when "injury or
damage is caused by mischance or accident, unexpected and undesigned".

232 of 268
The National Quality Forum's list of "Never Events" are grouped into six
categories: surgical, product or device, patient protection, care management,
environmental, and criminal. The incident described in this question does not
meet the description of any of these "never events".
Content Category: Patient Safety
Cognitive level required for a response: Application
Tasks on the CPHQ exam content outline to which the question is
linked: Perform or coordinate risk management

Question 85 of 105
977-The Emergency Department of Hospital X sees an average of 200
patients per month for the management of acute chest pain. The
department plans to conduct a retrospective chart review to determine
the compliance with a protocol for managing chest pain. In reviewing
the charts of patients seen in the department in Year 2009, an
appropriate size of a randomly selected sample is

A. 50.
B. 70.
C. 100.
D. 200.
Question was not answered.

Answer: B
The total number of cases for the year was 200 × 12 = 2400. As indicated
by The Joint Commission guidelines for sampling, the most appropriate
sample size is 70. This number of charts is sufficient to give the department
staff the answer they are seeking. A smaller number will not give adequate
precision while a much larger number will be excessive and therefore waste
resources.
Content Category: Information Management
Cognitive level required for a response: Analysis
Tasks on the CPHQ exam content outline to which the question is
linked: Perform or coordinate data collection methodology

Question 86 of 105
978-The senior management team of Hospital Z is reviewing data from
several initiatives aimed at improving the inpatient mortality rate from
several conditions. The data is summarised in the table below.

233 of 268
In order to prevent the highest possible number of deaths from these
conditions, for which two conditions should the hospital implement
evidence-based care bundles (proven to reduce mortality) in the coming
year?

A. Acute Myocardial Infarction and Asthma.


B. Asthma and Surgical Site Infections.
C. Community-Acquired Pneumonia and Surgical Site Infections.
D. Ventilator-Associated Pneumonia and Surgical Site Infections.
Question was not answered.

Answer: C
To answer this question, several assumptions have to be made (as in many
other decision-making scenarios), e.g. all other factors that contribute to
inpatient mortality remain the same. The overall impact of the initiatives is
measured by the product of the number of cases and the magnitude of effect
of the interventions. You should be aware that hospitals are implementing
bundles of care to improve survival outcomes for all the conditions above. The
benchmark figures can be assumed to be the best in the industry, which gives
us the potential change in effect. Note that this hospital outperformed the
benchmark for two conditions - the hospital should prioritise its improvement
initiatives on other conditions to get the best return, which is survival in this
case. The table below shows you the calculations involved to derive the
correct answer.

Content Category: Performance Measurement and Improvement


Cognitive level required for a response: Analysis

234 of 268
Tasks on the CPHQ exam content outline to which the question is
linked: Facilitate establishment of priorities for process improvement activities

Question 87 of 105
979-A 45 year old patient died in the Surgical Intensive Care Unit
overnight after receiving medication intended for another patient. The
Crisis Management Team is most effective if it is chaired by the

A. Chief Executive Officer


B. Head of the Department of Intensive Care
C. Director of Quality & Patient Safety
D. Chief Pharmacist
Question was not answered.

Answer: A
The Crisis Management Team following a serious clinical adverse event is
most effectively led by the Chief Executive Officer. For more information, read
the IHI White Paper on "Respectful Management of Serious Clinical Adverse
Events".
Content Category: Patient Safety
Cognitive level required for a response: Application
Tasks on the CPHQ exam content outline to which the question is
linked: Facilitate assessment and development of the organization’s patient
safety culture

Question 88 of 105
980-The Board and Chief Executive Officer have renewed their
commitment to improving quality in Hospital X. Your primary role as the
Director of Quality & Patient Safety should be

A. Data Consultant.
B. Team Leader.
C. Facilitator.
D. Quality Champion.
Answer: C
In the ideal situation, the healthcare quality professional facilitates
performance improvement in an organization by acting as a coach/internal
consultant. The other roles listed above may also apply but they should not be
the primary responsibility of the healthcare quality professional.

235 of 268
Content Category: Management and Leadership
Cognitive level required for a response: Recall
Tasks on the CPHQ exam content outline to which the question is
linked: Facilitate assessment and development of the organization’s quality
culture

Question 89 of 105
981-As the Director of Quality, you have recommended the engagement
of an external healthcare quality consultant to fill some gaps in
knowledge and time.

In selecting an external consultant, which of the following has the


LEAST impact on your decision?

A. Reputation of the consulting firm.


B. Reputation of the consultant(s) in the consulting firm.
C. Recommendations of previous clients.
D. Budget.
Question was not answered.

Answer: A
When working with consultants, it is important to choose the right consultant
for the job, i.e. one who is able to deliver on time and on budget. In general, a
reputable firm is desirable but you are hiring the expertise of the firm, i.e. the
people in the firm. You should seek recommendations from previous clients.
Your budget also plays a large part in whom you decide to engage.

Content Category: Management and Leadership


Cognitive level required for a response: Application
Tasks on the CPHQ exam content outline to which the question is
linked: Monitor the activities of consultants (e.g. quality and patient safety)

Question 90 of 105
982-In implementing a care bundle for the management of acute
myocardial infarction, the recording of the extent to which smoking
cessation counseling is provided is a measure of

A. structure.
B. process.
C. outcome.
D. process and outcome.

236 of 268
Question was not answered.

Answer: B
To learn more about structure, process and outcome measures, read our
article on Assessing Quality of Care: Structure, Process, and Outcome.
Content Category: Information Management
Cognitive level required for a response: Application
Tasks on the CPHQ exam content outline to which the question is
linked: Facilitate development or selection of process and outcome measures

Question 91 of 105
983-Transparent communication with patients and families after a
serious clinical adverse event

A. reduces patient satisfaction.


B. increases the risk of litigation.
C. improves patient satisfaction but increases the risk of medico-legal
claims.
D. improves patient satisfaction and reduces the risk of medico-legal
claims.
Question was not answered.

Answer: D
Recent research indicates that disclosure and apology (when appropriate)
following serious unanticipated clinical outcomes improve patient satisfaction
and reduce the risk of medico-legal claims.

Content Category: Patient Safety


Cognitive level required for a response: Recall
Tasks on the CPHQ exam content outline to which the question is
linked: Integrate patient safety concepts within the organization

Question 92 of 105
984-For which of the following scenarios would an uncontrolled before-
and-after evaluation design be most appropriate?

A. Evaluating the effectiveness of a one-day training course for nurses on


infection prevention and control.
B. Evaluating the effectiveness of a four-week educational program for
nurses to prevent back injury.

237 of 268
C. Evaluating the effectiveness of a training program for nurses to prevent
patient slips and falls after an unusually high rate of patient falls in the
preceding 12 months.
D. Evaluating the effectiveness of an Acute Myocardial Infarction (AMI)
care bundle in reducing inpatient AMI mortality.
Question was not answered.

Answer: A
The before-and-after evaluation design, common in healthcare quality
improvement work, is exposed to several threats to (internal) validity. This
design is most useful in demonstrating the immediate impacts of short-term
programmes, such as a one-day training course, i.e. with a pre-test and a
post-test (provided the two are not identical and the answers to the post-test
are not given to the group during the training). Over a four-week program
(answer option B), it is more likely that other factors contribute to the outcome
while the intervention of interest is administered, i.e. threats to internal validity.
Answer option C is not the best answer because the post-training result may
be affected by "reversion-to-the-mean". In other words, a period of higher-
than-average fall rates is likely to be followed by a period of lower rates.
Therefore, we cannot be sure if any observed improvement in the patient fall
rate is due to the intervention or by other factors. Likewise, the results of
measuring the effectiveness of an AMI care bundle in reducing the AMI-
related death rate may be confounded. For this reason, it is often useful to
compare your organization's data with those of organizations in the same
community.

Content Category: Performance Measurement and Improvement


Cognitive level required for a response: Application
Tasks on the CPHQ exam content outline to which the question is
linked: Evaluate effectiveness of performance improvement training

Question 93 of 105
985-The Model for Improvement, developed by Associates in Process
Improvement,

A. is identical to the PDSA cycle.


B. may be applied in only a limited number of clinical areas.
C. may replace Lean Six Sigma that an organization is using.
D. is used to accelerate improvement.
Answer: D
The Model for Improvement consists of two parts:

238 of 268
 A "Thinking Part", and
 A "Doing" Part", which is the PDSA cycle.
This tool has been successfully applied in many different clinical settings to
accelerate improvement. However, it is not meant to replace existing
improvement methodologies.

Read our article on the Model for Improvement for more information.
Content Category: Performance Measurement and Improvement
Cognitive level required for a response: Recall
Tasks on the CPHQ exam content outline to which the question is
linked: Facilitate performance improvement teams

Question 94 of 105
986-After a comprehensive review of the benefits and risks, a hospital's
Board of Directors decided to cease the oncology service within the
next 6 months. This is an example of

A. risk avoidance.
B. risk prevention.
C. risk shifting.
D. risk financing.
Question was not answered.

Answer: A
In the field of risk management, risk control includes risk avoidance, risk
prevention and risk shifting.
Content Category: Information Management
Cognitive level required for a response: Application
Tasks on the CPHQ exam content outline to which the question is
linked: Perform or coordinate risk management

Question 95 of 105
987-A patient diagnosed with hepatocellular carcinoma is receiving a
novel chemotherapeutic agent based on promising preliminary data
from clinical trials and the absence of other viable treatment options.
The dimension of quality for which the medication was chosen is its

A. efficacy.
B. effectiveness.
C. safety.

239 of 268
D. appropriateness.
Question was not answered.

Answer: A
Efficacy refers to the medicine's capacity to produce a desired effect.
Effectiveness is the degree to which the desired outcome is achieved.

Safety, in this context, refers to the extent to which the risks of taking the
medication are reduced.

Appropriateness refers to the degree to which the care provided is relevant to


the patient's clinical needs.

Content Category: Performance Measurement and Improvement


Cognitive level required for a response: Application
Tasks on the CPHQ exam content outline to which the question is
linked: Analyze/interpret performance/productivity reports

Question 96 of 105
988-As the Director of Quality & Patient Safety, you introduced the
Institute for Healthcare Improvement (IHI)) Global Trigger Tool for
measuring adverse events about 2 months ago. You now intend to
present data collected using this tool to the hospital's Board of
Directors, most of whom are laypersons. Which of the following
measures will you choose to present your findings?

A. Adverse events per 1,000 patient-days


B. Adverse events per 100 admissions
C. Adverse events per 1,000 admissions
D. Percent of admissions with an adverse event
Question was not answered.

Answer: D
One of the most important responsibilities of a healthcare quality professional
is to communicate data to a variety of people. In this case, "percent of
admissions with an adverse event" will be most easily understood by
laypersons. See Page 13 of the white paper on the IHI Global Trigger Tool for
Measuring Adverse Events for more information.
Content Category: Information Management
Cognitive level required for a response: Application
Tasks on the CPHQ exam content outline to which the question is
240 of 268
linked: Coordinate the dissemination of performance improvement
information within the organization

Question 97 of 105
989-Healthcare workers should perform hand hygiene

A. before entering a patient's room.


B. before entering a patient's room and again immediately before touching
a patient.
C. after touching a patient's bedside table.
D. after leaving a patient's room.
Question was not answered.

Answer: C
Every healthcare quality professional is expected to know common clinical
processes. These include processes in infection control and medication
management. Not surprisingly, the CPHQ exam will include some questions
that appear to be "clinical" and not really related to quality management.
Answering such questions may be straightforward to those with plenty of
clinical experience but challenging for others.
The question above addresses the indications for the use of hand hygiene.
Hand hygiene should be performed:

 Immediately before touching a patient (close to the site of care to avoid


recontamination) or when entering a "patient zone", i.e. the patient and his/her
immediate surroundings.
 Just after touching a patient and before touching any object located
outside the patient zone.
 After touching objects located in the vicinity of the patient. (Answer
option C for the question above.)
 Immediately before touching non-intact skin.
 Before manipulating invasive devices
 After contact with body fluids, mucous membranes, nonintact skin, or
wound dressings—even in the absence of visible soiling and even when
gloves have been used.
This video on hand hygiene provides an excellent synopsis of hand hygiene.
Answer option C is the best answer.

241 of 268
Content Category: Performance Measurement and Improvement
Cognitive level required for a response: Recall
Tasks on the CPHQ exam content outline to which the question is
linked: Participate in the process of infection control processes

Question 98 of 105
990-The performance improvement model adopted by any healthcare
organization should include all of the following, EXCEPT

A. a focus on prioritized areas.


B. the PDCA cycle.
C. use of statistical and analytical tools.
D. action for improvement.
Question was not answered.

Answer: B
The PDCA cycle is merely one of many different performance improvement
models, and may not be applicable in all healthcare organizations. The other
characteristics listed are common to all models.

Content Category: Management and Leadership


Cognitive level required for a response: Recall
Tasks on the CPHQ exam content outline to which the question is
linked: Evaluate applicability of performance improvement models (e.g.
FOCUS, PDCA, Six Sigma)

Question 99 of 105
991-Effective quality management in healthcare requires leaders who
are

A. well-respected clinicians.
B. department chairs.
C. committed to the organization's mission, vision and values.
D. members of the Quality Council.
Question was not answered.

Answer: C
Effective leaders in healthcare quality first and foremost demonstrate a
commitment to the mission, vision and values of the organization.

242 of 268
Content Category: Management and Leadership.
Cognitive level required for a response: Recall
Tasks on the CPHQ exam content outline to which the question is
linked: Facilitate development of leadership values and commitment

Question 100 of 105


992-Which of the following is the most effective way to prevent
accidental intravenous administration of epidural bupivacaine (a local
anaesthetic) due to epidural catheters being inadvertently attached to
intravenous lines?

A. Redesigning epidural catheters so that they cannot be attached to an


intravenous line.
B. Regular reminders to doctors and nurses to be careful when
administering epidural bupivacaine.
C. Affixing stickers that state epidural catheters are for epidural use only.
D. Training of doctors and nurses.
Question was not answered.

Answer: A
Redesign of epidural catheters so that they cannot be attached to an
intravenous line offers the best chance of overcoming human error, therefore
preventing accidental attachment of epidural catheters to intravenous lines.

Content Category: Patient Safety


Cognitive level required for a response: Application
Tasks on the CPHQ exam content outline to which the question is
linked: Integrate patient safety concepts within the organization

Question 101 of 105


993-Following a non-fatal overdose of intravenous heparin (a blood
thinner) in a 43 year old man in a cardiac care unit, which of the
following is LEAST like to prevent the occurrence of a similar event?

A. Requiring an additional member of the clinical team to check all


intravenous administrations of heparin.
B. Taking disciplinary action against any nurse found to have
administered an incorrect dose of medication.
C. Introducing a combined heparin order form and documentation tool.
D. Encouraging low molecular weight heparin (administered
subcutaneously) in lieu of intravenous heparin.

243 of 268
Question was not answered.

Answer: B
Answer options A, C, and D give methods that have the potential to reduce
adverse events. Answer option B offers an action that is common among
hospitals but is erroneous for at least two reasons—the practice of modern
patient safety emphasizes a non-punitive (or just) healthcare system and a
focus on system (rather than individual) failures, among other things.

Content Category: Patient Safety


Cognitive level required for a response: Application
Tasks on the CPHQ exam content outline to which the question is
linked: Integrate patient safety findings into governance and management
activities (e.g. bylaws, administrative policies, and procedures)

Question 102 of 105


994-Which of the following is a healthcare-associated infection?

A. A patient who was admitted for the treatment of acute myocardial


infarction and who had three positive urine cultures all isolating Pseudomonas
aeruginosa. The patient did not have an indwelling urinary catheter.
B. A patient who was admitted with fever and cough, and was diagnosed
with severe Pseudomonas aeruginosa pneumonia.
C. Cytomegalovirus infection in a newborn that was diagnosed 36 hours
after birth.
D. A 65-year old man who was admitted for elective bowel surgery and
developed shingles 48 hours after his operation.
Question was not answered.

Answer: A
Only the first answer option meets the criteria for a healthcare-associated
infection, in particular asymptomatic bacteriuria.
Content Category: Performance Measurement and Improvement
Cognitive level required for a response: Application
Tasks on the CPHQ exam content outline to which the question is
linked: Participate in the process of infection control processes

Question 103 of 105


995-A healthcare quality professional is conducting a study to examine
the relationship between cigarette smoking and unanticipated
admission to the Intensive Care Unit (ICU). All patients were categorised

244 of 268
as being current smokers or non-smokers (the latter category included
ex-smokers). The patients were also categorised as having been
admitted to the ICU unexpectedly or not. Assuming the expected value
in any category is greater than 20, the most appropriate statistical test is
the

A. t-test
B. z-test
C. chi-squared test
D. eye-ball test
Question was not answered.

Answer: C
The chi-squared test is often used to conduct tests of hypothesis that involve
data presented in a 2 × 2 contingency table.

Content Category: Information Management


Cognitive level required for a response: Application
Tasks on the CPHQ exam content outline to which the question is
linked: Use epidemiological theory in data collection and analysis

Question 104 of 105


996-On which of the following is internal benchmarking most
dependent?

A. Validity
B. Sensitivity
C. Reliability
D. Accuracy
Question was not answered.

Answer: C
Internal benchmarking is the process by which an organization compares
performance of various units and identifies best practices for dissemination
within the organization.

In this case, validity and accuracy mean the same thing, i.e. the degree of
closeness of a measurement to the true (actual) value.

Sensitivity, in statistics, refers to the proportion of patients with the


disease/condition detected by a positive test.
245 of 268
In statistics, reliability is "the consistency of a set of measurements or of a
measuring instrument, often used to describe a test". The term is analogous
toprecision.
For internal benchmarking, reliability is important to derive meaningful
conclusions due to the need for comparison. On the other hand, validity,
accuracy and sensitivity are all desirable but not essential for internal
benchmarking.

Content Category: Management and Leadership


Cognitive level required for a response: Recall
Tasks on the CPHQ exam content outline to which the question is
linked: Assist in developing objective performance measures/indicators

Question 105 of 105


997-A laissez-faire leadership style

A. is preferred in Total Quality Management (TQM).


B. involves the leader/manager providing regular feedback to let
employees know how well they are doing.
C. involves no decisions by the leader/manager.
D. involves the leader/manager presenting a tentative decision, receiving
feedback from the group, and then making the final decision.
Question was not answered.

Answer: C
In a laissez-faire style of leadership, no limits are set and no decisions are
made by the leader/manager.

Answer options A, B and D give false statements about a laissez-faire


leadership style.

Content Category: Management and Leadership


Cognitive level required for a response: Application
Tasks on the CPHQ exam content outline to which the question is
linked: Identify the appropriate team structure (e.g. cross functional, self-
directed)

Preparation for the Certified Professional in Healthcare Quality (CPHQ)


Exam Course

Quiz 3
246 of 268
Please answer the following 15 multiple choice questions

998-The formal functions of management include all except:


A. Planning
B. Organizing
C. Directing
D. Inspecting

999-The principle underlying the selection of an organization-wide


quality council is:
A. One oversight body
B. Leadership control
C. One-cross functional team
D. Elimination of department/service quality control

1000-Why should a UM plan include a conflict of interest statement


A. To provide for unbiased decisions
B. To prevent economic credentialing
C. To provide for security and integrity of information
D. To provide immunity for physician reviewers

1001-The key advantage of case management in Managed care is:


A. Control of clinical risk
B. Control of hospital use
C. Coordination of care
D. Prevention of illness

1002-The written scope of care and service of a healthcare organization


is best described as:
A. A plan describing the linkages between care processes and outcomes
B. The delineated activities performed by governance, management,
clinical and support personnel
C. A logical sequence of operations to be performed to care for and serve
delineated populations of patients
D. An interactive series of steps, processes, functions, and systems

247 of 268
B

1003-The key issue in integrating the functions of utilization


management, quality management, and clinical risk management
resolves around:
A. Consolidation of leadership
B. Information management
C. Control by the quality council
D. Cross-training staff

1004-According to total quality management principles, managers


should:
A. Lead with autocratic decision making
B. Communicate successes or failures only to the boss
C. Lead with participative decision making
D. Focus on short term financial impact of quality improvement techniques

1005-Medical center had begun performing angioplasty procedures for


cardiac patients in anticipation of providing cardiovascular surgery
services. The administration then failed to negotiate a contract with the
cardiac surgery team of physicians. The hospital and its cardiologists
then negotiated an exclusive contract with another hospital in the area
to refer all cardiac patients needing angioplasty to that facility. This
action constitutes:
A. Risk retention
B. Risk avoidance
C. Risk shifting
D. Risk prevention

1006-Your hospital case management program monitors length of stay


(LOS) by condition. LOS for four conditions has decreased slightly each
of the last six quarters. To evaluate cost and quality of care impact, you
recommend which measures?
A. Denials, comparison with previous two years LOS, readmissions.
B. Staff productivity changes, reimbursement, LOS at next level of care
C. Reimbursement, comparison with conditions with increasing LOS,
denials
D. Outcome of transition plan, reimbursement, readmissions

1007-In revising the Utilization Management (UM) Plan, which of the


following is most important to consider?
248 of 268
A. External UM contract requirements
B. Accreditation survey results impacting UM
C. UM performance measures results
D. Clinical pathway length of stay variances

years old women with right Hip replacement surgery , fall from 75 -1007
bed in hospital to floor and the other hip was fractured. Risk manager
visit the patient family and told them the hospital will be written off. This
:action is

.A. Risk loss

.B. Risk shift

.C. Risk identification

.D. Potential compensated event

:Strategic management does not include -1008

.A. Creating a fit between the organization and its external environment

.B. Facilitate consistent decision making

.C. Foster anticipation, innovation and excellence

.D. Make day to day work flow easy

Health care quality program had prepared a balance score card, -1009
that displayed: patient satisfaction was 98%, financial target has been
met , medication error had been increased by 30%, and the heart
surgery rate decrease 3%. What additional information the governing
?body may ask for

.A. Type of medication error

.B. Heart surgery case

.C. Patient satisfaction data

.D. Review patients’ complaint

Which of the following could best be solved by quality -1010


?improvement team

249 of 268
.A. Customer complaints

.B. Claim issue

.C. Financial problem

.D. Administration issue

During the process of strategic planning, the organization should -1011


:look for the basic components, like

.A. Internal and external environment

.B. Human resources

.C. Finance

.D. Management

To increase the impact of reengineering and to assure the greatest -1012


…opportunity for success, the quality leader should first

.A. Select conduct review


.B. Plan carefully, communicate widely and lead effectively
.C. Define the concept of reengineering to employees
.D. Develop policies first

The most important initial step in preparation for accreditation -1013


… survey is

.A. Ensure clinical competency

.B. Provide teaching tools

.C. Standards education

.D. Quality improvement activities

Health care quality professional has been asked to present -1014


information to senior leadership about hospital publicity. The report
:should include
250 of 268
.A. Clinical expert

.B. Computer training

.C. Comparison of data with benchmark

.D. Customer satisfaction

In a health care organization, chief executive officer requested the -1015


most effective recommendations to assess the organization readiness.
Which of the following method should health care quality professional
?recommend first

.A. Administer survey to evaluate organization culture

.B. Review of performance up result

.C. Contact a quality consultant to conduct a review

.D. Walk through organization

Which of the following is the least important in developing a -1016


?standard of practice

.A. Clinical knowledge of peer

.B. Search finding

.C. Patient expectation

.D. Clinical trials

For continuous quality improvement team to be successful, who -1017


?must be included in the team

.A. Administrator

.B. Department supervisor

.C. Staff

.D. Facilitator

251 of 268
The responsibility to pass quality improvement values to the -1018
:organization is of the

.A. Leader

.B. Quality professionals

.C. Senior staff

.D. Accreditation agency

To gain the leadership commitment for performance improvement -1019


projects, the quality professional should focus first on which of the
:following

A. Importance of the project on employee satisfaction and financial health of


.the organization

B. Importance of the project on the accreditation status and the clinical


.process

C. Importance of the project on the financial health and accreditation status of


.the organization

.D. Effect of the project on the community

:The important things for a particular team are -1020

.A. Culture of the group

.B. Norms of behavior

.C. Values of the group

.D. Language communicated by the group

After providing training, the trainer evaluated the increased skill -1021
?within the trainees, this is considered what level of evaluation

.A. Reaction level

.B. Results level

.C. Behavior level

.D. Learning level


252 of 268
D

Training within an organization should be delivered to which -1022


?levels

.A. Group by group training

.B. Just on time training

.C. On price tag training

.D. Individualized training

To enhance coping of the desired behavior by the employees, you -1023


:should

.A. Punish the undesired behavior makers

.B. Make rewards on the desired behavior

C. Make the desired behavior appear as normal requirement and needs


.no recognition

.D. Blame and change the undesired behavior makers

One of the team members that keep members on track and focus -1024
:on the process is

.A. Leader

.B. Facilitator

.C. Quality manager

.Minutes recorder .E

An example of integrating results of a utilization management -1025


:assessment in the performance improvement process is

.A. Educate case manager in discharge planning

.B. Hiring more nurses

.C. Reporting assessment results to executive staff

253 of 268
.D. Assessing results of patient satisfaction survey

Analysis of events, trends, and customer need is the initial phase -1026
:of

.A. Resource management

.B. Strategic planning

.C. Forecasting

.D. Cost/benefit analysis


B

1027-The key to creating sustained value in the organization is to

A.Delegate policy-making and oversight to the quality council


B.Develop s strategic that derives from the version, strategic goals, and cost
–benefit analyses
C.Adopt an organizational ethics policy and code linked to mission, vision,
and values
D.Act on predictive performance measures aligned to strategic goals and
departmental objectives

1028- The person/group legally responsible for maintaining quality


patient care is the
A. Governing body
B. Quality improvement council
C. Chief executive officer
D. Medical/ professional staff

1029- "Organization function" refer to


A. Key governance, management, clinical, and support activities
B. Functions of the governing body
C. Cross-functional team activities
D. Legal and fiduciary obligations to patients

1030-Which of the following statements refers only to strategic planning


and not to the former traditional "master planning "?
A. Planning focuses primarily on producing new services
B. Planning begins with the statement of mission
C. Planning is an integral part of management
254 of 268
D. Planning ignores the political environment of the organization

1031-The mission statement of the organization describes


A. Where the organization is going
B. The purpose of the organization
C. The strategic direction of the organization
D. The long-term goals of the organization

1032-A team has been selected from all linked services in several
healthcare organizations in the WeCare Health plan network to address
information management . the best term describes a team is
A. Departmental
B. Service-line
C. Interdepartmental
D. Cross-function ional

1033-Strategic leadership is linked to success in meeting


A. budget requirements
B. Intended objectives
C. Governing body policy
D. Contract requirements

1034-In crises situation, when a manger must make a rapid decision, the
most effective leadership style is:
A. Consultative
B. Participatory
C. Autocratic
D. Democratic

1035-In the "Quality –based strategic planning" model


A. Representatives from each active QI team from the strategic planning
team
B. The steering council leads strategic planning as an ongoing activity
C. Licensed independent practitioners lead the strategic planning effort and
formulate the performance measures
D. Input from management and staff is the key assessment activity

255 of 268
1036-As a performance measurement system, the key value of the
""balanced scorecard" concept is its ability to
A. Serve as a comparative "report card" with like organizations
B. Focus the organization on financial measures of survival and success
C. Encompass all the organization's clinical and non clinical measures
D. Align measurement with the vision and strategy of the organization

1037-Leadership during a lengthy period of crises in the organization is


A. Based on the leader's position in the organization
B. A participative activity performed by anyone committed to lead
C. Dependent on a set of personal characteristics
D. An autocratic style with decision made solely by the leader.

1038-Having management pay attention to workers' activities results in


A. Decreased anxiety
B. Decreased productivity
C. Increased anxiety
D. Increase productivity

1039-You have joined the newly merged preferred health, a for-profit


integrated delivery system (IDS), as Vice President for quality. You are
responsible for reengineering or otherwise integrating the QM/QI
function across the provider network. Based on your understanding of
systems, corporate culture, ethics, quality , leadership's influence , and
strategic planning, answer the questions :

1- Which of the following statements and


document are most likely to reveal the
organization's underlying or true value
system?
A. Mission, ethics policy, strategic initiatives
B. Vision, ethics policy, corporate bylaws
C. Values, QM/QI plan, utilization management plan
D. Mission , vision, values

2- The measures most indicative of the IDS'


ability to provide value to its stakeholders are:
A. Improvement in patient outcomes and reduced costs of car
256 of 268
B. An annual report with a positive bottom line
C. Improvements in patient outcomes and patient satisfaction
D. Reduced costs of care and competitive pricing

:Utilization review includes all of the following except -1040


A. Readmissions.
B. Post-hospital care.
C. Referrals.
D. Peer review.
Answer: B
Post Hospital care = follow up service
Referrals & readmissions measure premature discharge "underutilization" and
complication of care
Peer review, can judge Doctors level of utilization in comparison to peers

For effective UM, integration of data concerning all of the following -1041
:is mandatory except
a. Risk management.
b. Case mix.
c. Professional liability.
d. Severity of illness.

:Community case management program targets-1042


A. Acute cases.
B. Chronic cases.
C. Epidemic conditions.
D. Endemic conditions.

1043-Risk insurance coverage is a mechanism used by healthcare -


organizations as a means of:
A. Risk prevention.
B. Risk transfer.
C. Risk control.
D. Risk retention.

257 of 268
B

1044- Patient safety goals focus on:


A. Identifying problematic areas in healthcare.
B. Identifying means of patient satisfaction / dissatisfaction.
C. Describing critical / clinical pathways.
D. Describing expected healthcare personnel roles and responsibilities,

1045-The executive nurse of the ICU discovered that the night shift
nurse has prepared a unit dose of the medications required for each
patient in the unit, but she did not label the bottles, she only wrote the
patient names on the bottles. The best decision in such a situation is:
A. Give each patient the medication labeled with his / her name.
B. Send the bottles to the pharmacist to identify and label each.
C. Discard all the bottles immediately.
D. Bring another sample from each drug registered for each patient

1046-All of the following are challenge facing the implementation of


the EMR, but the most challenging is:
A. Cost
B. Resistance of physicians to change
C. complexity
D. Maintaining the confidentiality of patient information

1047-Incident reporting is a tool for:iq


A. Early warning
B. Establishing a penalty system.
C. Identification of organizational threats and opportunities.
D. Performance of gap analysis.

1048-Incident reporting is indicated for all of the following except:


A. Death of a recipient of care.
B. Attempted suicide of a recipient of care.
C. Increase post-surgical infection rate.
258 of 268
D. Medication error.

1049-A process variation that did not affect an outcome, but its
recurrence carries the risk of an adverse outcome is a / an:
A. Near miss.
B. Adverse event.
C. Potential compensable event.
D. Negligence.

A
1050-pain management is one of the major component of
a. Home care
b. Hospice care
c. Emergency care
d. Ambulatry care

1051-A company comprising 500 employees, negotiated the delivery of


comprehensive package of health services to its employees during the
coming year, to be paid as a fixed rate on monthly basis. This is
mechanism of payment is:
a. Case rate.
b. Per-diem charges.
c. Capped rate.
d. Capitation rate.

1052-A patient was admitted to El-Salama hospital suffering from liver


insufficiency which required treatment for an average of one week.
during his stay, the patient developed broncho-pneumonia which
prolonged his stay for 3 more days. The insurance company denied
responsibility for the extra 3 days, this is justified under:
a. DRG system.
b. Capitation system.
c. Capped rate system.
d. Case rate system.

1053-The insurance mechanism which entails paying a fixed rate per


visit by the insured, while the insurer covers the rest of the required
expenses according to the care provided is:
259 of 268
a. Capitation.
b. Co-payment.
c. Deductible.
d. Fee- for service.

1054-After 3 days of an abdominal surgery, the patient developed


severe pain. The ultrasonography revealed the presence of some
surgical gauge in the patient's abdomen. This state of negligence
refers to:
a. Expanded liability.
b. Corporate liability.
c. Res Ipsa Loquitur
d. Tort liability.

Discharge planning for elective procedures differ from traditional -1055


:discharge planning in being conducted

.Directly after patient admission a.

.Before patient discharge b.

.Before patient admission c.

.After patient discharge d.

Disease management is a coordinated system of healthcare -1056


interventions for populations with all of the following characteristics,
:except

.Significant potential for improvement a.

.Treatment by both primary care and specialist physicians b.

.High risk of sudden changes c.

.No variation in practice patterns d.

260 of 268
Discovering the need for dealing with the problem of "look alike -1057
drugs" due to repeated medication errors, the QP proposed the
administration of unit-dose medications. This approach is considered a
:means for

.Risk avoidance a.

.Risk transfer b.

.Risk prevention c.

.Risk prediction d.

An 80 years old patient fell while being transferred from his room -1058
to the radiology department. The patient developed fracture hip due to
:the fall. This event is considered

.An act of mal-practice a.

.A potentially compensable event b.

.An adverse patient occurrence c.

.Negligence of conduct d.

A healthcare organization decided to adopt a new culture to -1059


enhance patient safety, the top management should concentrate on all
:of the following except

.Changing individual behaviors a.

.Changing the organization mission, vision and goals b.

.Enhancing the organization infra-structure c.

.Changing systems and processes d.

:to re-enforce a "Just" culture-1060


261 of 268
.Seek out and punish staff members involved in reckless behaviors a.

.Avoid the discussion of errors to avoid blame b.

Discuss errors on the level of leaders , top management and c.


.department managers

Support the organizationwide discussion of errors and communicate d.


.lessons learned

1061-A chest x-ray was prescribed for an 80 years old patient with renal
insufficiency. on the way to the x-ray department; the patient-being
unattended- slipped and fell. The best decision in such a situation is:
A. Tell the patient's family the whole truth and offer them an apology and
appropriate compensation.
B. Ignore the issue, until addressed by the family.
C. Manipulate the circumstances to avoid organizational blame.
D. Seek the responsible and punish him /her to satisfy the family.

1062-All of the following information is crucial for enhancing safe


medication except:
A. Drug standard dosages.
B. Drug means of storage.
C. Drug labels.
D. Drug chemical constituents.

1063- Read-back process involves:


a. The donor of the information repeats the message twice.
b. The recipient of the message repeats the received order.
c. The donor of the message must first have it written down.
d. The recipient receives only written messages.

1064-Apanel of care providers decided on a certain treatment


plan for a patient, the plan included some indispensable drugs
that may cause some adverse reactions, the best choice in
such a situation is:
A. Avoid informing the patient about the possible adverse

262 of 268
reactions to ensure his compliance.
B. Inform the patient of the whole plan and discard it if he disagree.
C. Apply the treatment plan and inform the patient if any of the
expected adverse reactions did occur.
D. Discard the whole plan without informing the patient and shift to
other medications even if it did not produce the desired effect.

1065-Pre-operative verification process includes all of the


following, except:
A. Correct person
B. Correct procedure
C. Correct diagnosis
D. d.Correct site

1066-Plan of patient care is based-on:


A. Patient needs and values.
B. Benchmarking standards.
C. Professional credentials,
D. Accreditation standards.

1067-Prioritization of problems to be addressed is based on:


A. The potential for harm.
B. Organization mission, vision and code of ethics.
C. Professional credentials.
D. Level of care.

1068-Among the most important contributing factors to safety is:


A. Effective communication.
B. Patient's age.
C. Efficient reporting mechanisms
D. Availability of patient safety committee.

263 of 268
A

1069-Patient safety goals must be consistent with all of the


following, except:
A. Accreditation standards
B. Organization mission
C. Patient needs
D. Staff credentials

1070-The primary safety goal is to prevent:

A. Negligence.
B. Patient fraud.
C. Accidental injury.
D. Sentinel events.

1071-In reducing medication errors, CPOE involves alerting the


Prescriber to all of the following except:
A. Exceeding the upper ceiling of the drug.
B. The in efficacy of the drug to patient condition.
C. The patient being allergic to the drug.
D. The interaction of the prescribed drugs with others the patient is
receiving.

1072- BPOC serves to prevent medication errors in the:


A. Administration phase
B. Transcribing phase
C. Prescribing phase
D. Dispensing phase

1073-Incident reporting is performed by:


A. CEO.
B. Top management.
C. Quality council.
D. Any employee.
264 of 268
D

1074-An event that entails zero % acceptability is regarded as:


A. An adverse event.
B. A sentinel event.
C. A near miss
D. A potential compensable event

1075-A 24 years old woman delivered a full term female infant in the
maternity ward of a public hospital. The following day the mother and
child were discharged. On reaching home the mother discovered that
the hospital has given her a baby boy instead. On investigation it was
discovered that this event has happened twice before. Read this story
then answer questions 1 & 2:

1-This event should be investigated as:


An act of negligence.
An adverse event.
A sentinel event.
A potential compensable event.
265 of 268
2-The recommended way to deal with such event is to:
Report and perform a root cause analysis.
Compensate the family and conceal the occurrence.
Find who is responsible and punish.
Put control barriers to prevent recurrence of such event.
A

1076-A sentinel event is regarded as a:


a. Common cause variation.
b. assignable variation.
c.Noise.
d.Random variation.

1077-Removal of the left leg, when the right leg was the diseased part
that required removal, would be seen legally as

A. non compis B. quid pro quo


C. res Judicata D. res ipsa loquitor

1078-Grouped healthcare providers competing in a given geographic


region would most likely be called

A. case management B. managed care


C. managed competition D. prospective payment

1079-A managed care plan is said to have undergone favorable selection


when it(s)

A. members are sicker than the average person in the capitated population
B. members are healthier than the average person in the capitated population
C. makes less money than previous years
D. makes more money than previous years
Answer B

266 of 268
The most accepted answer is the cost effective but here no mix of
answers (Money +Health) so we will go for Health

1080-Providers who traditionally serve large numbers of low income and


uninsured people are termed

A. indemnity health plan


B. provider sponsored organization
C. point-of-service option
D. safety net providers

D
Which of the following situations best describes the term -1081
?“Misuse” of Resources at healthcare facilities

a) Patients receive appropriate medical services that are provided poorly,


.exposing them to added risks of preventable complications

b) Patients undergo treatment or procedures from which they do not


.benefit

.c) Patients do not receive beneficial health services

d) None of the above

Mortality reviews are a critical element of Risk Management and -1081


Quality Improvement, conducted to determine

A. if the practitioner(s) involved was/were appropriately licensed and


.credentialed

.B. if treatments and patient care were adequate and appropriate

C. who was responsible for the mortality and what disciplinary actions need to
.be taken

.D. what the unit staff was doing at the time of the mortality

The first assessment step the CPHQ makes to prevent risks to the -1082
patients, the staff, or the organization is to

267 of 268
.A. evaluate the corrective actions

.B. identify potential risks

.C. analyze the risks

.D. create an evaluation team

An organization-wide early warning system should be in place to -1083


screen patients and identify

.A. departments that carry the highest risk

.B. high-risk incidents that occurred within the past 24 hours

.C. assessments that should be conducted when a risk incident occurs

.D. adverse patient occurrences and potentially compensable events

Utilization management assessments measure-1084

.A. how practitioners utilize Performance Improvement data

.B. the use of services, procedures and facility resources

.C. how employees utilize outcomes and manage daily responsibilities

.D. the use of high-risk equipment and the level of risk

268 of 268

You might also like